Download as pdf or txt
Download as pdf or txt
You are on page 1of 333

SOLUTIONS MANUAL FOR

ORGANIC
CHEMISTRY:
AN ACID-BASE
APPROACH, SECOND
EDITION
by

Michael B. Smith
SOLUTIONS MANUAL FOR
ORGANIC CHEMISTRY:
AN ACID-BASE
APPROACH, SECOND
EDITION

by
Michael B. Smith

Boca Raton London New York

CRC Press is an imprint of the


Taylor & Francis Group, an informa business
CRC Press
Taylor & Francis Group
6000 Broken Sound Parkway NW, Suite 300
Boca Raton, FL 33487-2742
© 2016 by Taylor & Francis Group, LLC
CRC Press is an imprint of Taylor & Francis Group, an Informa business

No claim to original U.S. Government works

Printed on acid-free paper


Version Date: 20151027

International Standard Book Number-13: 978-1-4822-3827-3 (Ancillary)

This book contains information obtained from authentic and highly regarded sources. Reasonable efforts have been made to publish reliable data and
information, but the author and publisher cannot assume responsibility for the validity of all materials or the consequences of their use. The authors and
publishers have attempted to trace the copyright holders of all material reproduced in this publication and apologize to copyright holders if permission
to publish in this form has not been obtained. If any copyright material has not been acknowledged please write and let us know so we may rectify in any
future reprint.

Except as permitted under U.S. Copyright Law, no part of this book may be reprinted, reproduced, transmitted, or utilized in any form by any electronic,
mechanical, or other means, now known or hereafter invented, including photocopying, microfilming, and recording, or in any information storage or
retrieval system, without written permission from the publishers.

For permission to photocopy or use material electronically from this work, please access www.copyright.com (http://www.copyright.com/) or contact
the Copyright Clearance Center, Inc. (CCC), 222 Rosewood Drive, Danvers, MA 01923, 978-750-8400. CCC is a not-for-profit organization that provides
licenses and registration for a variety of users. For organizations that have been granted a photocopy license by the CCC, a separate system of payment
has been arranged.

Trademark Notice: Product or corporate names may be trademarks or registered trademarks, and are used only for identification and explanation
without intent to infringe.
Visit the Taylor & Francis Web site at
http://www.taylorandfrancis.com
and the CRC Press Web site at
http://www.crcpress.com
Chapter 02
Solutions Manual
Organic Chemistry. An Acid-Base Approach 2nd Edition
27. (a)
O H O
N O OH2 N O H3O+
O O
Ka = [NO3–] [H3O+]
[HNO3] [H2O]

Nitric acid is a stronger acid than water, so water is the base and nitric acid is the acid in this
reaction to generate the nitrate anion (conjugate base) and the hydronium ion (conjugate acid).
(b)
H H H
NH3 O H-NH3
O

Ka = [HO–] NH4+]
[H2O] [NH3]
Water is a stronger acid than ammonia, so ammonia is the base and water is the acid in this
reaction. The reaction products are hydroxide anion (conjugate base) and the ammonium ion (conjugate
acid).
(c)
Br H-NH3
Br H
NH3

Ka = [Br–] [NH4+]
[HBr] [NH3]
Hydrobromic acid is a stronger acid than ammonia, so ammonia is the base and HBr is the acid
in this reaction to generate the bromide anion (conjugate base) and the ammonium ion (conjugate acid).
(d)

Cl H OH2 Cl- H3O+

[Cl -] [H 3O+]
Ka =
[HCl] [H 2O]
Hydrochloric acid is a stronger acid than water, so water is the base and HCl is the acid in this
reaction to generate the chloride anion (conjugate base) and the hydronium ion (conjugate acid).
(e)
Cl Cl
Cl C NH3
Cl C H
Cl NH2 Cl
Na
Ka = [CCl3–] [NH3]
[HCCl3] NaNH2]
Note that the acidic hydrogen atom is attached to a carbon atom. The amide anion is clearly a
base, which a very strong base. Therefore, the strong base can react wit the weakly acidic “carbon
acid”. In other words, NH2– is the base and chloroform is the acid in this reaction that generates the
anion –CCl3 (the conjugate base) and ammonia, NH3, which is the conjugate acid.

28. The main reason is likely the relative size of the bromide ion (182 pm or 1.82 Å) versus the chloride
ion (100 pm, or 1.00 Å). Greater charge dispersal for the bromide ion leads to greater stability of that
conjugate base, and a larger Ka.

29.
[H3O+] [Cl–] [H3O+] [Cl–]
Ka = Ka =
[HCl] [HCl] [H2O]
In this reaction, water is the base that reacts with the HCl. If water is omitted, the base has been
excluded from the equilibrium expression for an acid-base reaction.

30. (a) NH3 (b) H3CO– (c) –NO3 (d) Br– (e) –:NH2 (f) –CH3

31. The ‘ate. complex in the “box” is the reaction product, where N of ammonia is the electron donor
(the base).

H3N: BF3 F3B NH3

32. Oxygen is more electronegative than nitrogen, so nitrogen is more electron-rich, and will be a better
electron-donor. In addition, the ‘ate’ complex from ammonia is an ammonium salt whereas water will
react to form an oxonium salt. The ammonium salt is more stable, which contributes to the overall
increased Lewis basicity of the nitrogen atom in ammonia.

33. In these neutral molecules, phosphorus is larger than nitrogen, with covalent radii of 106 and 71 pm
(1.06 Å and 0.71 Å), respectively. The charge density of nitrogen is greater. Therefore, ammonia is
expected to be the stronger Lewis base.

34. An ate complex is the salt generated by reaction of a Lewis acid with a Lewis base. The atom
derived from the Lewis base expands its valence and assumes a positive charge, whereas the atom
derived from the Lewis base expand its valence and assumes a negative charge.

35.
H3C Cl
O Al Cl
H3C Cl

36. The C—H bond is much stronger than the N—H bond, so it is more difficult to break. Nitrogen is
larger and better able to accommodate charge relative to carbon, so H3C- is significantly less stable
(more reactive) than H2N-. In addition, nitrogen is more electronegative than carbon, so the amide anion
is more stable, which means that it is less reactive (it is a stronger base). If the -NH2 conjugate base is
more stable, Ka is larger, and ammonia is a stronger acid.

37. (a) CH3OH is the strongest acid in this series. The O—H bond is more polarized and easier to
break, and the methoxide anion, H3CO-, is more stable than the anions from CH4 or CH3NH3. NaF does
not have an acidic proton, and it is not a Brønsted–Lowry acid.
(b) As explained in section 2.4, the size of the conjugate base increases from fluoride towards iodide, so
the iodide in is more stable. This means that Ka is larger for HI and decreases going towards HF. Since
iodide is much larger, the H—I bond is longer, and weaker, s it is easier to break relative to the others.

38. The iodide in is much larger, and the charge is dispersed over a greater area. Therefore, it is more
difficult for iodide to donate electrons to an acid relative to fluoride. In other words, iodide is a weaker
base.

39.
O H O O O
N O BASE H-BASE N O N O N O
O O O O
As shown, nitric acid generates the resonance stabilized nitrate anion. In the nitrate anion, the
charge is dispersed over several atoms, which makes it more difficult for that species to donate electrons
to an acid. For hydroxide ion, HO-, the charge is concentrated on the oxygen atom, and it is much easier
to donate electrons. The charge is not dispersed as in the nitrate anion, and hydroxide is more basic.

40. The fluoride is much more stable relative to the methide anion, H3C:-, because fluoride is more
electronegative. Therefore, it retains electrons whereas the less electronegative carbon does not.
Therefore, carbon will donate electron more easily and the methide anion is the stronger base.

41. Determine the pKa for each of the following.


(a) Ka = 1.45x105 (b) Ka = 3.6x10–12 (c) Ka = 6.7x10–31 (d) Ka = 18 (e) Ka = 3.8x1014
pKa = -log10 Ka. (a) 5.16 (b) 11.44 (c) 30.17 (d) –1.26 (e) –14.6

42. The more acidic acid will have the smaller pKa. Of this series, HCl is the strongest acid (pKa -7)
relative to HF (pKa 3.17). Water has a pKa of 15.7 and ammonia has a pKa of about 25. Clearly, HCl
has the smallest pKa.
43. The least acidic acid will have the largest pKa. Of this series, HCl is the strongest acid (pKa -7)
relative to HF (pKa 3.17). Water has a pKa of 15.7 and ammonia has a pKa of about 25. Clearly,
ammonia is the least acidic and has the largest pKa.

44. NaF is an ionic salt, Na+ and F-. The electron rich fluoride ion is the only atom of these two that
can donate electrons, so F is the basic atom.

45. Clearly, water is more acidic. Figure 2.5 shows that oxygen is more electronegative than sulfur, so
the O—H bond is more polarized than the C—S bond, and should be weaker. This is consistent with a
larger Ka for water (smaller pKa). Further, the more electronegative oxygen atom in the HO- anion
makes it a poorer electron donor relative to HS-, so hydroxide is the stronger base. This is also
consistent with a larger Ka for water.

46.
• •

•  •
•  •

•  •

•  •
•  •
O O
• •

O
A C C
C H • •
H3C
H3C O

• •
H3C O O
•  •

• •
• •
•  •

• •

• •
•  •
•  •

•  •

O
•  •

• •
O O

•  •
C H C C H
B H C H
H3C C H 3 H3C
H H H
The reaction of A gives the conjugate base shown, and B gives the conjugate base indicated. In
both cases, the charge is dispersed over three atoms (resonance). The OH bond is weaker than the NH
bond, and that proton is easier to remove. Oxygen holds onto electron better than nitrogen (it is more
electronegative), so the conjugate base from A is less likely to donate electrons (it is more stable, which
shifts the equilibrium towards the conjugate base). Although it is not obvious from the diagram, the
charge dispersal is more efficient in the conjugate base from A. All of these combine to make A much
more acidic (pKa of 4.8 versus 46).

47.
O O O
+ NaOH C C + HOH
C H H3C
H3C O H3C O O

H H
H H
H + NaOH + HOH
H O H O
(a) The conjugate base derived from formic acid is resonance stabilized by charge dispersal over several
atoms, as shown. The conjugate base from methanol has the charge concentrated on oxygen, and no
charge dispersal is possible.
(b) If there is a larger concentration of the conjugate base, the equilibrium is shifted toward the right
(towards the conjugate base), and Ka is larger.
(c) If Ka for formic acid is much larger, it will be the stronger acid, and will react better with NaOH.
48. Since F is more electronegative than B, one predicts that C—F is the more polarized covalent bond.
49. Aluminum has d orbitals and boron does not. According to the rules presented for Lewis acid
strength, the d orbitals are more available for donation, which makes AlCl3 the weaker Lewis acid

50.

NH4Cl + NaOH NH3 + HOH + NaCl


conj. conj.
base acid
Chapter 03
Solutions Manual
Organic Chemistry. An Acid-Base Approach 2nd Edition

22. Al is 1s22s22p63s23p1 He is 1s2 Be is 1s22s2 Mg is 1s22s22p63s2


Cl is 1s22s22p63s23p5 Br is 1s22s22p63s23p63d104s24p5 Ti is 1s22s22p63s23p63d24s2
Cu is 1s22s22p63s23p63d104s1

23.

1s 2s

24. Both are group 1 elements. Potassium is a larger atom, which means that the 4s1 electron is held
less tightly than the 3s1 electron on sodium. Using a very simple rationale, it should be easier to lose the
electron from potassium, and the resulting K+ ion is larger than the Na+ ion and will be more stable.

25. No single orbital may hold more than two electrons, and if there is more than one orbital of the
same energy (degenerate orbitals), no one orbital may hold two electrons until all orbitals hold one. For
the second row, there is one s-orbital and three degenerate p-orbitals. Therefore, adding one electron to
the 2s-orbital yields lithium (Li) and adding the second to that orbital yields beryllium (Be). Adding one
electron to a 2p-orbitals gives boron (B), and the second electron goes to a different 2p-orbital to give
carbon (C), and the third fills the last 2p-orbitals to yield nitrogen (N). The next electron will spin pair
in a 2p-orbital to give oxygen (O) and the net electron will spin pair to give fluorine (F).

26. The four hydrogen atoms about carbon in methane arrange as far as possible from each other, with
carbon in the middle: a tetrahedron. Ammonia has three atoms and a lone electron pair, with nitrogen in
the middle of a tetrahedron. Since the electron pair cannot be seen, the shape of the three hydrogen
atoms and the nitrogen constitutes a pyramidal structure.

27.
sp3 sp3
sp3 sp3
sp3 sp3 sp3
sp3 O Cl CH2CH3
Br Br sp3
(a) C (b) CH3 C CH3 (c) CH3 C N (d) C
Br CH2 CH2 Br
Br CH=CH2
The sp3 hybridized carbon atoms are indicated by the arrows.

28.
H Br
(a) CH3O- Na+ (b) C (c) CH3C≡C-H (d) CH3C≡C-Na (e) NaF
H
H
Ionic Covalent Covalent Ionic Ionic
O CH2
(f) CH3—CH3 (g) (h) (i) O
C C H

Covalent Covalent Covalent Covalent

29. The numbers indicate that it requires less energy to remove an electron from K (4.341 eV, 124.3
kcal mol-1), so it is expected to be the most reactive. Since Li (5.392 eV, 113.5 kcal mol-1) has the
highest ionization potential, it is more difficult to lose an electron and it will be less reactive than Na
(5.139 eV, 100.4 kcal mol-1), the least reactive of the three. Therefore, K is more reactive than Na,
which is more reactive than Li.

30.
H H
Cl H H H C
(a) C Cl (b) C O (c) H O
Cl H C
Cl H H
H
H
H H H H
C H H
(d) H H C H C
H N C (e) C (f) C H
C H
H C C H H Cl Cl
H H H H H
H HH
.

31. ΔH° = H°products - H°reactants


(a) ΔH° = (H°C-Br + H°H-O) - (H°H-Br + H°C-O)
ΔH° = (67 + 104.2) - (87.4 + 257.3) = 171.2 - 344.7 = -173.5 kcal mol-1.
(b) ΔH° = 2xH°C-I - (H°C-C + H°I-I)
ΔH° = 2x50 - (145 + 36.5) = 100 - 181.5 = -81.5 kcal mol-1.
(c) ΔH° = (H°C-O + H°C-H) - (H°O-H + H°C-C)
(257.3 + 80.6) - (104.2 + 145) = 337.9 - 249.2 = 88.7 kcal mol-1.
(d) ΔH° = (H°C-I + H°N-H) - (H°C-N + H°H-I)
ΔH° = (50 + 75) - (184 + 71.4) = 125.0 - 255.4 = -130.45 kcal mol-1.

32. I—CH3 (C—I = 56 kcal or 234 kJ mol-1); Br—CH3 (C—Br = 70 kcal or 293 kJ mol-1);

Cl—CH3 (C—Cl = 84 kcal or 351 kJ mol-1.


Iodine is larger than Br, which is larger than Cl. The C—I bond distance is the greatest, and the
C—Cl is least, which correlates with the weaker C—I bond and the stronger C—Cl bond.

H H H
H
H H
H C C C
C H
H C
C H C H
H
H H H H H
H
33. Propane Butane

34.
(a) C N (b) N O (c) C H (d) Cl Br (e) B C
(f) Li C (g) C F (h) N H (i) H Cl

35.
(a) C N (b)
N O (c) C H
Polarized covalent Polarized covalent Nonpolarized covalent

(d) C F (e) C C (f) Li C


Polarized covalent Nonpolarized covalent Polarized covalent

36. The covalent bond formed from two sp3 hybridized carbon atoms is predicted to be relatively strong
when compared to a polarized covalent bond (it is much weaker than an ionic bond). Since it is not
polarized, the electrons are distributed equally between the two carbon nuclei. This symmetrical bond
with most of the electron density between the nuclei is predicted to be strong.

37.
H Cl
Cl O
(a) (b) (c) C Cl
C H C H
H Cl
H H Cl
H
4 Polarized bonds, but
they all cancel.
Dipole moment = 0
CH3 Cl H

(d) (e) (f) C Cl


C CH3 C Br
H3C H Cl
CH3 H Cl
No polarized bonds.
Dipole moment should be 0
38. After loss of one electron, the electronic configuration of the ion F+ is 1s22s22p4

39.

s-Orbital p-Orbital Hybrid orbital d-Orbital

40. Iodine is much larger than fluorine, so the C—I bond distance is greater than that of C—F. Based
on this simple analysis, the C—I bond should be weaker than the C—F bond because there is less
electron density per unit distance between the nuclei. Based on this simple analysis, the C—I bond
should be more reactive with a nucleophile in a chemical reaction.

41. Lithium tends to form Li+ with loss of one electron in the outmost shell, to give the Nobel gas
configuration, which is quite stable. Fluorine is in group 17 tends to form an anion by gaining one
electron to give the Noble gas electronic configuration, which is more stable. Therefore, LIF tends to
form an ionic bond.
Carbon is in the middle of the periodic table and electron transfer is not energetically feasible,
but sharing electrons in a covalent bond is energetically practical (i.e., lower in energy).

42. A 2p-orbital has a dumb-bell shape , whereas the 4s-orbital is spherical.

43. The C—C bond is the least polarized because there is no heteroatom, and no difference in
electronegativity of the bonded atoms. In C—N, C—O and C—F, one atom is more polarized than the
other, leading to a polarized covalent bond.

44. Sodium (Na) has the electronic configuration 1s22s22p63s1. Loss of one electron generates the ion,
Na+, so the electronic configuration is 1s22s22p6.

45. There is greater difference in energy between 1s- and 2p-orbitals than between 2s- and 2p-. Mixing
a 1s- and a 2p-orbital is expected to generate a higher energy molecular orbital, if it can form at all,
which is unlikely.

46. The C—O bond will be a sp3 hybrid orbital in a σ-covalent bond, similar to that formed for a C—C

bond. It will have the familiar hybrid orbital shape:


Chapter 04
Solutions Manual
Organic Chemistry. An Acid-Base Approach 2nd Edition

25.
Butane Decane

Tridecane

26.
(a)
Longest chain = 4 Longest chain = 3 Longest chain = 5
Longest chain = 4 propane pentane
butane
butane
CH2 CH3 H3C CH3 CH2 CH2
CH2 CH3 H3C CH CH H3C CH CH3
H3C CH2
CH3 CH3 CH3

C5H10 C6H14 C5H8 C100H202 C60H120

C5 = pent C5 = pent
(b)

27. (a) Structures (a)-(d) + (g) have the same empirical formula, and are isomers.
CH3
CH2CH3
CH2 CH2 CH2 CH3 (c) CH2 CH2 CH
(a) (b) CH2 CH CH3 H3 C CH2 CH2 CH3
H3C CH2 CH2 CH2
H3 C CH2 CH2
C8H18 C8H18
C8H18
CH2
CH3 CH2 CH3 CH3
(d) CH2 CH CH2
H3C CH3
(e) CH2 CH CH2
H3C CH2 CH2 CH3 H3C C CH2 CH3 (f) CH2 (g)
CH2 CH2 CH2 H3C C CH3
CH3 CH CH2 H3C CH3
C8H18 CH2 CH2 CH3
C9H20
C8H18
C10H22
(b) The isomers with the formula C6H14 are marked.
Isomers

C6H12 C6H14 C6H14 C8H18 C6H14

28.
CH2CH3 CH2CH3
CH2 CH2 CH2 CH3 CH2 CH CH2 CH
(a)
H3C CH2 CH2 CH2
(b) H3C CH2 CH2 CH2 CH2CH3
Octane
3,7-Diethyldecane
CH2
CH2 CH3
(c) H3C CH3 CH2CH2CH2CH3
H3C C CH2 CH CH2
(d) CH2
CH2 CH2 CH2 CH3
CH2 CH2 CH2 CH2 CH CH2
3,3,7-Trimethylundecane
H3C CH2 CH2 CH2 CH2 CH2 CH3
6-Propylpentadecane

29.

(a) (b)

1,1,4,4-Tetramethylcyclohexane 3,9-Dimethyl-8-(2,2-dimethylpropyl)nonadecane

Br
(c) (d)
Octane
6-Bromo-6-ethyl-3-methyldecane
30. (a) Isomers of 1-bromooctane will have the formula C8H17Br.
Several are shown, but there are others.
Br

Br
Br
2-Bromo-6-methylheptane 3-Bromo-4-methylmethylheptane 2-Bromo-2-3-dimethylhexane
Br Br
Br

3,3-Dimethyl-4-bromo-hexane 2-Bromo-2,4,4-trimethylpentane 3-Bromo-2,3,4-trimethylpentane

(b) With a formula C6H12, assume they are cyclic alkanes. Several are shown, but there are others.

Cyclohexane Methylcyclopentane 1-Ethylcyclobutane

1,2-Dimethylcyclobutane 1,1,3-Trimethylcyclopropane

31. (a) 3,5-Dimethylheptane has a formula C9H20.

C9H20 C9H20 C8H18


C9H18

(b) Isomers of cycloheptane will have the formula C7H14.

C7H16 C7H14 C7H14 C7H16


32.
(a) 1,2,3-Triethylcycloheptane (b) 3,4-Dichloro-5-(3-methylbutyl)hexadecane
Cl
Cl

(c) 1-Chloro-2,2,4,4-tetramethylhexane
(d) 2,2-Dibromo-3-methyloctane

Cl

Br Br

(e) 1,1-Diethylcyclohexane (f) 5-(1-Methylpropyl)decane

33.
A sample weighing 0.6000 g was burned in the presence of oxygen to yield 0.7692 g of
water and 1.8827 g of CO2.
g C = 0.2727 x 1.8827 = 0.5134 g and g H = 0.1111 x 0.7692 = 0.0855 g
Assume the molecule is a hydrocarbon, so
%C = 0.5134/(0.5134 + 0.0855) = (0.5134/0.5989) x 100 = 85.81%
%H= (0.0855/0.5989) x 100 = 14.28%
Assume 100 g, so there are 85.81 g of C and 14.28 g of H
mol C = 85.81/12 = 7.15 and mol H - 14.28/1 = 14.28
so, 14.28/7.15 = 2H/C = CH2 = any cyclic alkane formula CnH2n
34. For a 5-carbon chain, use pent-.

35. In ethylcyclohexane, the substituent chain has fewer carbon atoms than the ring, so the base name is
cyclohexane. In cyclohexyldodecane, the long chain has more carbon atoms than the ring, so the base
name is dodecane.

Ethylcyclohexane 1-Cyclohexyldodecane
36. Calculate the % C and H, as well as an empirical formula for each of the following using the
combustion analysis provided.
(a)
A sample weighing 0.52 g was burned in the presence of oxygen to yield 0.6688 g of
water and 1.6344 g of CO2.
g C = 0.2727 x 1.6344 = 0.4457 g and g H = 0.1111 x 0.6688 = 0.0743 g
Assume the molecule is a hydrocarbon, so
%C = 0.4457/(0.4457 + 0.0743) = (0.4457/0.5200) x 100 = 85.71%
%H= (0.0743/0.52) x 100 = 14.29%
Assume 100 g, so there are 85.71 g of C and 14.98 g of H
mol C = 85.71/12 = 7.14 and mol H - 14.29/1 = 14.29
so, 14.29/7.14 = 2H/C = CH2 = any cyclic alkane formula CnH2n

(b)
A sample weighing 0.81 g was burned in the presence of oxygen to yield 0.8578 g of
water and 2.6208 g of CO2.
g C = 0.2727 x 2.6208 = 0.7147 g and g H = 0.1111 x 0.8578 = 0.0953 g
Assume the molecule is a hydrocarbon, so
%C = 0.7147/(0.7147 + 0.0953) = (0.7147/0.81) x 100 = 88.23%
%H= (0.0953/0.81) x 100 = 11.77%
Assume 100g, so there are 88.23 g of C and 11.77 g of H
mol C = 88.23/12 =7.35 and mol H - 11.77/1 = 11.77
so, 11.77/7.35 = 1.6H/C = CH1.6 x 2 = C2H3.2 or x 3 = C3H4.8 or x4 = C4H6.5 or x 5 = C5H8
The last one makes sense, and matches the alkane formula CnH2n+2, so, C5H8.

37.

Cl Cl
(a) (b) (c) (d)

4,4-Dichloro-2,6-dimethylheptane 1,2-
Dimethylcyclohexane
1,1,3,3-Tetramethylcyclooctane 1,3,5-Triethylcyclohexane

F
F
(e) F (f) (g) C
H
Nonane H H
F Fluoromethane
6-(1,1-Diethylbutyl)-3,3,7-trifluorotetradecane
Cl
Br
Cl
(h) (i)
(j)

2-Bromo-4-ethyl-3-methylheptane
6-Ethyl-5-(1,1-diethylpropyll)-2,2-dimethyltridecane
4,4-Dichloro-6-ethyl-7,7-di(1,1-
dimethylethyl)-tridecane

38.
Cl Cl

(a) (b) (c)

Br
6-Ethyl-3,4,4-timethylnonane 3-Chloro-6-ethyl-2-methylnonane 1-Bromo-3-chlorocyclohexane

I
(f)
(d) (e)

3-Iodo-2,2,4,4-tetramethylpentane

7-(1,1,2-Trimethylbutyl)-6,10-diethyl- 1,1,2-Triethylcycloheptane
2-methyltetradecane

39. A and F and G are identical, octane. B and E are identical, 3-ethylhexane. C and I are identical,
2,5-dimethylhexane. D and H are identical, 3,3-dimethylhexane.
There are four different structures of A-I, which are isomers, all with the formula C8H18.

A B C D

G H I

E F
40.

Nonane 3-Methyloctane 4-Methyloctane

2-Methyloctane
2,4-Dimethylheptane 2,6-Dimethylheptane

3,3-Dimethylheptane
2-Ethylheptane 4-Ethyl-2-methylhexane
2,3-Dimethylheptane

41.
(a) 3,3,5-Trichlorodecane (b) 2,6-Dimethyl-3-fluoroheptane

Cl
Cl Cl

F
(c) 2,2-Dichloro-4,4-dibromooctane
Br Br Cl Cl
Chapter 05
Solutions Manual
Organic Chemistry. An Acid-Base Approach 2nd Edition

59.
H H
O CH2 sp2
CH3 C CH2 CH
Br Br 2 Cl CH2CH3
sp3 C sp C sp3 C H (f)
(a) C (b) CH
Br 3 2 CH2 (c) C (d)
Br
(e)
H sp2 C
CH CH
sp Br Osp3 CH2
N sp sp3 CH=CH2
CH3 CH3 C H
H sp H
2

60. (a) C10H20 (b) C20H42 (c) C8H14 (d) C16H34 (e) C9H16 (f) C100H200
(a) alkene or cyclic alkane; (b) alkane; (c) alkyne; (d) alkane; (e) alkyne; (f) alkene or cyclic alkane

61.
(a) CH3CH2CH2CH2CH2CH3 (b) (CH3)2CHCH2CH2CH3

(c) CH3CH2CH2CH2CH2CH=CH2 (d) CH3CH2C≡C(CH2)8CH3

62. Structures (a), (c) and (d) have resonance forms that involve a C=N, C=O or C=Cl structure,
respectively.
H H
H3C H H •• • • H H H C H
H3C H 3
••

N O Cl O C N
••

(d) (b) (f)


(a) (c) (e)
C C C C C C
H H H H H H H3C CH3 H 3C CH3
H3C CH3 H H
Structure (b) is an oxocarbenium ion, and the adjacent carbon is sp3 hybridized. All valences are
satisfied, and there is not possibility of a C=O structure. The same is true for (f), where the nitrogen is
present as ammonium. Cation (e) cannot form a C=C unit to the adjacent sp3 hybridized carbon, which
has all valences satisfied.
63.
(a) C—N (b) N—O (c) C—H (d) Cl—Br (e) B—C

(f) Li—C (g) C—F (h) N—H (i) H—Cl

64.
(a) C N (b) N O (c) C H
Polarized covalent Polarized covalent Nonpolarized covalent
(d) C F (e) C C (f) Li C
Polarized covalent Polarized covalent
Nonpolarized covalent

65.
Cl H H Cl
(a) C H (b) C O (c) C Cl
H H Cl
H H Cl
H H Dipole is zero
(d) (e)
C H C Cl
Cl Br Cl Cl

66. Counting the electron pair, there are four different groups on N. If you make a model of both
structures, you cannot superimpose them: they are different. However, they rapidly interconvert by
fluxional inversion, so there is essentially a 50:50 mixture of both (see Sections 5.8.4 and 9.9.2).
H3C CH3
N °° °° N
(H3C)2HC CH(CH3)2
H3CH2C CH2CH3
67.
Lowest
Highest OH
OH
(a)

(i) OH
(ii) (iii)
NH CO2H OH
Lowest 2
(b) Highest

(i) (ii) (iii)


Lowest
(c) CH3Cl CH3NH2 CH3COOH Highest
(i) (ii) (iii)

68.

Br Br

69.
NH2 O
(a) (c)
(b)

Hydrogen-bonding London Dipole-dipole


OH
(d)
(e)
O

Hydrogen-bonding London

70. In (b), (c), and (d), the π-bond is directly connected to the negatively charged atom. Therefore, the
electrons on the negatively charged oxygen atom can be delocalized into the adjacent π-bonds, leading
to the resonance structures shown. Note that (b) and (c) are the same!
O
O

O CH2 O

O O–
(a) CH3O- (b) (c) (d)
O– CH2 O
71. B: H-CH2CN B:----------H-------CH2CN B:H + –CH CN
2
This carbanion is resonance stabilized, so the proton of acetonitrile is relatively acidic, and
relatively easy to remove if a strong base is used (see Chapter 22).
H H
C C N: C C N:
H H

72. Reactions (a) and (c) are two electron transfer reactions, whereas reaction (b) is a radical process,
which means that it proceeds by one electron transfer as shown.

(a) I CH3–Cl CH3–I Cl

(b) I CH3 I–CH3

(c) (CH3)3C Cl (CH3)3C–Cl

73. Reaction with a base generates alkoxide anion A. Resonance delocalization leads to the resonance
contributors shown, and the charge can be delocalized to the second oxygen, as in B. It is possible to
view B and A as an “extended carboxylate anion” because of the delocalization that is made possible by
the intervening π-bonds. This extension of reactivity by intervening π-bonds is known as vinylogy, and
tropolone is considered to be a “vinylogous” carboxylic acid. In other words, the OH proton of
tropolone is acidic in large part because the resulting conjugate base is resonance stabilized in a way that
allows the carbonyl group to be involved, much like the carboxylate anion seen in carboxylic acids.
O O O O O
O
OH base O O O O
O

A B

74. A and B are isomers. C and D and F are isomers. G and I are isomers. E and H have no isomers
C7H16O C8H16O C8H16O
C7H16O OH O

O H
A B O C D

HO O O
O O O
HO
E F OH G C6H14O2 H I
C8H18O C8H16O C8H16O2 C6H14O2

75. Note: The term E- should be used with (a) and also for (c), but this nomenclature will not be
discussed until Chapter 9. Therefore, for this problem, leaving out the E term is OK.
(a) 5-(2,2-dimethylbutyl)hexadec-2-ene (b) 4,5,6,7-tetraethyldodec-2-yne

(c) 7,8-di(1,1-dimethylethyl)pentadeca-1,3-diene (d) 1,3,3,5,5,6-hexamethylcyclohexene

(e) 1-cyclopropyl-2-ethylcycloheptene (f) 5,5-diethylnon-3-yne

76.
H3C(H2C)4

(a) (b) (c) (d)

(CH2)4CH3
4-Ethyl-4-methyloctane 6-Ethyl-7-methylnon- 6-Methyl-7-(3-methylbutan- 4,5,5,6-Tetramethyloct-3-ene
5-en-1-yne 2-yl)tridec-6-ene

(e) (f) (g) (h)

3-Ethyl-5-methylhex-1-yne 5-Ethyl-4,8-dimethyl- 7-Methyl-6-(tert-pentyl)dec-1-ene 2,4,4,5,5-Pentamethylhex-2-ene


nona-1,6-diyne
77.
I
Br
(a) (b) (c) (d) (e)

Br

3,3-Diethylcyclo- 4-Iodo-1,1-dimethyl-2- 4-(tert-bButyl)cyclo- 1,3,3,6,6-Pentamethyl- 3,4-Dibromocyclopent-1-ene


prop-1-ene ethenylcycloheptane hex-1-ene cyclohex-1-ene

Ph F

(f) (g) (h) (i) Ph


Ph

Prop-2-yn-1-ylcyclodecane 13-(1-methylethyl)-1,3-diphenyl- Cycloheptadecyne 4-Fluoro-4-phenyl-


cyclotridec-1-ene cyclonon-1-ene

Note: The (E/Z) nomenclature for certain alkenes is omitted, because those terms will not be
discussed until Section 9.6.3.

78. There are many answers for this question. Only 12 representative structures are provided.

1-Ethyl-2-methylcycloheptane Methylcyclononane Cyclodecane

1,2-Dimethyl-3-propylcyclopentane 1,1-Dimethylcyclooctane 1,3-Diethyl-2,4-dimethylcyclobutane

Dec-1-ene 7-Methylnon-2-ene 3-Methylnon-3-ene

6-Methylnon-3-ene 2,3,4,5-Tetramethylhex-1-ene 4,5-Dimethyloct-4-ene


79. Note: E/Z nomenclature is not introduced until Chapter 9, so it is omitted here.

Cl
Ph F
(a) (b) (c) (d)

F
5-Ethyl-5-phenyloct-(3Z)-ene 3-Chloro-4-ethyl- 1,1-Difluoro-3,4,4-trimethy- 6-Ethyl-4,5,5-trimethy-
oct-3-ene lpent-2-ene loct-1-ene

Br Ph
(e) (f) (g) C (h)
C Ph

4-Ethylhexa-1,2-diene 5-Ethyl-2,6-dimethyl- 7-Bromo-2-methyl-6-(1,1-dimethyl- 3,4,5-Trimethyl-5,6-diphenyl-


non-1-ene propyl)deca-2,3-diene hept-3-ene

Ph Cl
H3C(H2C)4 Cl
(i) (j) (k) (l)
Cl

(E)-5-isopropyl-2,3-dimethyldec-4-ene 3-Methyl-6-phenyl-3- 3,4-Diethylnon-1-yne 5,5,5-Rrichloro-2,4-dimethyl-


propylhex-1-yne penta-1,3-diene
Et Et
Ph
(m) (n) (o) (p) Ph

3,3,4-Triethyl-2-propyl- 3-Ethyl-5-phenyl- 4-(1-Methylbutyl)-4-ethyl- 4-(1-Phenylbutyl)-3,3-


hex-5-yn-1-ene hept-1,6-diyne 7-methyloct-2-yne dimethlnona-1,8-diene

Ph Ph H3C(H2C)4

(q) (r)

(CH2)4CH3
3-Ethyl-5,5-dimethyl-4,4- 3-Pentyl-4-hexyl-2,3-dimethyl-
diphenylhex-1-yne hept-5-4n-1-yne

80. The E/Z nomenclature is not discussed until Chapter 9, so it omitted here.
2,4-Dimethyl-3-phenylpentan-3-ol
OH Cl
HO Ph
Ph
(b) (d)
(a) (c)
Cl

3-Phenylhex-3-ene 2-Chloro-3-methylhex-2-ene
2-Chlorocyclopentanol
4-Ethylcyclohexanol OH
CH2(CH2)8CH3
(e)
(h)
CH2CH3 (f) (g)

HO 4-Methylhept-3-ene

4-Ethyl-3-methyltetradec-3-ene 7,10-Dimethyldodecan-6-ol
4-Cyclopropyl-2-methylpentan-2-ol
OH
Br (l)
(k)
(i) OH
(j)
OH
Hexane-2,3-diol Br 3-Methyl-3-hept-3-ene
4,5-Dibromooct-4-ene
Ph OH F C≡C-H
(m)
(o) (p)
(n)

OH Hept-6-yn-2-ol
Hex-3-ene Cl
4-Phenyl-hex-3-en-2-ol 4-Chloro-3-fluorohept-3-ene

81.
Ph OH OH
(a) OH (b) (c)

Ph Hex-2-en-1-ol
2-Methylcycloheptan-1-ol
5,6-Diphenylheptan-2-ol
OH
OH OH
OH
(d) (e) (f)

OH
Cl
8-Chloro-5-(3-ethylheptyl)pentadecan-1-ol Heptane-3,4,5-triol 1,2,3,4,5,6-Hexamethylcyclohexan-1-ol

OH

(g) (h)
Cl OH
OH Ph
4-Phenyloctane-1,8-diol 3-Chloronon-8-en-1-ol
82.
H
N CH3
(b) (c)
(a) N N

1-Butylpiperidine
N,N-(1-Methylethyl)butan-1-amine N-Methylcyclohexanamine

Ph NHPh
(d) Cl N Cl
H2N (e) (f)
CH3
H
2-Phenylpentan-1-amine 1-Chloro-N-chloromethyl-
7,7-Dimethyl-N-phenyloctan-2-amine methanamine
(bis(chloromethyl)amine)

83.
O
O
CHO (c) (d)
(a) (b)
Ph O
2-Ethylhexanal 2-Methylhexan-3-one
2-Methylcyclobutanone 3-Methyl-4-phenylpentan-2-one

O
(e) Br (g)
(f) CHO

6-Bromo-5,5-dimethylhexan-3-one O
1-Ethylcyclohexanecarbaldehyde Dicyclohexylmethanone
(Dicyclohexyl ketone)
Cl CHO O
Cl O
(h) (i) (j)
H H H
Formal 4-Phenylbutanal
4,4-Dichloro-2,3,6-trimethyloctanal (Formaldehyde)
84. Draw the structure of the following molecules.
(a) 8-Phenyloctanoic acid (b) 3,3,6,6-Tetrabromohexadecanoic acid
CO2H
Br Br

Ph CO2H
Br Br

(c) 2,5-Dimethylhexanedioic acid (d) 3-Chlorocyclohexane-1-carboxylic acid


CO2H

CO2H
HO2C

Cl
CHO
(e) 2,5-Dimethylcyclopentanecarbaldehyde

85. A π-bond is formed by sideways overlap of two adjacent and parallel p-orbitals. A σ-bond is
formed by the direct overlap of two hybrid orbitals, and all of the electron density of the bond is
concentrated on a line between the two carbon nuclei. In a π-bond, only some of the electron density is
shared. If the shared electron density is less in a π-bond, it is a weaker bond.

86.
Methanesulfonic acid
O O H O O
H H H H
H C S O H C S O H C S O H C S O
H H O H O H
O O

H O H O H O
H
H C C O H C C O H C C O
H H H
Acetic acid
(a) Both products are shown. Methanesulfonic acid gives the methanesulfonate anion and acetic acid
gives the acetate anion.
(b) A sight difference in O—H bond strength for sulfuric acid is based on the greater electron
withdrawing effect of the sulfonyl unit relative to the carbonyl unit. Both conjugate bases are resonance
stabilized, but there are more resonance contributors for the methanesulfonate anion, so it is more stable.
This increased stability for the conjugate base should shift Ka toward products, making the sulfonic acid
more acidic. In addition, sulfur is a large atom relative to carbon, and this will contribute to increase
charge dispersal, also making the sulfonic acid more acidic.
87. Fluoromethane (CH3F) has a single polarized bond and is the more polar molecule. Methane and
ethylene are hydrocarbons with no polarized bonds, and nonpolar. The four C—F bonds in
tetrafluoromethane cancel, so the molecule is nonpolar.

88. Propanoic acid (third choice from the left) is the only molecule capable of hydrogen bonding, since
it has the polarized O—H unit. The ether and the ketone are capable of dipole-dipole interactions, but
do not have a polarized X—H bond, and butane only has nonpolarized C—H bonds.

89. The methanesulfonate anion is resonance stabilized, as shown, so the charge is dispersed over
several atoms, and there is less electron density available for donation: it is a weaker base. The
methoxide anions derived from methanol is not resonance stabilized, so the charge is localized on the
oxygen atom, and readily available for donation: it is more basic. In other words, the charge density on
oxygen in methoxide is much greater than the charge density on oxygen in the methanesulfonate anion.

Methanesulfonic acid
O O H O O
H H H H
H C S O H C S O H C S O H C S O
H H O H O H
O O

H H H
H C O H C O
H H
Methanol

90.

sp H
H
(a) H2C C NH (b) C C N
sp2 H
sp2 ••

91. Hexane will have the highest boiling point. All are straight-chain alkanes, and hexane has the
greatest mass.
92.
FC = 5-0.5(6)-2 = 0
FC = 6-0.5(6)-2 = +1 H H FC = 5-0.5(8)-0 = +1
H H
•  •
O H
N C
(b) H (c) C N C

• •
(a) H C C Cl H
C H H H FC = 4-0.5(6)-2 = -1
H 3 e- pairs
H H
FC = 7-0.5(2)-6 = 0

FC = 6-0.5(4)-4 = 0 FC = 6-0.5(2)-6 = -1
•  •
H H O

• •
• •
C C H H C O

• •
(d) • •
H C • H FC = 6-0.5(2)-6 = -1 (e) H C C
 •
• •O H H H
N
•  •

FC = 4-0.5(8)-0 = 0 H H
FC = 7-0.5(2)-6 = 0 FC = 5-0.5(8)-0 = +1
Cl Cl
(f) Cl C C Cl
Cl Cl FC = 4-0.5(8)-0 = 0
all Cl's have 3 e- pairs

93.
OH

(4-Methylcyclohexyl)methanol
(4-Methylcyclohexanemethanol)
Chapter 06
Solutions Manual
Organic Chemistry. An Acid-Base Approach 2nd Edition
46. (a) Ka = 6.35x10-6 : pKa = 5.2 (b) Ka = 12.1x107 : pKa = -8.1 (c) Ka = 18.5x10-12 : pKa = 10.7
(d) Ka = 9.2x10-3 : pKa = 2.04 (e) Ka = 10.33x108 : pKa = –9.0 (f) 0.08x10-3 : pKa = 4.1

47. (a) pKa = 6.78 : Ka = 1.66 x 10-7 (b) pKa = -3.2 : Ka = 1584
(c) pKa = 23.5 : Ka = 3.12 x 10-24 (d) pKa = 10.3 : Ka = 5.01 x 10-11
(e) pKa = 35.8 : Ka = 1.583 x 10-36 (f) pKa = -11.1 : Ka = 1.26 x 1011

48.
OH
CH2 O CH2 O
(a) H3C C H H3C C + H-O-H

O O
CH2 O NH2 CH2 O
(b) H3C C H H3C C + H-NH2
O O

H3C O OH H3C O
(c) C H C + H-O-H
H3C H3C
CH3 CH3
NH2
H3C O H3C O
(d)
H3C C H H3C C + H-NH2
CH3 CH3
N≡C OH N≡C
(e) N≡C C H N≡C C + H-O-H
N≡C N≡C
N≡C NH2 N≡C
(f) N≡C C H N≡C C
+ H-NH2
N≡C N≡C
O O
H OH
(g) H3C S O H3C S O + H-O-H
O O
O O
H NH2
(h) + H-NH2
H3C S O H3C S O
O O
49.
Ph Ph Cl
(a) CO2H (b)
CO2H
3,3-Diphenylbutanoic acid
4-Chloro-2-methylpentanoic acid

(c) (d) CO2H


SO3H
5,5-Diethyloctane-1-sulfonic acid (Z)-Hex-4-enoic acid
See Section 9.6.3 for (Z)- nomenclature

50. As noted in A, the COOH group is close to the Cl, which is held in space due to the rigid nature of
the molecule. Therefore, internal hydrogen bonding is possible in A, a through-space effect, but not in
B. Internal hydrogen bonding will lead to enhanced acidity for A relative to B. In other words,
carboxylic acid A has a smaller pKa than carboxylic acid B.
H Me Cl
Me Cl
O H
H
O
O
H O
A B

51. Acid (A) is slightly more acidic because the proton of the COOH unit is closer to the Cl, which
allows better internal hydrogen bonding (a through-space effect) when compared to acid (B).
Cl H
H H

HO Cl O B
Cl Cl
O O
H H
CO2H CO2H
A
O O O
O
H H

Cl H
H H
H Cl
52.

O O
O S CH2CH3 O Cl O
O O
O O O
O
(a) (b) (c) (d) (e) NMe (f)
O S CH2CH3 O O Cl O
H3CH2C O
O no resonance O
no resonance no resonance
O
O O O Cl O

H3CH2C O S CH2CH3 O
O
O O
O Cl O
O

53. (a) Sodium hydroxide (NaOH). The conjugate acid is water, which is slightly more acidity than
propanol, so propanol will likely react as a weak acid.

(b) Hydrochloric acid (HCl). The conjugate acid is chloride ion, and HCl is a much stronger acid than
propanol, so proposal will react as a base.

(c) Water (H2O). The conjugate acid is the hydronium ion, which is a much stronger acid than
propanol, so propanol will likely react as a very weak base or a very weak acid. For all practical
purposes, it is neutral.

(d) Ethanol (CH3CH2OH). The conjugate acid is an oxonium ion, which is a much stronger acid than
propanol, so propanol will likely react as a very weak base or a very weak acid. For all practical
purposes, it is neutral.

(e) Sodium amide (NaNH2). The conjugate acid is ammonia, which is a significantly weaker acid than
propanol, so proposal will likely react as an acid.

(f) Butan-2-one. The conjugate acid is an oxonium ion, which is a much stronger acid than propanol, so
propanol will likely react as a very weak base or a very weak acid. For all practical purposes, it is
neutral.

(g) Methane (CH4). Methane is not acidic or basic, so there is no reaction at all with propanol, so it is
neutral.
(h) Sulfuric acid (H2SO4). The conjugate acid is hydrogen sulfate ion, and sulfuric acid is a much
stronger acid than propanol, so propanol will react as a base.

54.
O O

O -NH
2
(a) + H—NH2
or O O

These are enolate anions. See Section 22.5.1

O -NH O O
2
+ H—NH2
(b)
H H H

O O -NH O O O O
2 O O
(c) + H—NH2

This H is more acidic


due to proximity of two C=O units
-NH
(d) O 2
O + H—NH2
H

-NH O
(e) O 2 O
H + H—NH2
O O O

55. Maleic acid (A) with a pKa of ~1.8 is more acidic because one carboxyl group is relatively close to
the OH unit of a second carboxyl group (they are on the same side of the molecule), so internal
hydrogen bonding is possible. In fumaric acid (B), the pKa is ~3 is higher (less acidic), in large part
because the two COOH units are on opposite sides of the rigid alkene unit. There is no rotation about
the C=C unit.
O
H
O O
O
H
O H O
O H
A B O

56. In 5-bromopentanoic acid, the bromine atom is very far away from the polarized O—H unit, so there
is no through-bond or through-space inductive effect. In 2-bromopentanoic acid, the Br is relatively
close so there is a through-bond inductive effect, and internal hydrogen bonding is possible (leading to a
through-space inductive effect) that leads to enhanced acidity.
Br O O

O O

H Br H

57. Propanoic acid reacts faster with diethylamine, which is a much stronger base than diethyl ether, so
propanoic acid is more acidic in diethylamine than it is in ethyl ether. This means that Ka for the
reaction with diethylamine is larger and Ka for the reaction with diethyl ether is smaller.

58. If there is no base, HCl cannot react as an acid. Therefore, HCl is technically not a base under such
conditions. The real point of this question is to emphasize that an acid only reacts as an acid in the
presence of a base, and that the strength of the acid depends on the strength of the base it reacts with.

59. Butanoic acid is a weaker acid than acetic acid. Compare RCH2COOH with CH3COOH, where R is
an ethyl group. The alkyl group is electron releasing relative to the carbon attached to the carbonyl,
which diminishes the acidity by strengthening the O—H bond by inductive effects. Formic acid is a
stronger acid than acetic acid for the same reason. Compare HCOOH with CH3COOH, where the
methyl group is electron releasing relative to H.

60.

(a) Cl—H NEt3 Cl- H—NEt3

(b) F3B NEt3 F3B—NEt3

(c) CH3CH2COO—H NEt3 CH3CH2COO- H—NEt3

(d) Cl3Al NEt3 Cl3Al—NEt3

(e) CH3SO3— H NEt3 CH3SO3- H—NEt3

(f) Cl3Fe NEt3 ClFeAl—NEt3

61. It is possible to look at the reverse reaction of the conjugate acid derived from reaction of an amine
with an acid (an ammonium salt). For the reverse reaction where the ammonium salt–base pair is on the
left, and the amine and H:base are on the right, Ka can be determined. It is also possible to look at the
reaction of an amine with water to yield the conjugate acid–conjugate base, and use the term KB. The
term KB is used to evaluate the basicity of a base (e.g., an amine), and a larger value of KB (small pKB)
indicates a strong base whereas a smaller value of KB (large pKB) indicates a weaker base.

62. Look at the amine precursors to the amide anions. Diisopropylamine is a weaker acid than
diethylamine, which makes diisopropylamide a stronger base. The reason is probably some steric
hindrance of the isopropyl group relative to ethyl for removal of the proton from nitrogen, as it is
slightly more difficult for the base to approach the proton in diisopropylamine.

N N

63.
(a) Triphenylphosphine (b) Butylphosphine
Ph H3CH2CH2CH2C
P Ph P H
Ph H
(c) Ethylphenylphosphine (d) 1,2-(Diphenylphosphino)ethane
H Ph
CH2CH3 Ph P
P
P Ph
Ph
Ph

64.
H H
(a) (b) (c) N (d)
O O O
H3C CH3 H H H H
H

H H H
O H O H
(e) (f) S (g) (h) O
H3C CH3 H3C CH3
H
H
H H O
(i) N (j) O (k)
H H3C OH
65.
AlCl3
(a) O (b) O BF3 (c) N
BEt3

BF3
O
(d) (e) Cl FeBr3 (f) O ZnI2

66. The reaction of diethylamine with HCl yields Et2NH2+ as the conjugate acid, an ammonium salt.
Ammonium salts are acidic, and the NH unit of the ammonium salt is capable of hydrogen bonding with
the protic solvent ethanol, leading to increased stabilization of the conjugate acid. The hydrogen
bonding in diethyl ether is much less because there is no O—H bond, and the ethyl group provides some
steric hindrance when the oxygen atom approaches another molecule. The increased stability of the
conjugate acid in ethanol shifts Ka toward the conjugate acid, which is consistent with diethylamine as a
stronger base in ethanol than in diethyl ether.

67. The F is more electronegative than O, which is more electronegative than N. Based on this trend,
the electrons on nitrogen in -NH2 should be more available for donation, making it the strongest
nucleophile in this series.

68. The amide anion (-NH2) has a much higher charge density (two unshared electron pairs) relative to
the neutral molecule ammonia (NH3), which has one unshared electron pair. Therefore, ammonia is less
able to donate electrons to a positive carbon, and it is a much weaker nucleophile in its reaction with
acetone than is the amide anion.

69.
(a) CH3CH2–Br –OMe CH3CH2–OMe

CH3CH2–Br –:C≡CMe CH3CH2–C≡C-Me


CH3CH2–Br –CN CH3CH2–CN

CH3CH2–Br –I CH3CH2–I

(b) CH3–I –OMe CH3–OMe


CH3–I –:C≡CMe CH3–C≡C-Me
CH3–I –CN CH3–CN
CH3–I –I CH3–I
.

70. The conjugate base of propyne is CH3C≡C:-Na+, and the alkyne has a pKa of ~ 25. This pKa
indicates that the alkyne anion is a reasonably strong base. Therefore, the alkyne anion may react as a
base with water (pKa, 15.7), whereas no such reaction is possible if the neutral solvent THF is used.

71. Sodium methoxide (NaOMe) can react with the slightly acidic proton of methanol in an acid–base
reaction, but in order to attack the carbon atom as a nucleophile, OH must “leave” (be displaced). In
simple terms, hydroxide is a very poor leaving group in this reaction, and the acid-base reaction is more
facile and much faster.

72. ΔH° = H°products - H°reactants


(a) CH3OH + NH3 CH3O– + +NH4
(b) Cl3CH + H2O Cl2C– +OH3
(c) CH3ONa + ICH3 CH3OCH3 (ignore NaI)
(a) ΔH° = (H°N-H) - (H°H-O )
ΔH° = (75) - (104.2) = -29.2 kcal mol-1.
(b) ΔH° = (H°H-O) - (H°C-H)
ΔH° = (104.2) - (104) = 0.2 = 0.2 kcal mol-1.
(c) ΔH° = (H°C-O) - (H°C-I)
ΔH° = (91) - (56) = 35 kcal mol-1.

73. Assume that ΔS is zero, so ΔG° = ΔH. ΔH° = H°products - H°reactants


ΔH° = H°C-O - H°C-I = 91 - 56 = +35 kcal mol–1
This number indicates an endothermic reaction, which is consistent with a reversible process. It
is important to state that this is not the only criterion for reversibility in a reaction.
H

CH3OH + CH3I H3C O I–

CH3

74. (a) HCOOH (formic acid) (b) CH3OH (methanol) (c) CH3SO3H (methanesulfonic acid)
This question is very misleading. It was given to make a point. If the same base is used, all of
the bond dissociation energies will be the same in that an O—H bond is broken in the reactants.
Therefore, ΔG° and ΔH° calculations cannot be used to determine differences in reactivity. Stability of
the conjugate base is the usual criterion for determining differences in acidity for these three
compounds.

75. By analogy with carboxylic acids, the closer an electron-withdrawing substituent (e.g., chlorine) to
the sulfonic acid proton, the more acidic: due to internal hydrogen bonding (through space inductive
effects). Therefore, 2-chlorobutanesulfonic acid should be the most acidic. There is probably very little
difference in pKa for the other two sulfonic acids. 3-Methylbutanesulfonic acid has an electron-
releasing methyl group, but it is somewhat removed from the sulfonic acid unit. The methyl group
probably exerts a small effect, but this molecule is probably slightly less acidic than hexanesulfonic
acid.
SO3H SO3H
SO3H
Cl
Hexanesulfonic acid 3-Methylbutanesulfonic acid 2-Chlorobutanesulfonic acid

76. In the amide, the electron pair is partly delocalized on the adjacent carbonyl, and the electron-
withdrawing effects of the carbonyl diminish the availability of the electron pair for donation.
Therefore, it is a weaker base than the amine, where the electron pair is reasonably available for
donation.

77. In N-chloromethanamine (ClNHCH3), the electron-withdrawing chlorine group should diminish the
availability of the electron pair on nitrogen, relative to methanamine. Therefore, methanamine is the
more basic.

78. The more hindered amine is (Me3C)3N, with the three bulky tert-butyl groups, relative to the three
relatively unhindered ethyl groups in (CH3CH2)3N (e.g., Et3N). It is more difficult for the proton of
formic acid to approach the nitrogen atom in (Me3C)3N, making it less reactive and a weaker base.
Remember that fluxional inversion about nitrogen occurs in both amines, which exacerbates the steric
hindrance in (Me3C)3N.
O O
H
N H N
O H O H

O O
H
N H N
O H O H

79. Trimethylarsine (Me3As) has the larger arsenic atom relative to nitrogen of trimethylamine, but it is
slightly less electronegative (see section 2.5.1). In general, electronegativity and basicity decreases
going down the periodic table, so the electron density on nitrogen is more available, and it is the
stronger base. This means that trimethylamine will react faster with the Lewis acid, BF3.
80. There is no doubt that the oxygen of the ether is a much stronger base than the fluorine atom of
fluoromethane: O is less electronegative than F, and the oxonium ion product is more stable than the
C—F—Al unit resulting from fluoromethane. Therefore, AlCl3 reacts faster with diethyl ether.

O AlCl3 O AlCl3

F AlCl3 F AlCl3

Me Me
H N HgCl2 H N HgCl2

81. Me Me

82. The reason why certain products are marked as major will be explained in Chapter 10.
The “squiggle” line indicates that the halogen can be on either side of the molecule, as explained
in Chapter 9.

Cl
HCl
(a)

HBr Br
(b)

HCl Cl
(c) Cl
Minor product
Major product
HI
(d)

I
Br
HBr
(e) Minor product
Br Major product

HI
(f)
Minor product
I Major product I
83.
HBr
(a) OH Br

H2SO4 H
(b) O O HSO4–

HCl
(c)
OH Cl

Br OCH3
CH3O–Na+
(d)

CH3CH2CO2H H
(e) –O
N N 2CCH2CH3

OH OCH2CH3
1. NaNH2
(f)
2. CH3CH2I

O– Na+

BF3 Et3N:BF3
(g) Et3N

1. NaNH2
(h)
2. HCl

:– Na +

Spectroscopic Problems (to be done only after chapter 14 is read and understood)
In all of the following, IR data is from Table 14.3 (Section 14.3.3). The 1H NMR data is from
Table 14.5 (Section 14.4.7).
84.

CO2H 2500-3000 cm–1 OH IR 3300 cm


1

1730 cm–1 IR

12 10 8 6 4 2 04 3 2 1 0
PPM PPM

85. Triethylamine has no hydrogen atoms attached to nitrogen, so there will be no signal in the 3300-
3500 cm–1 region. The ammonium salt does have a N-H group, and will therefore show a bond in that
region.

86. The OH unit in propan-2-ol is capable of hydrogen-bonding, which effectively changes the O—H
bond distance, which in turn influences the amount of shielding. As the amount of internal hydrogen
bonding changes, the chemical shift of the proton changes.

87. The answer is similar to the phenomenon in question 86. The OH unit in a carboxylic acid is
capable of hydrogen bonding, which effectively changes the O—H bond distance, which in turn
influences the amount of shielding. The extent of internal hydrogen-bonding is much greater for the
acidic proton of a carboxylic acid, which greatly diminishes the amount of shielding. In other words,
that proton is very deshielded, and far downfield.

88. In the IR, the tertiary amine trimethylamine will have no signals in the 3300-3500 cm–1 region, the
secondary amine N-methylaminoethane will have a singlet in that region, and the primary amine,
propan-1-amine will have a doublet (2 peaks) in that region.

N N
H

3 2 1 04 3 2 1 0
PPM PPM
NH2

6 4 2 0
PPM

89.
O O

HO H H H

10 8 6 4 2 010 8 6 4 2 0
PPM PPM
Chapter 07
Solutions Manual
Organic Chemistry. An Acid-Base Approach 2nd Edition
17.
HO
NaCN CN
O
(a) HO
NaNH2 NH2 poor yield
O
NaI essentially
no reaction
Ph CN
NaCN

Ph I
Ph NH2
(b) NaNH2

Ph I
NaI

OH
NaCN CN
O H OH
(c) NaNH2 NH2 poor yield
H O H
NaI essentially
no reaction
H

18.
HI
I
(a) Major Minor
I
Also see Chapter 10
HBr
(b) Br

HCl
(c)
Cl
19. ΔH° = H°products - H°reactants
All of these hypothetic reactions are endothermic as written. The point of this exercise is to begin the
process of learning those reactions are reasonable and those that are unlikely.
H
(a) CH3I + CH3OH H3C O + I–
H CH3
(b) CH3CH3 + CH3NH2 H
N + CH3–
H3C CH3
(c) CH3-CH3 + (CH3)3C-Cl CH3Cl + (CH3)3C-CH3

(d) (CH3)3CCH3 + I – (CH3)3C– + CH3-I


ΔH° = H°products - H°reactants
(a) ΔH° = (H°C-O) - (H°C-I) ΔH° = (91) - (56) = +35 kcal (146.5 kJ) mol-1.
(b) ΔH° = (H°N-C) - (H°C-C) ΔH° = (87) - (88) = -1 kcal (-4.18 kJ) mol-1.
(c) ΔH° = (H°C-Cl + H°C-C) - (H°C-Cl + H°C-C)
ΔH° = (84 + 88) - (84 + 88) = 172 - 172 = 0 kcal (0 kJ) mol-1.
(d) ΔH° = (H°C-I) - (H°C-C) ΔH° = (56) - (88) = -32 kcal (-134.0 kJ) mol-1.

20. ΔG° = ΔH° + TΔS°, and ΔH° = 56 kcal (234.5 kJ) mol-1, T = 100 °C, and ΔS° = 3.2 cal-1.
Remember that temperature must be converted to kelvin. The calculation suggests that the temperature
must be lowered for -20,625 K, which is obviously impossible. The temperature cannot be lowered
below absolute zero, which is 0 K.
ΔG° = ΔH° + TΔS°, so
ΔG° = 56 kcal (234.5 kJ) mol-1 + 373.15 x 0.0032 kcal mol-1 = 56 - 1.19 = 54.8 kcal (229.4 kJ) mol-1
The ΔS° term is (0.0032/56) x 100 = 0.0057%.
To calculate the temperature for ΔG° = -10: ΔG° = -10 = 56 + x (0.0032)
(-10 -56 )/0.0032 = x, so x = -66/0.0032 = -20,625

21. The transition state is that portion of the energy curve that represents the point at which bonds begin
to break in the reaction and begin to form in the product. The difference in energy between the energy
of the reactants and the energy of the transition state is the activation energy.

22. A transition state is not a detectable or isolable entity, but rather a point on an energy surface. It is
not possible to “see” it.

23. No! Energy equal to the activation energy (Eact) must be added to initiate the reaction, and this has
nothing to do with whether or not the overall reaction is endothermic or exothermic.
24. Plot ln [A] vs time in seconds. The slope of the line is the first-order rate constant, k. The half-life
is 0.693/k. Based on the plot, using the best straight line possible, the slope is 0.006,
so the rate constant k = 0.006. Half-life = ln 2/k = 0.693/0.006 = 115.5 s.
450

![A]! ln[A] Time (s) 400

!2.0! 0.693 0 350

!1.5 0.405 !25 300

!0.95! -0.05 50 250

!0.7! -0.36 100

ln [A]
Series1

!0.52! -0.65 150 200

!0.40! -0.92 200 150

!0.29! -1.24 250 100

!0.23! -1.47 300 50

!0.19! -1.66 350


0
!0.12 -2.12 !400 -2.5 -2 -1.5 -1
time
-0.5 0 0.5 1

25. Based on the plot of the data, the second order rate constant is 0.0051.
1.4

1.2

0.8
tme(sec)

ABo AoB ABo/AoB ln(ABo/AoB) time(s) Series1

.00048 .00048 1 0 0 0.6

.00043 .00039 1.10 .095 17.8


.00042 .00036 1.12 .113 27.3 0.4

.00038 .00029 1.31 .270 53.1


.00035 .00023 1.52 .419 86.6 0.2

.00031 .00015 2.07 .728 151.0


.00029 .00011 2.64 .971 191.8 0
0 50 100 150 200 250 300
.00028 .00008 3.5 1.253 240.1 ln(ABo/AoB)

26. Calculate the half-life, given the following data. First-order half-life = 0.693/k. Second-order half
life = 1/k[Ao], where for second-order reactions, assume [A]o = 0.5.
(a) A first-order reaction where k = 1.2x10-6: half-life = 5.78x105
(b) A second-order reaction where k = 4.5: half-life = 0.44
(c) A first-order reaction where k = 5.8x103: half-life = 11.95x10–5
(d) A second-order reaction where k = 9.25x10-4: half-life = 2162
(e) A first-order reaction where k = 0.6x10-9: half-life = 1.16x109
(f) A second-order reaction where k = 3.44x1012: half-life = 5.83x10-13

27. ΔG° = -RT (ln K) = -2.303 RT (log K) and the reaction temperature is 25 °C (298.15 K).
R = 1.986 cal deg-1 mol-1, T = temperature in kelvin, and e = 2.718 (base of natural logarithms).
RT for all reactions = 1.986x298.15 = 592.13 x -2.303 = -1363.7
(a) 2.5 log 2.5 = 0.398 x -1363.7 = -7923
(b) 1.55x10-6 log 1.55x10–6 = -5.81 x -1363.7 = 15846
(c) 8.77x10-9 log 8.77x10-9 = -8.06 x -1363.7 = 10991
(d) 4.4x105 log 4.4x105 = 5.64 x -1363.7 = -7691
(e) 1.23x1018 log 1.23x1018 = 118.09 x -1363.7 = -161039
(f) 10.45x10-3 log 10.45x10-3 = -1.98 x -1363.7 = 2700

28. ΔG° = -RT (ln K) = -2.303 RT (log K) and the reaction temperature is 25 °C (298.15 K).
R = 1.986 cal deg-1 mol-1, T = temperature in kelvin, and e = 2.718 (base of natural logarithms).
RT for all reactions = 1.986x298.15 = 592.13 x -2.303 = -1363.7.
Assume that ΔS° is zero, so ΔG° = ΔH° in all cases, so (ΔG° /-2.303RT) = (ΔG° /-1363.7)
K = 10-(ΔG° /-1363.7)
(a) -1.5 kcal (-6.3 kJ) mol-1 (ΔG° /-1363.7) = 1.1x10-3 K = 0.997
(b) 100.3 kcal (419.9 kJ) mol-1 (ΔG° /-1363.7) = -0.074 K = 1.18
-1
4 4
(c) -4.5x10 cal (-18.8x10 J) mol (ΔG° /-1363.7) = 138.16 K = 6.92x10-139
-1
(d) 18.5 kcal (77.5 kJ) mol (ΔG° /-1363.7) = -0.014 K = 1.03
(e) -33 kcal (-138.2 kJ) mol-1 (ΔG° /-1363.7) = 0.024 K = 0.946
6 7 -1
(f) -12.5x10 kcal (-12.5x10 kJ) mol (ΔG° /-1363.7) = 9166.2 K = 0

29. At least five half-lives are required for 98% completion. If the half-life is 8 h, then 8x5 = 40 h will
be required.

30. If K = 10-(ΔG° /-1363.7), then for reaction A, K = 1.40, and for reaction B, K = 1.03. For reaction A, the
equilibrium constant suggests there is more product than reactant, so we anticipate the ability to isolate
the product. In the case of reaction B, the equilibrium constant is close to 1, so there will be close to a
1:1 mixture of reactants and products. The answer to this question depends on how easy it will be to
separate reactions and products. If it is relatively easy to separate these compounds, then product can be
isolated from both A and B, but the yield of product will be higher from reaction A.

31. H—I

I– I
32. If there are carbocation intermediates, then the final product must arise by the reaction of bromide
ion with carbocation B. However, the reaction of HBr and the alkene reactant must give carbocation A.
The only way to obtain the final product is for a skeletal rearrangement to occur, from A to B. This
rearrangement occurs by transfer of a hydrogen atom from A to B, generating the more stable tertiary
carbocation B from the less stable carbocation A. This rearrangement is referred to as a 1,2-hydride
shift.
Br
H–Br

H—Br
Br–

1,2-H shift
A (a rearrangement) B

33 There is no indication that a transient product appears and is then consumed. Based only on this
curve, one must conclude there is no intermediate.

34. For the two competing reactions, both reactions are second order, and the starting concentration is
assumed to be 1. Therefore, the half-life = 1/k. For reaction A, 1/360 = 0.003 s, and for reaction B,
1/3.5 = 0.29 s. Based on this calculation, formation of D is faster than formation of C, so D is likely to
be the major product of this reaction.

35. Which of the following is likely to be the best two-electron donor in a reaction with CH3Cl?
Explain. Based on electronegativity, and larger size of the ion, which means more charge dispersal,
chloride ion should be the poorest electron donor (weakest nucleophile). The methyl group in CH3O– is
electron releasing relative to the H in HO-. Therefore, the electron density on O in methoxide anion is
greater, and it should be the best electron donor (strongest nucleophile).
Chapter 08
Solutions Manual
Organic Chemistry. An Acid-Base Approach 2nd Edition

40.
(a) cis-2,3-Dimethyloxirane (b) N,3-Diethylpyrrolidine (c) 4-Hydroxypiperidine

O HO

N NH

(d) 3-Chlorooxetane (e) N-phenyl-2-methylaziridine (f) 3,4-Dimethyltetrahydrofuran

Cl

O N
Ph O

41. A gauche conformation is a staggered conformation in which the two substituents (Cl) do not
eclipse any other atom, and the substituents are not as far away as possible (the anti conformation).
Cl Cl H Cl Cl H

H H Cl H H HH Cl
H H

H H H Cl Cl H H H

eclipsed-syn staggered-anti eclipsed staggered-gauche

42. Assuming that each chair conformation is locked, then the molecule with the fewest axial
substituents will have the least transannular interactions. The molecule circled has no axial substituents,
only equatorial, whereas all of the others have several axial substituents. The last structure on the left is
a boat conformation, which does not have axial or equatorial substituents, but the “flagpole” chlorine
atoms are close in space, and this constitutes significant transannular strain.
Cl Cl Cl Cl
Cl Cl
Cl Cl
Cl Cl
Cl
Cl Cl Cl
Cl Cl
Cl
Cl Cl Cl
Cl Cl
43. The circled structure has the most axial bromine atoms.
H H Br ax H H H ax Br Br ax
H H H H
H H H H
H H H H
Br Br Br H Br Br H H
H H H H
H Br H H H
ax BrH H BrH H Br ax
H H H H

44. Cyclopropane is a flat molecule with significant Baeyer strain. It is reasonable to assume that the
molecule with the most cyclopropane rings will have the greatest Baeyer strain. The circled molecule
fits that description, but what may be less obvious, is that attaching four cyclopropane units to the four-
membered ring will flatten the four-membered ring, introducing even more Baeyer strain.

45. The anti conformation is marked. It is a staggered conformation with the two Cl atoms as far apart
as possible (180°).
H H Cl H H H Cl Cl
Cl H
Cl Cl
H H
H H H H Cl H Cl H H H
staggered-anti staggered-gauche staggered-gauche eclipsed-syn

Cl
Cl H H H
H Cl Cl H
46. •
H H
H H
H H Cl Cl
H
47. In the gauche conformation, internal hydrogen bonding is possible in an aprotic solvent, but
methanol is a protic solvent. Therefore, intermolecular hydrogen-bonding with the solvent should
stabilize the anti conformation more than the gauche.
H
H H H-OCH3
H N
H H
Staggered-gauche H NH2 H2N
H N
H3CO-H Staggered-anti
H H
H H

top Br
Br Br
Br Br 1 Br
4
H 2
H
48. H
Br
2 H
1
H Br 4
BrBr Br
H
Br

Br Br Br
Br Br
H
49. H Br H H
H H H H Br H HH HH H
H BrH
Staggered-gauche Staggered-anti Eclipsed-syn Eclipsed

50. The flat structures are the highest in energy, and the boat-conformations are the lowest. The boat-
conformations with the two chlorine atoms in flagpole positions has the highest transannular strain, and
the boat-conformation that has hydrogen atoms in the flagpole positions (circled) has the least amount of
transannular strain, and is the lowest energy conformation.
Cl Cl H H
H H Cl Cl
H H Cl Cl
H Cl H H

51. The large chlorine atoms are in close proximity, and the steric repulsion will elongate or at least
distort the three-membered ring to accommodate the six chlorine atoms. For this reason, the C—C
bonds in hexachlorocyclopropane are expected to be weaker when compared to cyclopropane. Indeed,
molecular modeling indicates the C—C bond distance in cyclopropane is 1.502 Å (150.2 pm), whereas
the C—C bond distance in hexachlorocyclopropane is 1.524 Å (152.2 pm).
52.
Br Br
Br Br

Br Br
Br Br Br Br

2 Axial 1 axial
Br Br

Br
Br

Br Br
Br
Br
1 axial Br
Br Br
Br Br 2 Axial
Br

53. There are two boat conformations in equilibrium. The molecule that is circled has two bromine
atoms on each carbon, so both boat conformations will have the Br—Br transannular strain. In the third
molecule, there is a Br—Br interaction in one boat, but a H—H interaction in the other, which is less
than the transannular stain in the molecule circled. In the other two molecules, the H—Br transannular
interaction is much less.
Br Br Br H
Br Br Br H Br Br H Br
H Br H H
54. Conformation A has three axial groups (Cl, Br, Me) and B has two axial groups (Br, Cl). Since the
relative steric demands of the substituents are unknown, simply count the number of axial groups, and B
has only two. Therefore, the equilibrium should favor more B and less A.
CH3 Cl Br
H H
Br Cl
Cl 1
Br 4 5 Br6 H H 5
Cl 1
2 6 B
2 BrCH3
6 A H 2 H H 4 3
Cl 3
5 1
3 H H
4 H
Br Cl H CH3 H H H

55.
CH3
H H H3C CH3
(a) butane anti
H H H syn
CH3 H H
H
F
H H FF
(b), 1,2-difluoroethane anti
H H H syn
F H H
H
CH3
H3C CH3 H3C CH3
(c) 2,2,3,3-tetramethylbutane anti
H3C CH3 CH3 syn
CH3 H3C CH3
H3C
OMe
H H MeO OMe
(d) 1,2-dimethoxyethane anti
H H H syn
OMe H H
H
H
H H C3H7H
(e) pentane anti
H H H syn
C3H7 H H
H

H
H H ClH2C H
(f) 1-chloropropane anti
H H H syn
CH2Cl H H
H
56.

H H CH3

(a) butane anti syn


CH3 H H
CH3 CH3

H H H H
H H F

anti H H
F F F
(b), 1,2-difluoroethane syn
H H H H
H3C CH3 CH3

CH3 anti H3C CH3


CH3 CH3 syn
(c) 2,2,3,3-tetramethylbutane
H3C CH3 H3C CH3

H H OM e

anti H H
OM e OM e syn
OM e
(d) 1,2-dimethoxyethane
H H H H
C3H7
H H

(e) pentane anti H H syn


C3H7 H H

H H H H
H H CH2Cl

(f) 1-chloropropane anti H H syn


CH2Cl H
H

H H H H
57.
H H

CH3 Br
H H
H H H H
CH3
Br H
0° - 360° CH3
H Br
H 300° 60°
H

270° 90°

H H
240°
CH3 H
120° H H
180°
CH3 H
Br H
H H
H Br
CH3
H H

Br

58.
H H
H H
CMe3 CMe3 anti
CMe3 syn
H H
H H
CMe3
59. The hand-held models do not show the actual size of the atoms, and certainly not the amount of
space that a methyl group actually occupies. The space-filling molecular model shown is much better,
and clearly indicates that the two terminal methyl groups effectively compete for the same space, which
is the source of the steric hindrance.

60. The covalent radius of Cl is 99 pm and a methyl group has a covalent radius is 200 pm. The
methyl group is larger. However, each chlorine atom has three unshared electron pairs that contribute to
the Cl—Cl repulsion, which effectively raises the energy of the steric repulsion.

61.

Me Br low E Et H high E
high E
(a) (b) Et H
Me Br low E

Br Et H Et H
Me Br
OMe OMe
Et Me Me H
low E high E low E
Et Me (d) Me H high E
(c) OMe OMe OMe CHMe2 CHMe2
OMe

Et Me H Me Me H
Et Me
H Cl
low E I
Me F
high E
low E
(e) (f) Me F high E
Cl H I I I
Cl H
F Me Me F
Cl H
62. When the bonds are all syn, one gets a twisted structure as shown that is much higher in energy than
the extended structure obtained when all the bonds are anti.

63. Octane has an extended type structure, assuming all bonds possible have an anti conformation. cis-
Oct-4-ene shows a twist in the middle whereas trans-hex-3-ene has an extended structure. The all trans
tetraene also has an extended structure.

octane cis-oct-4-ene

trans-hex-3-ene all-trans 1,3,5,7octatetraene

64. In the gas phase, the molecule should exist primarily in the gauche conformation shown due to
stabilization via internal hydrogen-bonding. In the hydrogen bonding solvent methanol, intermolecular
hydrogen-bonding effectively makes the OH groups larger and the minimal conformation will have the
OH groups anti in order to minimize steric hindrance.
H OH
OH
H
H H

65. The syn and anti rotamers are shown. In the presence of water, intermolecular hydrogen-bonding
effectively increases the ‘size’ of the OH group, so there is a greater energy preference for the anti-
rotamer.
H H H H H O
H H
H H
H O-H O
CH3 CH3 CH3
H
H H H H
O-H
syn anti In water
66. The trans C—C=C—C unit has the constraint that bond angles about each sp2 carbon atom is 120°,
which imposes severe constraints on the remainder of the molecule. In other words, the trans double
bond requires the other two carbons to have their bond angles distorted, and the bond lengths must also
be distorted. It is simply too high in energy to exist. Cyclohexyne is worse. The C—C≡C—C unit is
linear, which requires even greater distortion of bond angles and bond distances.

67. The geometry of the C=C unit is planar, but in A it is not possible to flatten the molecule due to the
bicyclic nature of the molecule. To flatten the molecule would essentially squash the atoms together,
which is simply too high in energy. The C=C unit in B can flatten out without distortion of the rest of
the molecule.

A B

68. Attempting to confine three adjacent linear C≡C units to a ring would require sever distortion to the
point that it is simply impossible.

69. The lowest energy rotamer will have the planar C=C units anti. The planar nature of these units
makes them effectively larger than the methyl groups.

H CH3
H3C H

70. Octa-1,3,5,7-tetrayne will be linear because each C≡C unit is linear:

71. In 3,4-dichlorooctane, the two Cl atoms will be anti, as will the two propyl groups. Each propyl
group is only slightly larger than a methyl because there is rotation about each bond, and each bond will
effectively exist as an anti rotamer, as shown in the extended conformation to the left.
Cl Cl
H CH2CH2CH3
H3CH2CH2C H
Cl
Cl
72. Two possibilities are shown. There does not appear to be a large difference, but A may have a bit
more steric hindrance because Br is larger than Cl, and there is no great difference in steric hindrance
between methyl and ethyl. The choice is not obvious, however.
Br Br Br
H CHClEt H Et
or
H3C H H3CBrHC H
Cl Br Br Cl
A B

73. All geminal compounds are in red. All vicinal compounds are in blue.
Cl
Cl Cl Cl
Cl
Cl
Cl Cl Cl Cl Cl
Cl
Cl Cl Cl
Cl Cl Cl Cl

Cl Cl
Cl Cl Cl
Cl
Cl Cl
Cl Cl
Cl Cl
Cl Cl Cl
Cl Cl
Cl Cl
Cl Cl Cl
Cl Cl Cl
Cl
Cl Cl Cl Cl
Cl
Cl
Cl Cl Cl Cl Cl Cl Cl Cl
Cl Cl
Cl Cl
Cl Cl
Cl
Cl Cl Cl Cl Cl Cl
Cl

Cl
Cl Cl Cl Cl
Cl Cl
Cl Cl Cl
Cl
Cl
Cl Cl Cl
Cl Cl Cl Cl
Cl Cl
Cl Cl Cl Cl Cl
Cl
Cl
Cl Cl
Cl Cl Cl Cl Cl Cl
Cl
74. Examination of 2,2,5,5-tetramethylhexane-3,4-diol shows that if the two very large tert-butyl groups
align in an anti conformation to minimize steric hindrance, then the two hydroxyl groups are also anti.
The conformation is therefore driven by alignment of the sterically bulky groups.

H
OH

OH OH

OH
H

75. The angular nature of the ether linkage allows an extended conformation in which all C—C bonds
are anti.
O

76. Intramolecular hydrogen bonding of the hydroxyl group and the acidic carboxyl proton is possible,
as shown. In hexane solvent, a nonpolar solvent that cannot form a hydrogen bond with the acid,
intramolecular hydrogen bonding is maximized, so it is assumed that 3-hydroxybutanoic acid assumes a
conformation similar to that shown. In other words, the intramolecular hydrogen-bonding overrides the
normal syn-anti conformations.
H H
O O

77. The conformation is not obvious. The planar nature of the C=O units may lead to the two CHO
groups in an anti relationship. However, aldehydes are capable of internal dipole–dipole interactions,
and a gauche rotamer is shown with the assumption that this conformation may have more stability.
Note that this is an educated guess, and the rather simple analysis done here cannot give the answer.
This problem is presented to emphasize that conformation is often the result of complex interactions,
and sometimes the correct answer is, I don’t know for sure.
H
H
O
O
H H
O
H
H
O
78. In terms of Bayer strain, (f) has the most, followed by (b). Planar structure (a) has some Baeyer
strain, as does (e). The order is likely (f) > (b) > (a) > (e) > (c) > (d).
In terms of torsion strain, planar cyclohexane has more than cyclopropane or cyclobutane, and
chair cyclohexane has none. The order is (a) > (b) > (f) > (e) > (c) > (d).
H H H H H
H H H HH H
H H H
(a) H H (b) H
H
H (c) H
H H H H
H H H H HH
H H H H
H H
H H H H H H
(d) H H (e) H (f)
H H H H
H H H
H H H H
H H H H

79. The hydrogen atoms indicated in all three molecules are sufficiently close that there are transannular
interactions. The representative transannular interactions are marked in each structure.
CH3 CH3 H H CH 3

CH3 CH3
H H H H CH3
CH3 CH3
H H H H

80.

(a) (b) (c) (d)


81. In trans-1,4-dimethylcyclohexane, the transannular interaction involves methyl-hydrogen in both
conformations. The energy of each conformation is about the same, so there should be a roughly equal
amount of each conformer. In cis-1,4-dimethylcyclohexane, however, one conformation has a methyl-
methyl transannular interaction and the other has a hydrogen-hydrogen interaction that is much lower in
energy than the conformation with methyl interactions. Therefore, the cis-isomer will have a large
preference for the lower energy conformation.
CH3 H CH3 H3C
H CH3 H3 C H H H
H3C CH3
H H3C
trans H H
cis-

82. Either chair conformation of 1,2-di-tert-butylcyclohexane requires that one of the very bulky tert-
butyl groups assume an axial position, which imposes a high energy barrier due to A-strain. In the boat
conformation, both tert-butyl groups are “pseudo-equatorial”, which removes the A-strain and is much
lower in energy.

versus

83. It appears that the diisopropyl derivative, with the larger isopropyl groups, will have a greater
amount of A-strain. The methyl groups will spend more time “outside” the ring cavity, however, as
shown. This phenomenon is due to rotation about the C—C bonds (ring carbon—CHMe2) to minimize
steric interactions. This finding means that the difference in A-strain for isopropyl vs methyl is not
necessarily as great as might be expected.

cis-1,3-Dimethylcyclohexane cis-1,3-Dibromocyclohexane cis-1,3-Diisopropylcyclohexane


84. The two chair conformations of cis-1,2-diisopropylcyclohexane show that one group is axial and one
is equatorial in each conformation. Therefore, the A-strain is the same in both conformations, and there
should be a roughly equal amount of each. In trans-1,2-diisopropylcyclohexane, one chair conformation
has both groups axial, which imposes a great deal of A-strain, whereas the other chair confirmation has
both groups equatorial. The latter is much lower in energy because it has no A-strain. Therefore, trans-
1,2-diisopropylcyclohexane will exist primary as the diequatorial conformation with very little of the
diaxial conformation.

H
H
H
H
cis- H trans- H
H
H

85. From the diagrams, it is clear that the boat conformation of 1,1,4,4,5,5-hexamethylcycloheptane has
some transannular strain because the methyl groups are closer together than in the chair conformation.
CH3
CH3 CH3 CH3
H3C CH3
H3C CH3 H3C CH3
H3C

chair CH3 boat

86. Cyclooctane has a relatively high amount of transannular strain, and cyclooctanol also has this
transannular strain. Oxidation to the ketone (Section 17.2) flattens out the three carbons atoms
associated with the carbonyl unit, which diminishes some of the transannular strain, which is more
apparent in the space-filling models. Diminished transannular strain makes the oxidation more facile,
whereas reduction of the ketone to yield the alcohol will introduce more transannular strain. This
increased strain imposes a slight energy barrier to the reduction, which makes it a bit more difficult.
H H
H H
H oxidation H
H H H H
H H
H OH H
H H O
H H reduction H H
HH HH
H
H H
H H
87. The two chair conformations of cis-3,6-dimethylcyclohexene show that there is one pseudo-axial
methyl and one pseudo-equatorial methyl in both conformations. Therefore, they are expected to have
roughly the same energy and one will not be present in greater amount than the other.
H H
CH3 CH3
H3C H H CH
3
H H
H H H H
H H

88. Conversion of the flat C=C unit to the methyl group introduces A-strain in methylcyclohexane,
which means that it is higher in energy than methylenecyclohexane. The A-strain is the source of the
energy barrier mentioned in the question.
CH2 CH3
H H
H H
H

89. One boat conformation of cis-1,4-cyclohexanedicarboxylic acid (shown) has internal hydrogen-
bonding between the two carboxyl groups, which stabilizes that conformation. Therefore, there is a
relatively high percentage of this particular boat conformation.

O HO
HO O

H H

90. Methoxycyclohexane (A) exists primarily in the chair conformation due to diminished A-strain
when compared to the conformation with the OMe group axial. In 2-methoxypyran (B) you can
imagine an interaction of the lone electron pairs on the two oxygen atoms when the OMe group is
equatorial (B’), which is minimized when the OMe group is axial. This simplistic explanation indicates
that the electron pair interaction is higher in energy than the A-strain imposed by the axial methoxy
group. as noted in the question, this is an overly simplistic explanation for a more complicated issue,
but based on what is known from this chapter, it is a reasonable explanation.
O

OMe
MeO O
OMe
A B B'
Chapter 09
Solutions Manual
Organic Chemistry. An Acid-Base Approach 2nd Edition

36. The molecule shown has a carbon atom with four different groups attached. In the molecule marked
CHBrF, carbon has only three substituents, and it is not a real molecule - it is a trick. The circled
molecule has four different atoms or groups (H, Cl, Br, OH), so the carbon is a stereogenic center, and
the molecule is chiral.
FCH2CH2Br CCl4 CHBrF BrCHClOH

37.
c b d a
(a) CH2CH2Br CH2CHBrCH3 CH2CH2CH2CH2OH CH2F

(b) CH2CH2OH CH2CHBrCH3 CH2CH2CH2CH2I CH2CH3


b a c d

38. A(+70°) + A’(-70°) = -35°. Assume that A + A’ = 1, so A = 1-A’


(1-A’)(+70°) + A’(-70°), so 70° -70°A’ - 70°A’ = -35°
-140°A’ = –35°-140° = -105°, so A’ = -105°/-140° = 0.75.
If A’ = 0.75, then A = 0.25, or 25% of A and 75% of A’.

39. All contribute to specific rotation except group priority.


path length group priority concentration α

40. If [α] for the (R)-enantiomer of a molecule is –50°, then [α] for the (S)-enantiomer has the same
magnitude but with the opposite sign, which is +50°.
+50° -50° +5° -5° 0°

41. If [α] for R is -20°, then 0.6(-20°) + 0.4(+20°) = [α] for a mixture of enantiomers = -12°+8° = -4°.
+10° –10° +20° –20° 0° +4° –4°

42. The [α] for a racemic mixture is always 0° because a racemic mixture is a 50:50 mixture of both
enantiomers. Therefore, 0.5(+° R) + 0.5(-° S) = 0°.
+100° -100° +50° -50° 0°
43. If the specific rotation of a pure enantiomer is +100°, then specific rotation for the other enantiomer
is -100°. If the specific rotation of a mixture of both enantiomers is -20°, then
-20° = x(+100°) + y(-100°), and x+y = 1, so x = 1-y. Therefore, -20° = 1-y(+100°)-100°y
and -20° = 100°-100°y-100°y, or -20° = 100°-200°y, and -20°-100° = -200°y,
and -120° = -200°y, so y = -120°/-200°, and y = 0.6, so x = 0.4. The ratio is therefore 60:40.
50:50 20:80 30:70 40:60 10:90

44. Which of the following is the enantiomer of (2R)-bromohexane?


Br Br (S) (R)

(R) (S) Br Br
2-bromohexane 2-bromohexane 3-bromohexane 3-bromohexane

45. The circled solvents each have a stereogenic center, and therefore they cannot be used for
determining [α] of a compound in a polarimeter.
H
H2O CH3 Cl
O
HO H O Cl
H H
has a chiral center
has a chiral center

46.
Me
Et
(R) (R)
(S)
(b) Br (R)
(a)
(c)
OH (R) (S) Cl
(S) (R) HO CH2Br
(d)
(e) (f) (S)
(S)
(S)
Et (S) C3H7
O
CH3 NH2
OH
47.
Br Br
(S) OH
(S) Br
(a) (b) (R) (R) (c)
(S)
Cl
Br
Chiral. It has an enantiomer A meso compound. It has a There is no stereogenic center
superimposable mirror image, so so it has a superimposable
it does not have an enantiomer mirror image, and no enantiomer
Me Me
(f)
(d)
(e)
Br OH OH OH OH
OH OH
Chiral. It has an enantiomer This diol is symmetrical, and The mirror images are not superimposable.
it has a superimposable mirror These compounds are enantiomers, and
image, so there is no enantiomer represent a chiral molecule

48.
OH OH
H
*
(a) ** 2 22 23 24 25 26
H3C * *
Cl
H3C H
OH OH
Cl
* * 23 24 25 26 27 28
(b) Cl H
HO
* *
H3C H *

49.
CH3
CH3 (R)
(S) CH2CH2CH3 Cl H
H OH (R) (S)
(a) (R) (b) H3C H (c) H Cl
H Br (R)
C8H17 H Cl
CH2CH3
CH2CH2CH3
(3R)-Bromopentan-(2S)-ol (R)-4-Methyldodecane (2R,3S,4 R)-2,3,4-Trichloroheptane
CH2CH3 CH2CH3
(S) (R)
(d) H OH (e) H3C CH2CH3 (f) H OH
CH2CH2CH2CH3 CH2CH2CH2CH2CH3 CH2CH2CH2CH3
Heptan-(3S)-ol 3-Ethyl-3-methyloctane Hept-1-en-(3R)-ol
This question is a trick!
C3 has two ethyl groups, so there is no stereogenic center
50.
CH2Br
HO CH2Br CH2CH3 H (R)
Me3C (R)
(R) H
(a) (R) (b) (c) Cl Br (d) (S)
H3C CH2CH3 H3C CHMe2 HO CH3
Et
CH2CHMe2
Br CH=CH2 H CH2CH2CH2Br
(R) (R) (S)
(e) (f) Me2HC (g) OH (h) H CH2OH
(S)
H
CH3 CH3 CHMe2

51. In all cases, convert centimeters (cm) to decimeters (dm), 1 cm = 0.1 dm.
(a) +18°/(1.1x2.5) = +6.55° (b) -176°/0.3 x 5.0) = -117.3°
(c) -1.4°/(5.4 x 2.5) = -0.10° (d) +94°/(2.3 x 3.0) = +13.6°

52. Calculate the percentage of each enantiomer and the %ee for the mixture given the following
information.
(a) [α]20 20
D = +18.6° for the S-enantiomer and [α] D = -2.5° for the mixture.
S(+18.6°) + R(-18.6°) = –2.5°C. Assume that S + R = 1, so S = 1-R
(1-R)(+18.6°) + R(-18.6°), so 18.6° -18.6°R -18.6°R = -2.5°
18.6° -37.2°R = -2.5°, so R = -2.5°-18.6°/-37.2° = -21.1/–37.2 = 0.567.
Therefore, 56.7% R and 43.4% S. Using Figure 9.15, 15% ee (R).
20 20
(b) [α] D = -166° for the R-enantiomer and [α] D = -154° for the mixture.
S(+166°) + R(-166°) = -154°C. Assume that S + R = 1, so S = 1-R
1-R(166°)-166°R = -154°, so 166°-166°R-166°R = -154°
-332°R = -154°-166°, so -332°R = 320°, so R = 320°/-332° = 0.964.
Therefore, 96.4% R and 3.6% S. Using Figure 9.15, 98% ee (R).
20 20
(c) [α] D = -45° for the S-enantiomer and [α] D = +27° for the mixture.
S(-45°) + R(+45°) = +27°C. Assume that S + R = 1, so S = 1-R
(1-R)(-45°) + 45° = +27°, so -45°+45°R +45°R = +27°
90°R = 27°+45°, so 90°R = 72°, so R = 72°/90° = 0.8.
Therefore, 80% R and 20% S. Using Figure 9.15, 60% ee (R).
20 20
(d) [α] D = +208° for the R-enantiomer and [α] D = -118° for the mixture.
S(-208°) + R(+208°) = -118°C. Assume that S + R = 1, so S = 1-R
(1-R)(-208°) + R(208°) = -118°, so -208°+208°R+208°R = -118°
-208°+416°R = -118°, so 416°R = -118°+208°, so 416°R = 90°, so R = 90°/416° = 0.22.
Therefore, 22% R and 78% S. Using Figure 9.15, 56% ee (S).

53. If the product is a racemate, then there is a 50:50 mixture of the (R) and (S) enantiomers, which
means that the specific rotation is 0°.
. If the [α]20
D = -77° for the (S)-iodide and it reacts to give a new iodide with complete inversion, the
product will be the (R) iodide, and the specific rotation will have the same magnitude as that for the S-
iodide, but it will have the opposite sign: +77°.

55. Determine the absolute configuration of each stereogenic carbon in the following molecules.
O OCH3
O CH3 (R)
(a) (R)
(b) (R) (c)
(S) (S)
CHMe2
OH
OH NHMe
H (e) (f) OH
(d) (S) (R) N
Me (S) (S) (R)
Me H
Me H
CH3 N
MeO (R)
O Due to fluxional inversion,
N is racemic and should not be marked

56. Determine if each of the following alkenes has an (E) or (Z) double bond, or no stereoisomers.
Et
(Z)

(a) (b) (E) (c)


no (E)/(Z) isomers Cl
are possible Cl
HO
(E)
(d) (e) (f)
Et C3H7
(Z)

57.
(a) 3,3-Diphenyl-(4E)-nonen-1-ol (b) 2,3,4,5-Tetrachlorohex-(2Z)-ene
Cl
Ph (E)
(Z)

HO Cl
Ph
Cl Cl
(c) 3-Bromo-6-fluorodeca-(3Z,6 E)-dien-2-one (d) 3-Ethylhept-(2E)-ene
F O
(E) (Z)

Br
(E)
(e) 5-(1-Methylethyl)-4-(2,2-dimethylpropyl)dodec-(4Z)-ene (f) 3,4-Dichlorohex-(3Z)-ene

(Z)
(Z)
Cl
Cl

58.
Br Et
(Z)

(a) (Z) Br (b) (c)


CH2Cl
(Z)

2,3-Dibromo-(2Z)-pentene 3-(Chloromethyl)-(3Z)-hexene
3-Ethyl-(3Z)-octene
cis-2,3-Dibromo-2-pentene trans-3-(chloromethyl)-3-hexene
cis-3-ethyl-3-octene
overlap overlap
OH
(E) (E)
(d) (e) (f)
Et Me
(Z)

4-Ethyl-(3E)-heptene 3-Ethyl-4,5-dimethyl-(2Z)-hexene
cis-4-ethyl-3-heptene 4-(1-Methylethyl)-6-methylhep-(4E)-en-3-ol
cis-4-(1-methylethyl)-6-methylhep-4-en-3-ol

59. E = enantiomers. D = diastereomers. M = meso.


(a) 3,4-Dichloroheptane (b) 2-Bromo-3-methylhexane
CH2CH3 CH2CH3 CH2CH3 CH2CH3
(S) (R) (S) (R)
H Cl Cl H H Br Br H
E E
(R) (S) (S) (R)
H Cl Cl H H3C H H CH3
CH2CH2CH3 CH2CH2CH3 CH2CH3 CH2CH3
D D D D
CH2CH3 CH2CH3 CH2CH3 CH2CH3
(S) (R) (S) (R)
H Cl Cl H H Br Br H
E E
(S) (R) (R) (S)
Cl H H Cl H CH3 H3C H
CH2CH2CH3 CH2CH2CH3 CH2CH3 CH2CH3
(c) 4-Phenyl-3-heptanol (d) 3,4-Dibromohexane
CH2CH3 CH2CH3 CH2CH3 CH2CH3
(S) (R) (S) (R)
H OH E HO H H Br Br H
M
(S) (R) (R) (S)
H Ph Ph H H Br Br H
CH2CH2CH3 CH2CH2CH3 CH2CH3 CH2CH3
D D D
D
CH2CH3 CH2CH3 CH2CH3 CH2CH3
(S) (R) (S) (R)
H OH E HO H H Br Br H
E
(R) (S) (S) (R)
Ph H H Ph Br H H Br
CH2CH2CH3 CH2CH2CH3 CH2CH3 CH2CH3
(e) 2,5-Hexanediol (f) 3,4,5-Heptanetriol
CH3 CH3
(S) (R) CH2CH3 CH2CH3
H OH HO H (S) (R)
H OH HO H
H H M H H (s) M (s)
H OH HO H
H H H H (R) (S)
(R) (S) H OH HO H
H OH HO H
CH2CH3 CH2CH3
CH3 CH3 D D
D D
CH3 CH3 CH2CH3 CH2CH3
(S) (R) (S) (R)
H OH HO H H OH HO H
(r) E (r)
H H H H HO H H OH
(R) (S)
H H E H H H OH HO H
(S) (R) CH2CH3 CH2CH3 D
HO H H OH D
CH3 CH3 CH2CH3 CH2CH3
(S) (R)
D D H OH HO H
CH2CH3 CH2CH3 E
H OH HO H
(S) (R)
H OH HO H (S) (R)
HO H H OH
E
HO H H OH
CH2CH3 CH2CH3
(S) (R)
HO H H OH
CH2CH3 CH2CH3
CH3 CH3
(S) (R)
H Br Br H
H H H H
(g) 2-Bromo-5-methylhexane
H H H H
H CH3 E H3C H
CH3 CH3

CHMe2 CHMe2 CHMe2 CHMe2


(S) (R) (S) (R)
H CH3 H3C H H CH3 H3C H
(h) 2,3,4,5-Tetramethylhexane (R) (S) (S) (R)
H CH3 H3C H H3C H H CH3
CHMe2 M CHMe2 CHMe2 D CHMe2

60. One of the diastereomers is a meso compound, which should not be used in reactions because it is
not a chiral molecule. Only the chiral diastereomer should be used.

CO2Et Meso CO2Et


O OH OH O
(R) (S)
H OH HO H
(R) OEt EtO (S)
EtO (S) (S) (R) (R) OEt
H OH HO H
OH O CO2Et CO2Et O OH

CO2Et CO2Et
O OH OH O
(R) (S)
H OH HO H
(R) OEt EtO (S)
EtO (R) (R) (S) (S) OEt
HO H H OH
OH O CO2Et CO2Et O OH

61. Both enantiomers are shown. If ibuprofen racemizes, then it will exist as a 50:50 mixture of (R):(S)
enantiomers, and the specific rotation will equal 0°.

Ibuprofen

(R) (S) (S)


HOOC Me Me COOH HOOC Me
62.
CH3 CH3
(S) (R) (R) Br Br (S)
H OH HO H (b)
(a) (R) (S)
H OH HO H
(S) Br Br (R)
CH3 CH3 Meso
Meso
OH OH
(S) (R)
(R) (S) CH3 CH3
(S) (R)
OH OH H OH HO H
H H H H
CH2CH3 CH2CH3
(S) (R) H H H H
H OH HO H (s) (s)
(S) (R) H OH HO H
(c) H OH HO H (d)
(R) (S) H H H H
H OH HO H
H H H H
CH2CH2CH2CH3 CH2CH2CH2CH3 (R) (S)
H OH HO H
Meso
CH3 Meso CH3
OH OH OH OH OH OH
(S) (R) (S) (R) (S) (R)
(S) (R) C4H9 C4H9 (S) (R) (s) (s)

OH OH OH OH OH OH

63. In (a), (b) and (c) there are superimposable structures, either the mirror image, or the mirror image of
the ring flip conformation. Therefore, all of these are meso compounds and the structures shown
represent one single structure. Compound (d) has two enantiomers and the two different chairs are
diastereomers of each other, as are their mirror images.

(a ) cis-2-Chlorocyclohexanol (b) trans-1,4-Dimethylcyclohexane


Cl Cl
CH3 CH3

Cl Cl
superimposable

CH3 CH3
Cl Cl superimposable
H3C CH3
Cl Cl
CH3 H3C
superimposable superimposable
(c) cis-1,3-Cyclohexanediol (d) cis-1-Bromo-2-chlorocyclohexane
Br Br
OH OH
enantiomers
OH OH
Cl Cl
superimposable
all diastereomers

Cl Cl
superimposable enantiomers

HO OH Br Br
OH HO

64.
H O OH
H
H C8H19 N Me
(S)
(S) O (S) (S)
HO (R)
O
(R) (c)
(a) O (R) O (Z)
(b) (R)
O O 27 (S)
23 H 24
(R)
(S)
(S)
(R) HO (S) N
O
OH OH
HO CO2H
OH CO2H
(d) (S) (E) (E) HO2C (S) (S)
(e) (S)
(R) (R) (S) (f) (S)
(S) (Z)
HO OH (E) OH
(S)
23 23 HO2C N
24 OH
H
OH

65. Give an unambiguous IUPAC name to each of the following.


OH
(S) (S)
(a) (b) (c) (S)
(R) (E) CO2H
(R) (R) CHO
2-Methylpentan-(3R)-ol (3R,5 R, E)-5-(1-Methylethyl)-3-methyl- (2S,3S,4S)-2-Ethyl-3,4-
oct-6-enoic acid dimethylhex-5-ynal
Ph O Br
HO (S) (S) (E)
(d) (R) (e) (R) (R) (R) (f) Cl (R)
(R)

Ph Br Me Me

(1S,3S,4 R)-1-Cyclopropyl-3- (3R,5 R,7 R)-5-Bromo-3-methyl- (4R,5 R, E)-4-Bromo-1-chloro-5-


methyl-4-phenylhexan-1-ol 7-phenyloctan-2-one methylhepta-1,6-diene
66. Name each of the following
H
(S) Et
(S)
(a) (b) (R)
(c)
(Z) (s)
Et (Z)
OH H (s)

(4S)-5,5-Dimethylbicyclo- (1R,5S)-3,3-Diethylbicyclo (1s,4s)-Bicyclo[2.2.2]-


[2.1.1]hexan-1-ol [3.1.0]hexane octa-2,5-diene
Cl Me
Cl Cl H
(S) (S) (R)
(d) (e) (R) (f) (S) Me
(R) (R)
(Z)

H Cl H
(1R,5S)-9,9-Dichlorobicyclo- (1R,2S,6 R)-2,7-Dichloro- (1R,5S)-1,3-Dimethylbicyclo-
[3.3.1]non-2-ene bicyclo[4.2.0]octane [3.3.0]octane

67.
CH3 CH3 CH2CH3 CH2CH3
CH3 (S) (R) (S) (S)
H Br Br H H Cl H Cl
(S) CH3 (R) (R) (R) (R)
Cl H Br H Br H Cl H Cl
H (R) Meso CH3 CH3 Meso
CH3 CH2CH3
Br H
COOH CH2CH3 CH2CH3
(S) (R)
Meso (R)
H Br Cl H Cl H
Meso (R) (S) (S)
H Br H CH2CH3 Cl H

COOH Cl CH2CH3

68.
Mirror image (2S,3S) (2R,3R)
CH3 CH3 CH3
CH3 CH2CH3
(R) (R) (S)
CH3 Br H Br H H Br (S)
(S) H Br
Br (S) (R) (R)
Br H H Br H Br (R)
H (S) H Br
H Br CH3 CH3 CH3 CH2CH3
69.
Br CH3 HO CH3 CH2CH2OH F
(S) (R)
(S) (S) H3C CH2CH3 (H3C)3C Cl
Br Br O CH2OH Br

Br CH3
Cl H3C CH2CHMe2
H Br (S)
(R)
(R)
CH2OH H3C
H OH H Me2HC CH2CMe3
CH2CH2CH2Br
Br (S)
CH3 CN

Br HO CH3
CH2CH2I Cl
(R) (S) (R)
(R) CH2OH
H CH2CH2CH2Br
H3C OH

CH3 CH2CH2CH3
H3C CH2CHMe2 Br
H3CH2C (S) (S) CH(CH3)2
Me2HC Br
CH2CMe3
Cl (S) CN

70.

Cl Cl

(3R,4 R)-3,4-Dichlorohexane (R) (S) (3S,4S)-3,4-Dichlorohexane


(R) (S)

Cl Cl
Cl Cl

(3R,4S)-3,4-Dichlorohexane (S) (R) Two structures but one compound


(R) (S)

Cl meso Cl

71.
CH3 CH3

(2R,3 R)-2-Bromo-3-methylhexane (R) (S) (2S,3S)-2-Bromo-3-methylhexane


(R) (S)

Br Br
CH3 CH3

(2S,3 R)-2-Bromo-3-methylhexane (S) (R) (2R,3S)-2-Bromo-3-methylhexane


(R) (S)

Br Br
72.
(R)

(S)

Bicyclo[4.3.0]nonane (1R,6S)-Bicyclo- Bicyclo[3.3.1]nonane Bicyclo[3.1.1]heptane Bicyclo[3.2.2]nonane


[4.2.1]nonane

73.
OH Cl (H3C)3C CH3
(R) (S) (S)
(a) CH2CH2CH3 (b) H CH2CH3 (c) (R)
H
CH3 H
Br Cl
Br
(S) CH2CH2CH2OH
H Br Br CH3 (R)
(d) (R) (e) (f) H OH
H Br (S)
H3C CH(CH3)2
CH(CH3)2
Br

74.
(S) (R)
Et CH3 CH3
Br
HO OH (R)
Cl
(S)
Et
(S)
Cl CH3 Cl Et
OH (R)
(R) Meso
Meso (S) (R) (R)
Cl CH3 Cl Et Et CH3
(S)
(R) Cl Et CH3 Cl
OH

75.
Cl CH(CH3)2 HO CH3 Cl CH2CH3 CH2CH2CH2OH
(R) (S) (R)
(R) Br CH3 (S)
H (R) H Br HO CH3
H (R) (S)
CH2OH Cl H Br H3C CH(CH3)2 CH2CMe3
Cl
CN Br

CH(CH3)2 BrH2CH2C CH3 CH2CH2Cl CH2CH2CH2OH


Br CH3
(S) (R) (S) (R) (R)
H H CN (R) H Br H2N H HO CH3
(R) Cl (R) (S)
H3C H CH2CH2CH2Cl Cl Br H CH2CH2CH2CH2Br CH2CMe3
CH3
CH3

enantiomer diastereomer racemic enantiopure


76.

77. Since (2Z)-3-methylpent-2-ene has no stereogenic center, the specific rotation is 0°.
78.
This is 3,4-dibromo-
Meso Meso hexane
CH 3 CH 3 CH 3 CH2CH3
CH3
(R) (R) (S) (S)
(S) CH3 Br H Br H H Br H Br
Br (S) (R) (R) (R)
Br H H Br H Br H Br
H (S)
H Br CH3 CH3 CH3 CH2CH3

79.
(a) (3S,4S)-Dichloroheptane (b) (2R)-Bromo-(3S)-methylhexane (c) (4R)-Phenyl-(3R)-heptanol
CH2CH3 CH3 CH2CH3
(S) (R) (R)
H Cl Br H HO H
(S) (S) (R)
Cl H H3C H Ph H
CH2CH2CH3 CH2CH2CH3 CH2CH2CH3

(d) (3R,4R)-Dibromohexane (e) (2R,5R)-Hexanediol (f)( 3S,4S,5R)-Heptanetriol


CH3
(R) CH2CH3
CH2CH3 HO H (S)
(R) H OH
Br H H H (s)
(R) H OH
H Br H H (R)
(R) H OH
CH2CH3 H OH
CH2CH3
CH3
(g) (2S)-Bromo-(5S)-Methylhexane
(h) 2,(3R,4R),5-Tetramethylhexane
CH3
(S)
CH3
H Br This is a trick H CH3
H H question, because C5 (R)
is not a stereogenic H3C H
H H center. Therefore, (R)
there is no 5S. H CH3
H CH3
H CH3
CH3
CH3

80. Using Figure 9.15: (a) 82:18 R:S = 62% ee R (b) 55:45 R:S = 10% ee R
(c) 99:1 R:S = >99.5% ee R (d) 75:25 R:S = 50% ee R

81. S(+100°) + R(-100°) = +91°C. Assume that S + R = 1, so S = 1-R


(1-R)(100°) + R(-100°) = 91°, so 100°-100°R-100°R = 91°
100°-200°R = 91°, so -200°R = 91°-100°, and -200°R = -9°, and R = -9°/-200° = 0.045
Therefore, 4.5% R and 95.5% S. Using Figure 9.10, 92% ee (S).

82. The term 0% indicates there is no excess of one enantiomer over the other, which means that it is a
50:50 mixture (racemic).

83. It simply means that fluxional inversion is much more difficult with phosphines when compared
with amines. The P has a (S) configuration as drawn.

84. Determine the absolute configuration for each nitrogen atom in the following molecules:
OH HO CH3
H (S)

(R)
(R) N
(a) (Z) (b) (S)

(R) (S) (R)


N Ph N H
CH3 (S) (S)

CH2CH=CH2
Ph
CH3

(R)
N (S)
(S)

(R)

(c) CH3 (d) (R)


N H
(S) (R)
(S)
N
Ph
(E) (R)

H3C
H3C
OH
Cl

85.
Cl Cl
Br Br

Cl
Cl
trans- cis- trans- cis- trans- cis-
Chapter 10
Solutions Manual
Organic Chemistry. An Acid-Base Approach 2nd Edition
63. Carbon substituents are electron releasing with respect to a π-bond, so the more carbon substituents,
the more electron rich the π-bond, and the more stable it will be. The circled alkene is the most highly
substituted (the most carbon substituents).

64. Br2 HCl BH3 CH3CO3H H2O

65. Only an alkyne will react with HBr to give a vinyl bromide. The two alkynes are circled.

C≡CH

66. A peroxyacid is required to convert an alkene to an epoxide.


CH3OH CH3CO2H CH3CO3H NaOH

67. Secondary carbocations are more stable than 1° > 2° vinyl carbocation > 1° vinyl carbocation.
Least stable Most stable

68. The resonance stabilized carbocation is more stable than the 1° or 2° aliphatic carbocations.
Allylic cation
resonance stabilized
69.

3° product is more stable, so no rearrangement No other carbocation is more stable,


so no rearrangement

Initially formed 2° carbocation can rearrange to Initially formed 2° carbocation can rearrange to
a more stable 3° carbocation by a 1,2-H shift a more stable 3° carbocation by a 1,2-methyl shift

70. Of these reagents, only borane adds to alkenes or alkynes in an anti-Markovnikov manner.
HCl OsO4 Br2 BH3 H2O/HgCl2

71.

Cl2 (E) Cl
Cl

a cis-dichloride a trans-dichloride an E-mono-chloride a Z-mono-chloride


The reaction proceeds by initial formation of a vinyl chloronium ion. Subsequent anti attack by
the chloride counterion leads to the vinyl dichloride shown, with the (E)- stereochemistry.

72. Both HBr and sulfuric acid are strong enough acids to react with the alkene to yield an intermediate
carbocation. When HBr reacts, the counterion is the bromide ion and the hydrogen sulfate ion results
from the reaction with sulfuric acid. The hydrogen sulfate ion is resonance stabilized, making it a
relatively weak nucleophile, whereas the bromide ion is a very good nucleophile. Therefore, the more
nucleophilic bromide ion dominates the reaction with the carbocation, leading to bromocyclohexane.

73. 2,3-Dimethylbut-2-ene reacts faster with HCl than does but-2-ene because it is an acid-base reaction,
and the more highly substituted C=C unit in 2,3-dimethylbut-2-ene leads to a more electron rich π-bond,
which makes it a stronger base. In other words, it will react faster with HCl. In addition, the tertiary
carbocation intermediate derived from 2,3-dimethylbut-2-ene is more stable than the secondary
carbocation derived from but-2-ene, and the increased stability should lead to a lower activation energy
and a faster reaction.
Cl

Cl
74. Addition of 1 equivalent of HCl will add to 1,1-diphenylhexa-1,5-diene to give the chloride shown,
via carbocation A. Reaction with the other C=C unit generates a secondary carbocation B, whereas A is
a benzylic carbocation where the charge is delocalized into two benzene rings. Carbocation A is much
more stable as an intermediate, and will form preferentially over B, yielding the major product shown.

HCl

HCl A resonance stabilized


benzylic carbocation

Cl

75. The reaction of 2-methylprop-2-ene with acid will generate a tertiary carbocation, which is relatively
stable. Reaction of acetone with an acid generates the resonance stabilized oxocarbenium ion shown.
Two resonance contributors are drawn, one with the charge on oxygen after reaction with the acid, and
the second contributor with the charge on carbon, which is generated by transfer of the two electrons in
the π-bond to oxygen, leaving behind C+ and placing the electron pair on oxygen. In the absence of any
other information, the resonance stabilized oxocarbenium ion should be more stable that the tertiary
carbocation where here is no resonance.

H H
O O O

76. In both reactions, the tertiary carbocation is formed. In the case of HCl, chloride ion is a good
nucleophile, reacting with the carbocation to generate the chloride product. In the case of sulfuric acid,
they hydrogen sulfate anion is resonance stabilized, and a weak nucleophile. In addition, if the
hydrogen sulfate anion reacts with the carbocation, the hydrogen sulfate product is rather unstable, and
fragments to regenerate the carbocation. Under these conditions, the hydrogen sulfate anion reacts as a
base to remove a proton from the carbon adjacent to C+, generating the alkene. This latter reaction is
known as an E1 reaction, and will be discussed in Section 12.4.
H2SO4 HCl
Cl

77.

Br
HBr Br–

A 1,2-alkyl shift generates a tertiary benzylic


carbocation, which is more stable than
the original tertiary carbocation

78.
OCH3 OCH3 OCH3
Cl
HCl
An oxocarbenium ion
Although the reaction is presumably done in water, chloride ion is the best nucleophile.
Therefore, the chloride ion reacts with the initially formed oxocarbenium ion to give the chloride.

79. In A, the allylic alcohol leads to a resonance stabilized allylic carbocation, and the tertiary cation
site is more reactive when reaction occurs with the next C=C unit, to generate a secondary carbocation.
The alkyne unit in A will ultimately react with a carbocation to form a vinyl carbocation. Vinyl
carbocations are much more reactive, and will quickly react with water to give an enol, which
tautomerizes to a ketone. This latter reactions will effectively stop the cation cyclization process.
Me
O
C Me
HOCH2CH2OH ,
C ClCH2CH2Cl
Me CF3CO2H , 0°C Me H

H H

A B
Me OH Me
80. Nitrogen is not a polarizable atom, so there is essentially no driving force to react with an alkene π-
bond. Diatomic bromine has the polarizable bromine atom, and proximity to a π-bond leads to a
polarized Br—Br bond, which leads tot the ionic bromonium ion intermediate. Diatomic nitrogen is not
similarly polarized, so similar reaction does not occur.

81. The molecule shown has relatively large isopropyl groups that block the approach of the bulky Br—
Br molecule to the C=C unit. Therefore, formation of the bromonium ion is sluggish. If it does form,
the isopropyl groups will sterically hinder approach of the nucleophilic bromide ion to one of the carbon
atoms of the three membered ring bromonium ion, again limiting the amount of product formed. The
steric hindrance in the alkene is perhaps better seen in the molecular models that are provided,
especially the space-filling model on the right.

H H H H
H H H H
C=C

82. The major product or products are shown in each case. No mechanisms are provided.

HCl Cl
(a) +
Cl
HBr Br
(b)

Br
Br2 , CCl4
(c)

Br
OEt via rearrangement of the
cat. p-TsOH initially formed 2° cation
(d) then reaction with EtOH
EtOH
Cl
HCl
(e)

(f) I2 , CCl4 I

HOCl Cl Assume the reaction proceeds


(g)
aq. THF via a 2° carbocatin and NOT
OH by a chloronium ion
Br2 , CCl4
(h) Racemic
Br
Cl Br
HCl via rearrangement of the
(i) initially formed 2° cation
OEt
cat TsOH via rearrangement of the
(j)
initially formed 2° cation
EtOH , heat

83. Initial reaction with HCl generates the 2° vinyl carbocation. This reaction produces the nucleophilic
chloride ion, which is more nucleophilic than the oxygen atom of ethanol, which is the solvent.
Subsequent reaction with the nucleophilic oxygen atom of ethanol leads to an oxonium, and loss of a
proton, in an acid-base reaction (probably with ethanol as the base, or another molecule of the alkyne)
leads to the vinyl ether product.

cat H+ H—OEt - H+
O H O
84. In principle, formation of the bromonium ion, and the anticipated trans-dibromide product will
generate a trans C=C unit in the cyclooctene product. this product should be rather high in energy, due
to the constraints of the ring and the geometric demands to the trans- double bond. The predicted higher
energy of the product suggests that the reaction will have a high activation energy and/or may be
endothermic. It is difficult to predict endo- vs exothermic energy without looking at the enthalpy, and
the entropy of the reaction.
Br2
Br
Br

85. The major product or products for each reaction are shown. No mechanisms are provided.
1. BH3 , ether
(a)
2. H2O2 , NaOH OH

1. 9-BBN , ether
(b)
2. NaOH , H2O2
OH
H
1. O3 , -78°C
(c) +
2. Me2S O
O
HBr
Br
(d)
t-BuOOt-Bu

H
1. 9-BBN , ether
(e) via enol
2. NaOH , H2O2
O
HgSO4 , Hg(OAc)2
(f) +
H2O , H3PO4
O via two initially
formed enols O
HCO3H
(g)
aq THF O

1. O3 , -78°C
(h)
2. H2O2 O
HO2C
OH
OsO4 , aq t-BuOOH H
(i)
H
1. excess O3 ,
OH
OHC O
-78°C
(j) + formaldehyde
2. Me2S
O

86. Molecule A is symmetrical, so hydroboration occurs at either carbon of the C=C unit to yield to the
same product, with 9-BBN leading to two possible transition states. In one, from the “top” face, there is
severe steric hindrance between a methyl group and the bulky 9-BBN unit, whereas the other transition
state, formed from the “bottom” is essentially free of steric hindrance. Therefore, the lower transition
state predominates to give the alkylborane shown, and oxidation yields the alcohol as the major product.
Steric
hindrance
Me Me
B
H
Me Me
9-BBN
versus

Me Me
Me Me Me Me
A
Oxidation

B
B OH
H
87.
Me Me Me
cat. H+

Me Me Me
CH2 CH3 CH3

H2O H
O OH
H

88.

(a) 1-Ethylcycloheptene
OH

Ph Ph
(b) 2-Phenylbut-1-ene
OH

(c) 3,4-Diethylhex-3-ene

OH

C3H7
C3H7 C3H7
(d) 3,3-Dimethylhex-1-yne
O
OH
H

89. The major product or products are shown. No mechanisms are provided.

I2 , CCl4 I
(a)
I
1. O3 , –78°C O
(b) +
2. H2O2
O HO

OH
(c) OsO4 , Me3COOH

OH
Br2 , CCl4
Br
(d)
racemic
H
B Br

NaOH
(e)
ether H2O2
OH
HOCl OH
(f)
Via a carbocation, so
Cl a cis/trans mixture
O
O
(g) H O H + HCO2H

O Br
Br2 , CCl4
(h) racemic

Br
HBr
(i) Br

O O
O
H O H + HCO2H
(j)

I2 , CCl4
I
(k)
I

catalytic H2SO 4
(l)
H2O
OH
HBr Br
(m)

Cl via rearrangement of the


HCl
(n) initially formed 2° cation

Br
(o) Br2 , CCl4 racemic

Br Br
HBr
(p)

Cl
Cl2 , CCl4
(q) H
Cl
1. O3 , -78°C
(r)
2. CH3SCH3 CHO
O
CH3CO3H O + CH3CO2H
(s)

90. Give the major product for each reaction.


Br
HBr
(a)
Br
Br2
(b)
CCl4 racemic
Br
I
I2 , CCl4
(c)
I

OH Via a carbocation, so
(d)
HOCl a cis/trans mixture
Cl
H2O , cat H+ OH
(e)

H2O
No reaction (N.R.)
(f)

2 HBr , CCl4
(g)
Br
Cl Br
HCl
(h) via rearrangement of the
initially formed 2° cation

1. HgCl2 , H2O
(i) 2. NaBH4 no rearrangement

HO2C O OH
1. O3 , -78°C
(j)
2. H2O2

1. BH3 , ether
(k) 2. NaOH , H2O2 OH

Br
(l) Br2 , CCl4
racemic

Br
(m) 1. O3 , –78°C O
+
2. H2O2
O HO
H2O , HgSO4 OH O
(n)

OsO4
(o)
OH
NaHSO3 , H2O
OH
1. BH3 , ether
(p) 2. NaOH , H2O2 O
OH
H
I
(q) HI
HI I
(r) ether
HBr
(s) racemic, via carbocation,
Br with rearrangement from
H Me initially formed 2° cation
Me

Br
Br2 , CCl4 racemic
(t)
H
Br
OsO4
(u) OH racemic
NaHSO3 , H2O
OH
1. BH3 , ether

(v) 2. NaOH , H2O2 O


OH
H

91.
OH
1. Hg(OAc)2 ,H 2O
(a)
2. NaBH4
Ph Ph Ph
1. Hg(OAc)2 , MeOH OMe
+
(b)
2. NaBH4
OMe

(c) 1. Hg(OAc)2 , H2O OH


2. NaBH4

OEt
(d) 1. Hg(OAc)2 , EtOH

2. NaBH4

92.
1. BH3 , ether
(a) OH
2. NaOH , H2O2

1. Hg(OAc)2 , H2O
(b)
2. NaBH4
OH
OH
1. Hg(OAc)2 , H2O
(c)
2. NaBH4

1. BH3 , ether OH
(d)
2. NaOH , H2O2

1. O3 , -78 °C
(e) CHO + HCHO
2. Me2S

1. O3 , -78 °C
(f) O
2. Me2S +
CHO

Br I
1. I2 , CCl4
93.
2. Br2 , CCl4
Br I

94. Under acidic conditions, methoxyethene will react to form a rather stable oxocarbenium ion. The
increased stability, relative to the carbocation formed from ethene, will make subsequent reactions with
additional molecules of alkene slower.

95. The C=C unit reacts with the radical formed from AIBN to form a carbon radical, but the radical
may form on either of the two carbon atoms. Both A and B are secondary radicals, and should have
approximately equal stability. Since there is no difference in relative stability, it is anticipated that both
will form, in roughly equal amounts.

AIBN
+
A B

Read and understand Chapter 25 before attempting 96–99 and 114–117.

96.

1. Hg(OAc)2 , H2O H3O+


(a) (b)
2. NaBH4 OH
1. Hg(OAc)2 , H2O
(a)
2. NaBH4 PBr3

H3O+ OH Br
(b)
97.

98. 1. O3 , -78 °C CHO


2. Me2S CHO

99.
CH3CO3H O
+ CH3CO2H

O
O O

1. BH3 , ether
100.
2. NaOH , H2O2
OH

101.
1. O3 , -78 °C O O
+
2. H2O2 OH

102.

1. O3 , -78 °C
O
+
2. Me2S
O H
103.
OH
aq KMnO4 , NaOH

OH OH Hexane-2,3-diol
aq OsO4 , t-BuOOH

OH

104.
O
cat H+ , H2O/ether
C C

H+
H H
OH
H2O O
C C
H C C
C C –H+ H
H

105. The (E)-alkene has fixed stereochemistry because there is no rotation about the C=C bond. The
reaction proceeds via formation of a bromonium ion, which is a three-membered ring that does not allow
rotation, so the stereochemistry of all groups is fixed. The second bromine adds anti to the first one,
which means that it adds from only one face. Given that the stereochemistry of the groups is fixed in the
alkene, and fixed in the bromonium ion, anti attack leads to only one stereoisomer, the meso compound.
Br Br
(E) Br2 (R)
Et (E) Et H Et H Et H
Et Br – (S)
H Et H (S) (S) Et H Et
Br
Both (3S,4 R) and (3R,4S)
It is a meso compound

106.

F
(a) Br (E)
(b) (Z)

F
Br
2,9-Dibromo-3-methyl-4-(2,2-dimethylpropyl)non-(3E)-ene 1,2-Difluorocyclohexene
(c) (Z) (d) (E) (e)

3,4-Dimethyloct-(3Z)-ene Hepta-1,(5E)-diene 4-Phenylhex-1-yne

107.

catalytic H+
H2O
OH
H+
–H+
1,2-methyl shift OH2

OH2

108. The reaction of 3,3-dimethylpent-1-ene and BH3 leads to two transition states, A and B. In B,
there is more steric hindrance of boron and the carbon toms than in A, so A leads to the final product,
which is the borane shown, which has the BH2 group on the less substituted carbon atom.

BH2
A BH2
BH3 H

B H
B
H2

109.
H3C O
H3O+
H3C C C CH3 H2C C
CH3
H+
–H+
OH2 OH2 OH
H3C C C CH3
H H3C C C CH3 H3C C C CH3
H H
110.

H2O , catalytic H+

OH
H+
–H+
1,2-ethyl shift

OH2
OH2

111. HBr Br2 BH3 H2/Pd SOCl2


The reaction proceeds via an initially formed bromonium ion, and only diatomic bromine will
generate this intermediate. Since HBr reacts to form a carbocation, cis/trans has no meaning, and borane
gives a cis product whereas catalytic hydrogenation gives a mixture of cis and trans.

112.

O
2 O Initiation step
O

H—Br
O HO + Br

Br2 H—Br
+ Br
Br Br

+ Br Br
Br A termination step - it does Br
not give a major product

113.
1,2-H shift
HCl + Cl-

Cl
1. O3 , -78 °C
CO2H
114.
2. H2O2
CO2H

1. Hg(OAc)2 , EtOH EtO


115.
2. NaBH4

O
CH3CO3H
+ CH3CO2H
116.

cat H+
117.
H2O , THF
OH

Spectroscopy Problems. Chapter 14 must be read and understood before attempting these
problems.

118.

8 6 4 2 0
PPM
Ph Ph
Ph

A OH
OH
IR is similar for both
A compounds, but the proton
NMR is simpler and
symmetrical for B, but not
8 6 4 2 0 for A.
PPM
119. The 1H NMR for A shows one ethyl group (two signals) since it is symmetrical. The 1H NMR
for B shows four signals including a unique singlet worth three H atoms: a methyl group with no
neighbors.

3 2 1 0
PPM

O
B
O
A
IR is similar for
A both compounds

3 2 1 0
PPM

120.

3 2 1 0
OH PPM
OH Oxidative O
Cleavage

IR, 3300 cm –1
IR, 1725 cm –1

Chemical shift differences in


the NMR: >2 ppm for CH2 in the
ketone, but about 1.5 ppm for CH2
4 2 0 in the diol, and thr diol will have a
PPM signal at about 3.8 ppm for HO-CH
121.

no distinguishing signals
for either bromide in the IR 3 2 1 0
PPM

Br

Br

The most distinguishing


feature of this product is
the methyl group, which
is missing in the other
4 2 0 bromide
PPM

This is characterized by a downfield CH2


group of the bromomethyl, which is missing
in the other bromide

122. Hex-2-yne

123. 3-(Bromomethyl)pentane

124. C4H8O, Butan-2-ol

125. 1,5-Dichloropentane.

starting material, based on spectral data


126.
Br2
Br
or
A

127. 4-Methylpentan-2-ol
1. O3 , -78 °C 2. Me2S
O + HCHO
128. H A O
H O O H
Ozonide
Precursor to A
Chapter 11
Solutions Manual
Organic Chemistry. An Acid-Base Approach 2nd Edition
47.

48. Note that diethyl ether and THF are aprotic and will best facilitate second order reactions, but first
order reactions are most facile in water and slow in virtually all other solvents. The way the question is
worded, ethanol and formic acid allow second-order reactions to proceed faster than water, and so they
may also be circled.
O
O H2O
OH
O H OH

49. The circled carbocation is a primary carbocation, which is much less stable than the secondary or
tertiary carbocations that constitute the other choices. There are fewer stabilizing alkyl groups on the
primary carbocation.

50. The circled primary bromide is the most reactive because it offers the least amount of steric
hindrance in the pentacoordinate SN2 transition state, relative to the transition states for the other
choices.

Br

Br
Br Br
51.
Br Some SN1 here
Br NaI
NaI , THF
EtOH–H2O
Br Br
NaI , THF KCN , EtOH

No reaction here

52. Methane has no electrons to donate, and water and methanol are neutral molecules. Although both
of these neutral molecules are nucleophilic, the higher concentration of electron density on the
methoxide anion makes it the strongest nucleophile. The hydrogen sulfate anion is a weak nucleophile
due to resonance delocalization of the excess electron density.
CH3O– H2O CH4 CH3OH HSO4–

53.
SN1 SN2 acid-base radical addition

54. In water, ionization would give a primary carbocation, which is very unstable. It is also true that the
energy required to achieve such an ionization is too high. Therefore, an SN1 reaction is very unlikely.
Water is a weak nucleophile, and SN2 displacement of the bromide would give an oxonium ion.
Loss of a proton would then give the alcohol. This is unlikely due to the weak nucleophilicity of water
in this reaction, and the very slow rate of this particular reaction. It is more likely that there is simply no
reaction at all.

55. This is an SN2 reaction, and it proceeds with 100% inversion of configuration at the stereogenic
carbon.
(S) (R)
NaCN

Br CN

56.
Me Br KI Me Excessive steric hindrance in the
pentacoordinate transition state makes
I Br the activation energy for the reaction
Me Me
Me Me so high that it does not proceed

57. 2-Phenyloxirane reacts with ethanol and an acid catalyst to give an oxonium ion, but it may open to
form the resonance stabilized benzylic carbocation shown. If ethanol reacts with the oxonium ion,
attack should be at the less hindered carbon (path a). If the oxonium ion opens to give the carbocation,
however, then ethanol will attack the positive carbon to yield the observed product, B.
OEt
OH MAJOR
OH
Ph B
H+ Ph b
O
H
Ph
O a OH

Ph A
a Ph
b
HOEt HOEt
OEt

58.
OH2 OH

H+ H2O -H+
O O H
OH OH OH

59. The three-membered ring oxirane is much more strained than the four-membered ring oxetane.
Reactions with HI will open both rings, but the greater relief of strain in the oxirane makes that reaction
faster than the identical reaction with oxetane.
O I O
HO HO I

60. In the presence of triethylamine and thionyl chloride, butan-(2S)-ol is converted to (2R)-
chlorobutane because in proceeds with inversion of configuration due to the presence of the nucleophilic
chloride ion in the reaction medium, by a SN2 reaction.
(R) (R) (S) (S)

Cl NEt2 Cl OH

61.
Br Ph
(a) (b) OSO2CH3 (c)
I
(2R)-Bromo-4-phenylhexane Cyclopentyl methanesulfonate cis-1-Ethyl-2-iodocyclohexane

Ph Ph
Br Br
(d) (e) O CF3 (f)
S
O O
O
1-Bromo-4,4-diphenylheptan-3-one Pentan-2-yl trifluoromethanesulfonate 3-Bromo-3-ethylhexane

62. There is a high concentration of charge density on oxygen in methoxide, which makes it rather
reactive (relatively unstable) when compared to the methanesulfonate anion, which is resonance
stabilized by delocalization of the charge density. This stabilization of the ion after it leaves contributes
to its being a good leaving group. In addition, the C—O bond of the methanesulfonate is longer and
weaker than the C—O bond of methoxide because the sulfur atom is larger, and the sulfonate group is
larger and more electron withdrawing. The longer and weaker bond is easier to break, contributing to a
better leaving group.

63. 1-Bromo-2-cyclohexylethane is a “normal” primary alkyl halide, as is 1-bromo-2-cyclohexyl-2-


methylpropane. However, the pentacoordinate SN2 transition state for 1-bromo-2-cyclohexyl-2-
methylpropane is somewhat more sterically crowded that that for 1-bromo-2-cyclohexylethane, so the
reaction is expected to be slower, which means that the reaction will have a longer half-life.
H H
I Br CH3 I Br
Br H Br H
CH3

64.
(a) NaN3 , THF

Br N3
Via 2° cation that
(b) rearranges to 3°
Ph KI , aq THF , heat Ph
cation. Sterochemistry
I is lost once the planar
Br carbocation is formed.

1. MeSO2Cl , NEt3
(c)
2. NaCN , THF

OH CN
NaCN , THF , 0 °C
(d) No reaction (N.R.)

Br Assume smaller Et migrates


Ph via rearrangement of
Ph Ph KI , aq THF , heat Ph the initial 2° cation
(e)
Br (1,2-ethyl shift).
I
Note the Ph shifts are common
and often preferred, but the text
HI assumed the smaller group migrates
(f) +
OH I
O

65. In 1,3-dibromo-4,4-dimethylpentane, one bromide is a relatively normal primary bromide, but the
other is a neopentyl bromide. Neopentyl bromide is extremely unreactive in SN2 reactions due to
excessive steric hindrance in the pentacoordinate SN2 transition state. Therefore, the reaction with the
simple primary bromide is much faster and leads to the major product.
A neopentyl bromide unit

KI

Br Br Br I

A primary bromide unit

66. Of the solvents used in this book, for all practical purposes water is the only one that can ionize the
halide so an SN1 reaction can occur. Ionization is very slow in ethanol, so the SN2 process dominates
the reaction that leads to the major product because it is faster.

67. Since this is an SN2 reaction, the rate depends on the concentration of both RX and the nucleophile,
NaX. Increasing the concentration of one of them will allow the rate to increase and the reaction to be
completed faster. Since NaX is the cheap component, increasing the concentration of NaX to 10 equiv
for 1 equiv of RX will diminish the reaction time from 100 to 10 h. Adding more NaX can adjust the
rate further is necessary.

68. The boiling point of 1-iodopentane is 155–157 °C , DMF is 153 °C, THF is 66 °C, acetone is 56 °C
and diethyl ether is 35 °C. The dielectric constant of DMF is 36.7, THF is 7.6, acetone is 20.7 and
diethyl ether is 4.3.
The best solvent for this reaction is DMF, but the boiling point is too close to the product for
facile separation. The solvent will play an influential role, even if the product is isolated by liquid-liquid
(column) chromatography. Diethyl ether has a low boiling point and is easily removed, facilitating
isolation of the product, but the low dielectric indicates that the reaction will be slower. Acetone often
contains water and is difficult to dry. The best choice is probably THF. Note that DMF is a perfectly
good solvent and there are methods for separating product from solvent, it just may take more work.
69. The product of the reaction with diethylamine is an ammonium salt. Therefore, the transition state
has a different charge distribution that the normal SN2 transition state, as shown. In water, which
separates charges, charge separation will accelerate the reaction, whereas THF as a solvent does not
provide stabilization at all.
Et H Et Et
H δ−
δ+ δ− δ+ δ−
Br H N N Et N C Br I C Br
Et
Br Et H H H H
Et Et

70. The nitrate anion (NO3-) is resonance stabilized, so the charge is dispersed. If the charge is
dispersed over several atoms, then it is more difficult to donate electrons to an electrophilic atom, which
means it is less nucleophilic.
O O O
O N N
N
O O O O O

71. The iodide ion is a nucleophile and replaces the bromine atom in an SN2 reaction. Iodine is also a
better leaving group relative to bromine, so it is more reactive with the azide anion, which accelerates
the reaction. When iodide ion is displaced by azide ion, it once again becomes available for reaction
with another molecule of the alkyl bromide.

72. Reaction via carbon gives the usual nitrile, whereas reaction at nitrogen generates what is known as
an isonitrile. The formal charge on nitrogen in an isonitrile is +1 and the charge on the carbon is -1.
The isonitrile is charged, and presumably will form more readily if water is the solvent, whereas the
nitrile should form more readily in an aprotic solvent. If there is a mixture of nitrile and isonitrile, the
isonitrile can be removed, in principle, by simply washing the mixture with water since the more polar
isonitrile should be more soluble in water.

C≡N
Br C≡N Nitrile

N≡C
Br N≡C Isonitrile
73.
(R)

(S)
NaNH2 Br
:–

74. The secondary halide is less reactive than the primary halide due to more steric hindrance in the
pentacoordinate SN2 transition state.

75. Bromine is much larger than fluorine, so the C—Br bond distance is greater than the C—F bond
distance, so it is weaker. In addition, the bromide ion is much larger than the fluoride ion, so after it
leaves there is greater charge dispersal for bromide ion, which makes it more stable.

76. Many, if not most, organic molecules are insoluble in water, and alkyl halides are notorious for
being insoluble in water. On the other hand, water is required for ionization reactions. The solution is
to mix the water, or the ionization, with an organic cosolvent, to solubilize the reactants.

77. Ionization would generate the cycloheptatrienyl cation, which has four resonance contributors. This
carbocation is very stable due to charge dispersal, and the great stability indicates a low activation
energy and facile formation. The product is a nitrile generated by reaction of the nucleophilic cyanide
with the carbocation.
Br CN
78. Ionization of 4-phenyl-3-buten-2-ol generates a resonance-stabilized carbocation, with five
resonance contributors. The charge is delocalized into the benzene ring. The increased stability of the
intermediate means that formation of this intermediate is facile, which accounts for the rapid reaction in
aqueous solution. In aqueous solution, the primary alkyl bromide does not undergo ionization to a
primary cation, and the aqueous solution slows the SN2 reaction.

OH
I +

4-Phenylbut-(3Z)-en-2-ol I
E+ Z E+ Z

OH
I

4-Phenylbutan-2-ol

79. Ionization of butan-(2S)-ol leads to a planar carbocation, which in turn leads to the racemic 2-
iodobutane. If the observation is that 4,4-diethyl-2,2-dimethyl-3-(1-methylethyl)hexan-(3S)-ol
undergoes ionization to a cation but gives more of the (S)-enantiomer than the (R), then the carbocation
cannot be planar. In fact, the steric hindrance of the groups attached to C+ prevents planarity, so the
incoming nucleophile approaches from the most open face. The molecular model shows the steric
hindrance, and there is a slight distortion from planarity for the C+.
OH aq THF I

KI

OH aq THF I
KI
80. Ionization to the planar carbocation destroys the stereogenic center, and after rearrangement via a
1,2-hydride shift, the tertiary carbocation is symmetrical and achiral. In other words, when iodide ion
reacts to form the product, the carbon bearing the iodine atom is not chiral because it also bears two
propyl groups, making it a symmetrical molecule.
H
H2O–THF I
(R) (S) Achiral
KI , heat
H Br
–Br -
+I-
H
1,2-H shift
(S)

81.

OH HCl Cl
(a)
+H+
+ Cl –
OH2 - H2O

cat H+ , H2O

(b)
O OH
+H+ OH
- H+
+ H2O OH2

O
H OH OH

82. This result implies that the reaction is not ‘pure’ SN1, but rather proceeds by two different
mechanisms. Some of the reaction occurs by SN1, via ionization to a carbocation, and product arises by
that mechanism should not be influenced by the concentration of the nucleophile. Some of the product
must arise by an SN2 mechanism, in which case an increase in nucleophile concentration will have an
influence on the rate. In this book, there is a tendency to categorize reactions as 100% one thing or
another, proceeding by one distinct mechanism. In real life, there are reaction conditions and molecules
that react by more than one mechanism, which is the case in this example. It is one of those things you
should keep in the back of your mind if you actually run an experiment in the lab, although for the
purposes of doing the homework problems in this book, assume a single mechanism is operative.
83.

cat. H+
HO OH
(a) O
Pinacol
H+ - H+

- H2O 1,2-ethyl HO
HO OH2 HO shift

OH
(b) cat. H+
Pinacol
HO - H+
H+ O

OH OH 1,2-akyl OH
- H2O shift

H2O
HO
cat. H+
(c) OH OH
aqueous THF Demjanov
- H+
+ H+
1,2-alkyl
- H2O shift H2O
OH2 OH2

OH
(d) OH
cat H+ , aq THF
- H+
OH2

+ H+
OH2
- H2O + H+
+ H2O

- H+
Counterintuitive. To get this product, elimination must give the alkene and reaction with
H+ must give the less stable 2° carbocation, which can react with water. The reaction of the
alkene with H+ should be reversible, so it is not entirely unreasonable.
84. The products are the carbocation and nitrogen gas, which is a remarkably stable molecule.
Therefore, N2+ is a remarkably good leaving group, which facilitates ionization to the carbocation.
R-N2+ = R-N≡N+ → R+ + N≡N (nitrogen gas)
NH2 N2
(a)

2-Aminopentane
OH2 OH

NH2
N2
(b)

1-Amino-2-methylcyclohexane

OH2 OH

(c)
H2N N2

1-Amino-2,2-dimethylpropane

OH2 OH
85.
Me Me Me
Cl H+ Me
A Cl B
Me
Me H

Me Me Me Me

Me
Me Me
Me Me

86. As with question 82, there is a mixture of two mechanisms: mostly SN2, with some SN1.
Remember that the ammonium salt is an ion, and in aqueous media some ionization to the carbocation is
possible. This ionization is responsible for the observed stereochemical mixture of products.

87. The major product or products are given. No mechanisms are provided.

HBr
(a) OH Br

OH Cl
SOCl2
(b)

1. B2H6 , THF NaNH2 reacts with the alcohol


(c) 2. H2O2 , NaOH to give an alkoxide, which reacts
3. NaNH2 with CH3I by an S N2 reaction.
4. CH3I , THF OCH3

(d) NaN3 , THF

Br N3
(e) Cl2 , light
Cl

POCl3
(f) OH Cl
Br I
KI , H2O-THF
(g)

Cl NaBr , aq THF Br
(h)
reflux
OH
cat TsOH Via O H
(i) O NaCN , THF CN

(j) cat H+ , H2O NC


OH THF , NaCN

Br OCH3
CH3O –Na+ , THF
(k)

OH N3
1. I2 Pred
(l) Via iodide
2. NaN3 , THF-H2O
OH Br Via both S N2
(m) HBr and S N1, so loss
of stereochemistry
I CN
NaCN , THF
(n)

I CH3 NaN3 , THF


No reaction (N.R.)
(o) 0 °C

Cl OEt S N2 impossible, so slow


NaI , EtOH
(p) ionization inprotic EtOH
reflux , 200 d and S N1 with iodide ion

Br NaN3 , THF , heat N3


(q)

NaN3 , THF
Via S N2-like reaction of azide ion
(r) at less substituted carbon of epoxide
OH
O N3
88. The major product or products are given. No mechanisms are provided.
KI , THF
(a) Via S N1 with
H2O +
Cl I allylic carbocatin
E+ Z I
1, HBr
(b)
t-BuOOt-Bu Via 1° bromide, then S N2 with CN
2. NaCN , DMF
CN
(c) NaH , THF An internal S N2 via alkoxide
OH Br O O
O
1. 9-BBN , ether
2. H2O2 , NaOH Via 1° alcohol, then conversion to
(d) N
3. SOCl2 chloride, then S N2
4. K-phthalimide
O
OMe
OH
(e) conc. HBr
+ CH3Br

1, NaH , THF I
(f) 2. 2S-bromobutane HI
O OH
OH 3. HI (S)
Br
Via (R)
(S)

(g) 1 equivalent NaCN


THF
Cl Cl
I CN
(h) (R) KI , aq. THF S N1 product, with
heat rearrangement
Br I

CH3
1. NBS , hν
(i) 2. HC≡C – Na+, THF
3. NaH , THF
4. CH3I
1. HCO3H
(j) 2, NaCN , DMF CN Via CN opening of intial epoxide
3. dilute H3 O+ OH
HI
(k) OH + CH3I
OMe
I
1. CH3CO3H
2. NaI , heat
(l) Via I opening of intial epoxide
3. dilute H3O+
OH

89. Radical chlorination of cyclohexane gives only one product because there is only one type of
hydrogen. There are 12 hydrogen atoms, but they are all chemically identical, hence, one product.
Hexane has three different types of hydrogen atoms: six methyl hydrogen atoms and two sets of
different methylene atoms. Therefore, there are six identical methyl protons, four identical CH2 protons,
and another set of four identical CH2 protons, and radical chlorination gives three different products.
Cl

Cl

Cl
Cl

90. The lack of selectivity is due to formation of allylic radicals. There are two different kinds of
hydrogen atoms that lead to two different allylic radical. Each radical will lead to bromination at two
carbon atoms, but in one case the same bromide is formed. Therefore there are three different products,
although in two of the products, they will exist as (E) and (Z) isomers, so in reality there are five
different products.
E+Z
1-Bromopent-2-ene 3-Bromopent-1-ene
Br Br

+
Pent-(2E)-ene

Br Br
4-Bromopent-2-ene 4-Bromopent-2-ene

E+Z E+Z
91. For 3-methylpentane, there is no great difference in the rate of hydrogen abstraction for the different
types of hydrogen atoms. There are four different types of hydrogen atoms, so there are four different
products. In the case of 3-phenylpentane, however, one of the hydrogen atoms is benzylic, so formation
of the corresponding radical is very fast because of the resonance stability of the radical. Therefore,
there must be a large rate preference to form the benzylic chloride shown, which is the major product.
Cl Cl

Cl Cl

benzylic H

Cl

Synthesis. Do not attempt the following until you have read and understood Chapter 25.

92.
O 1. PBr3
O
2. Na phthalimide, THF
(a) O O
3. N2H4
OH NH2

OH 1. SOCl2 NMe2
(b)
2. Me2NH

1. SOCl2
2. Na phthalimide, THF
(c)
3. N2H4
HO H2N

Br 1. Na phthalimide, THF
(d) N
2. N2H4 H
3. PhCH2Br
93.
1. PBr3
(a) OH O
2. CH3CH2CH2CH2O –Na+
THF

1. PBr3
(b)
OH 2. NaCN , DMF CN

1. NaOEt , THF Br
(c) Br
2. Br2 , CCl4 Br O

1. HI
2. NaH , THF
(d) CH3 O
O 3. PhCH2Br

94.
1. HBr
(a)
2. NaOEt , THF O
1. NaNH2
2. 2-iodobutane
(b) H H
3. NaNH2
4. iodoethane
1. PBr3 CH3
(c) OH
2. MeC≡C: –Na+ , THF

1. PBr3
2. NaCN , DMF See Sections
(d) 19.3-19.5 for hydrogenation
OH 3. H2 , Pd-C
NH2
O
1. NaNH2
2. EtCH=CHCH2Br , THF
(e) N-H
3. N2H4
NH2

O 1. MeCO3H OMe
2. K phthalimide
(f)
3. NaH 4. CH3I
5. N2H4 NH2

Spectroscopic problems. Do not attempt these problems until you have read and understood
Chapter 14.
95.
Both C=C units will absorb
at about 1650 cm –1 in the IR

6 4 2 0 6 4 2 0
PPM PPM
2 Methyl groups and 2 alkene H 1 Methyl group and 3 alkene H

Br CN

(S) (R)
(2S)-bromohexane
IR; 2230 cm–1

3 2 1 0
PPM

Br CN

(S) (R)

2S-bromohexane
IR; 2230 cm–1

3 2 1 0
96. PPM

97.
OH OH

IR: 3300 cm –1
for both

4 2 0 4 2 0
PPM PPM
Unsymmetrical, so a more complicated The symmetry leads to a simple 1 H NMR spectrum
1H NMR spectrum. No singlet methyls and that includes a singlet methyl. No protons
there is a CH downfield, adjacent to the OH on a carbon adjacent to O suggests 3° alcohol
98. N,N-dimethylethanamine, C4H11N.

99. Hex-2-yne, C6H10.

100. Dipropyl ether, C6H14O.

101. 2-Ethyl-2-methyl-1,3-propanediol.

102. 2-Bromohexane.

aq H+ 1. NaH

B OH 2. CH3I C OCH3
103. A

104. 2-Methylbutanenitrile, C5H9N.


Chapter 12
Solutions Manual
Organic Chemistry. An Acid-Base Approach 2nd Edition
25. The circled alkene has more electron releasing alkyl groups on the C=C, which makes the π-bond
stronger, making the alkene is more stable.

26.
KOH , EtOH NaCl , ether NaNH2NH3

EtOH , NaOEt aq H2SO 4 H2O , 0 °C

27.
Br
Br
Cl

28.

1° 2° 3° 2°

29.
Cl
Br
(a) OH (b) (c) (d)
CH3 OH
2-Chloro-1-methylcyclopentan-1-ol 2-Bromohex-(2Z)-ene 2,3,4,6-Tetramethylhept-(3E)-ene Non-8-yn-3-ol

30.
Br
Br Br

Br
31. Cl– HO– CH3O– I– CH4

32. Although (2S)-bromopentane is chiral and has one stereogenic center, the β-carbon is NOT a
stereogenic center, and both protons are removed in an E2 reaction. Removal of one of the protons leads
to the (E) isomer, whereas removal of the other proton leads to the (Z) isomer. The rotamers for
removal of each β-proton are shown, and the alkene product resulting from each. Both products are
produced.
H
H (S) CH3 (E)

Br H3CH2C H
Br

(S) H
H3CH2C (S) CH3

H H (Z)
Br

33. A C=C unit attached to a bridgehead carbon is significantly more strained than one that is away
from the bridgehead. A C=C unit, made of sp2 carbon atoms that are trigonal planar, will flatten that
region of the molecule. Forming the C=C unit toward the bridgehead demands that those carbon are
flattened, which requires severe distortion and is too high in energy to occur in this system. Therefore,
an E2 reaction with 2-bromobicyclo[2.2.1]hexane will give bicyclo[2.2.1]hex-2-ene as the major
product.
KOH , EtOH
Br
2-Bromobicyclo[2.1.1]hexane Bicyclo[2.1.1]hex-1-ene
Bicyclo[2.1.1]hex-2-ene
Major product This alkene is NOT formed because
it would require severe distortion
at the bridgehead carbon atoms
34. The alcohol is more acidic than the β-hydrogen, so it is reasonable that reaction with ethoxide will
generate an alkoxide base. An SN2 displacement of the nearby bromine atom is possible, but this
reaction would form the four-membered ring ether (A). Formation of a strained four-membered ring is
rather difficult due to the higher energy of that strained ring. Remember that formation of the alkoxide
is a reversible acid-base reaction, and an alternative reaction is possible. Reaction of ethoxide with the
β-hydrogen atom next to the bromine-bearing carbon will initiate an E2 reaction to give alkene B.
Given the high-energy requirements for generation of A and the reversible nature of alkoxide formation,
it is likely that under these conditions the alkene (B) is the major product via the E2 reaction.
H O– H
O

NaOEt , EtOH
H A
OH Br
H OH
H
NaOEt , EtOH

H
Br B

35. The six-centered transition state of 2-ethylmalonic acid required for decarboxylation effectively has
the oxygen of one carbonyl attack the acidic proton of the other. In 2-ethylbut-3-enoic acid, the
requisite six-center transition state requires that an alkene unit attack the acidic proton of the carboxyl
group. A simplistic idea is that the carbonyl oxygen is more basic than the alkene unit, so the reaction
should be faster, and occur under milder conditions. The product of the dicarboxylic acid reaction is an
enol, whereas the product of the alkene-acid is an alkene. The enol is more stable due to the electron
releasing oxygen atoms, and remember that this enol tautomerizes to the stable carboxylic acid unit.
Therefore, the product is more stable, which also drives the reaction and makes decarboxylation of the
dicarboxylic acid more facile (it occurs at a lower temperature).
O
O O
- O=C=O H
H O - O=C=O
O
O H
O CH3
O CH2
H
H
36.
KOH
EtOH

Br
KOH
EtOH
H A tetrasubstituted alkene, so it
is more stable than the di- or tri-
substittuted alternatives. This alkene
will be the major product
Br

37. The two possible reactions are E2 and SN2. There are no β-hydrogen atoms, so no acid–base
reaction is possible and an E2 reaction cannot occur. For an SN2, a pentacoordinate transition state is
required, and the bulky groups raise the activation energy of that reaction so high that it does not occur.

Br

HO Br
H
Br H

No β-hydrogen atoms
Too much steric hindrance,
so no E2 so it is so high in energy
that reaction does not occur

38. Hydrogen bromide reacts with the alcohol to form an oxonium ion, which is either displaced by the
nucleophilic bromide ion to yield the bromide by an SN2 pathway, or ionization to a carbocation is
followed by reaction with the nucleophilic bromide ion by a SN1 pathway. When sulfuric acid reacts
with the alcohol, an oxonium ion is also formed, but the counterion is the hydrogen sulfate anion. This
anion is resonance stabilized and is not very nucleophilic. Therefore, either the SN2 or SN1 pathways
are problematic. In addition, the sulfate ester formed via the SN2 pathway is very unstable and will
ionize to form a carbocation. The hydrogen sulfate anion is basic enough to initiate an E1 reaction via
the carbocation, which leads to cyclohexene.
OH Br
H2SO4 HBr
39. In this reaction, 3-methylhept-2-ene is the product that is expected fro an E2 reaction. However, the
leaving group (NMe3) is tethered to the base (hydroxide), so hydroxide can only remove the β-hydrogen
atom via an eclipsed rotamer. This restriction means that the lowest energy transition state will lead to
product, which is the one with the least steric crowding, which leads to the less substituted alkene, 3-
methylhept-1-ene, as the major product.
Br NMe3 Br NMe3 OH

3-Methylhept-1-ene
H NMe3 OH
OH Leads to 3-methylhept-1-ene
H3CH2CH2CH2C CH3 H3CH2CH2CH2C(Me)HC H via a less stericallycrowded
NMe3 H transition state, so this reaction
Leads to 3-methylhept-2-ene,
via a more stericallycrowded is FASTER
transition state, so this reaction H CH3 H H
is SLOWER

40. Draw the major product expected from each of the following reactions.
Br
I
(a) KI , H2O-THF

(b) NaOEt , EtOH (E)


E+ Z
Br
1. SOCl2
(c) E+ Z
OH 2. t-BuOK , t-BuOH (Z)

(Z)
OH conc. H2SO 4
(d)

CH3 CH3
I CH3
KOH , EtOH +
(e) (S) (E)
(E) E+ Z
E+ Z
Et
Ph Me Br Ph Et
(R) KOH , EtOH
Br H (R)
(f) (S)
(S)
(Z)
Me H Me
H Et H
Ph
Me C3H7Br (Z)
(g) (S) (S) NaOEt , EtOH
(R) (R)

H H Me
Br
Br
Me Br
(h) (R) (R)
KOH , EtOH
(R) (R) (E)

Me H Me H
NEt3 Cl
(i) SOCl2
(S) KOH , EtOH (E)
(R) (S)
(S)
H H
OH chloride forms with inversion
Br (E)
(j) CHMe2
(S) (S) (R)KOH , EtOH
(R) (S)
(S)

Br
Br
(k) CHMe2 KOH , EtOH No E2 reaction
(S) (R)
(R) (R)
(R) (S)

Br Me
Me I
Ph I t-BuOK , t-BuOH
(l) (R) (S)
(S) (R) (S) (R) (E)
(R)
Ph
Ph H Me H
NMe3 OH 200°C
(m)
(R)

41. An E2 reaction with 2,3-dibromopentane will yield the more highly substituted alkene because the
reaction is under thermodynamic control. Therefore, elimination will occur by removal of the β-
hydrogen on the carbon bearing the bromine. If heated for a long time with an excess of base, the vinyl
bromide can also undergo elimination, to form an alkyne. Elimination to form the alkyne requires
removal of the β-hydrogen atom, and removing the β-hydrogen atom from a sp2 carbon on a C=C unit is
more difficult than removing a β-hydrogen atom from an sp3 carbon. In other words, the hydrogen atom
on an sp3 carbon is more acidic than the hydrogen on an sp2 carbon.
Br
Br

Br
42.
Br

No β-hydrogen atoms,
so no elimination
Br

43. There is only one β-hydrogen, and removal by the base will generate a new C=C bond, but this will
also generate an allene. Note that this question is more hypothetical, and that formation of allenes this
way is not as easy as this reaction suggests because loss of Br from a sp2 carbon is requires higher
energy, which usually requires significantly more heating for an extended time.
H KOH
C

Br

44. Loss of water from the initial oxonium ion leads to the carbocation. This benzylic C+ unit will
delocalize the charge into all three benzene rings (resonance), and the extensive charge delocalization
makes it very stable.

Ph Ph H
H+
Ph OH Ph O
H
Ph Ph

45. The reaction conditions stated in the problem favor an E1 reaction, which requires ionization to give
a carbocation. Formation of a carbocation, which has a sp2-hybridized carbon atom requires flattening
of the molecule at the atoms attached to C+. In this case, C+ is a bridgehead carbon atom, so flattening
would lead to significant distortion of the bicyclic system. This requirement raises the activation
energy for ionization and formation of the carbocation. If the carbocation does form, an E1 reaction
would lead to one of the two alkene products shown. A π-bond in this molecule requires flattening at
the bridgehead carbon, which is not possible because it would require extreme distortion of the bicyclic
system. In other words, the energy required to form the alkene product is prohibitively high. The
energy barrier imposed by the required distortion to form the carbocation intermediate and/or the alkene
products makes the reaction very difficult, if not impossible.

H2SO 4
or

OH
46. Examining the structure shown for 2-methylbutanedioic acid indicates that if one carbonyl attacked
the acidic proton, loss of CO2 would required “dumping” the electrons from the C—C=O bond on an
sp3-hybridized carbon (C*). This electron-transfer sequence would form a carbanion, which is a high-
energy intermediate, but not an enol because C* is not adjacent to the C+O unit marked in green.
Because a high-energy carbanion would be formed, the energy demands are too high. For 1,3-
dicarboxylic acids, the product is an enol, which can tautomerize to the carboxylic acid. Therefore,
there is an atom that can accept the electrons with 1,3-dicarboxylic acids, but not with 1,4-dicarboxylic
acids, and decarboxylation of 1,4-dicarboxic acid is very difficult if not impossible.
O

* O
H
O
HO

47. The oxygen atom in B is a stronger base in the reaction with the acidic proton of the COOH unit
when compared to the C=C unit. In addition, decarboxylation of B leads to an enol, which tautomerizes
to the carboxylic acid, whereas decarboxylation of A leads to an alkene. The enol derived from
decarboxylation of B is more stable than the product derived from decarboxylation of A, which makes
the overall process more facile (occurs at a lower temperature).
CO2H CO2H
O

A B

48. The base in this reaction is the oxygen atom that is attached to N, and an intramolecular acid-base
reaction with a β-hydrogen atom must occur via an eclipsed rotamer, as shown. The lowest energy
eclipsed rotamer will be the one in which the base removes the proton from the less substituted β-
carbon, leading to the less substituted alkene.
Me O
O
Me N H
N
CH3
CH3
CH3 H H
+ Me2N-OH
49. The reaction proceed y a syn-elimination mechanism, which is an internal acid-base reaction via the
lowest energy eclipsed rotamer, but the oxygen of an ester is a significantly weaker base than the
hydroxide in used in the Hofmann elimination, or than the negative oxygen in the Cope elimination from
question 48. Therefore, the reaction is much slower and requires much higher reaction temperatures to
overcome the high activation energy for reaction.
O O
H3CO H
OCH3
H3C CH3 H
CH3 H
A 2-Methylhex-1-ene

50.
1. HBr

2. KOH , EtOH

A B

51. The ring is so large that formation of a triple bond proceed without significant problems.

1. Br2 , CCl4
2. excess t-BuOK

t-BuOH , heat

A B

52. The actual conformation of the molecule is shown, and it is clear that the bromine is in an
equatorial position, so it cannot undergo an E2 reaction (no trans axial β-hydrogen). It is also true that
the bicyclic nature of the molecule effectively locks it into the conformation shown, so there is no
chance that the bromine can assume an axial position. Therefore, there is no E2 reaction.

Br
H CH3
CH3 H
KOH , EtOH
CH3 Br
H
CH3
53. Once the alkoxide is formed, the orbitals of the bonds are aligned so that transfer of electrons to
form the carbonyl will break the adjacent bond, and those electrons approach the bromine-bearing
carbon from the back (180° attack) to displace the bromide leaving group and form the new C=C unit.
Br OH
H H
NaH , THF
Br
heat
H
O– O
OH H
Br
H O

Synthesis Problems. Do not attempt until you have read and understood Chapter 25.

54.
1. excess MeI
NH2 2. Ag2O , H2O

3. 200 °C

55. Reagents are provided for each synthesis.


1. HBr
2. KOH , EtOH
(a) E+ Z

1. HBr OH
2. KOH , EtOH
(b)
3. BH3 , ether
4. NaOH , H2O2

1. PBr3
(c) OH
3. KOH , EtOH

Br
1. HBr
2. KOH , EtOH
(d)
3. Br2 , CCl4
Br

Spectroscopic problems. Do not attempt until you have read and understood Chapter 14.

56. 1-Methylcyclohexene
57.
E2

Br Minor Major

58.

IR: about 1650 cm–1


NMR: alkene proton and
C=C-CH3

6 4 2 0
NaOMe PPM

Br THF
IR: no distinguishing
peaks
OCH3 NMR: nothing past 3.6 ppm
and a OCH3 singlet.
4 2 0
PPM

59. 5,5-Dimethylhex-1-ene.

60. Methylcyclopentene.

61.
OH PBr3 Br KOH

EtOH

A B C

62. Hex-2-ene.
Chapter 13
Solutions Manual
Organic Chemistry. An Acid-Base Approach 2nd Edition
14.
O

NH3 Me3COH H2O


Cl O H OH OH

CCl4 CH3CH2OCH2CH3 CH3CH2OH


Cl

15.
O
NH3 DMF H2O
O O
H OH
OH
CCl4 CH3CH2OCH2CH3 CH3CH2OH Cl

16.

(S) (S) (R) (R)


Inversion
product I Br I Br

This is an SN2 reaction, so the reaction proceeds with complete inversion of configuration, and
(2R)-bromobutane is converted to (2S)-iodobutane.

CH3O– H2O CH4 CH3OH HSO4–


17.
Best nucleophile

18. An SN2 reaction of the secondary halide 2-bromobutane has a relatively low activation barrier to the
pentacoordinate transition state. However, 2-bromo-2-methylbutane is a tertiary halide with a large
activation barrier to the sterically hindered pentacoordinate transition state, so the SN2 reaction does not
proceed.

19. Water assists in the ionization of compounds via dipole interaction of both the H of water and the
oxygen of water. Therefore, there is a + and a - polarized atom, and the positive H forms a dipole
interaction with the negative ion or atom, and the negative oxygen forms a dipole with the positive ion
or atom. These dipole interactions facilitate the ionization, and once the ions are formed, these
interactions cause each ion to be effectively "surrounded" with water (solvation), which helps to
stabilize each ion. Therefore, water facilitates ionization and solvates the ions. No other solvent is as
efficient at solvating both cations and anions. Since SN1 and E1 proceed by ionization to a carbocation,
water facilitates these reactions, whereas ionization is very slow in most any other solvent. The other
solvents are simply not as good as solvation and stabilization.

20.

(Least stable) (Most stable)

21. NaOEt H2O CH4 NaNH2 HNO3

22. increased by 2 decreased by 2 no effect increased by 10 decreased by 10

23.

H2O , KI , heat

I
Br
– Br– + I–
1,2-H shift

24.
H3C CH3 H3C OEt
EtOH , reflux
2 weeks
–Br– CH3 –H+
Br
H
1,2-Methyl CH3
H3C CH3 shift HOEt H3C O Et
25.
H
KOH . EtOH HBr
(R) (S)
Br
Br (Z)
One enantiomer Racemic
H A
B
KI , THF
H Br2 , CCl4 Br
KOH . EtOH
(E) (R)
(R)
(R)
(S)
C I D Br
H E

26. The major product or products are shown. No mechanisms are provided.
KI , aq THF
(a) Br
I

NaCN , THF
(b) No reaction (N.R.)
0°C
Br
Ph Ph
cat. H+ , aq THF
(c)

OH
Me OH
Et Et
(R)
(R) I KOH , EtOH
(d) +
(S)
Et I
Me
Et Me
(R) H (R) (R)

(S)
H (S)
I H Et
(R)
H
OH NaH
(e) O
Br THF
Ph Ph
1. PBr3
(f)
2. MeC≡C-Na+

OH C≡CCH3
1. BuLi , THF
2. benzyl bromide
(g) 1-Heptyne
Ph
1. Pred/I2
OH
(h)
2. NaOEt , EtOH

CH3 Br CH3 CN
KCN , DMF
(i)
(S) (R) (S) (S)

H NaNH2 , NH3 (E)


(S)
(j) (S)

I
Br
KOH , EtOH
(k) (Z)

Br
KI , aq THF
(l)

I
OH 1. MeSO Cl CN
2
(m)
2. NaCN , DMF

Br NaOMe OMe
(n)
THF

OH O
(R)
1. NaH , THF
(o)
2. (2S)-bromopentane

(p) 1. HBr Br
2. NaI , THF
(Z)
N.R. with NaI
Br CN
NaCN , DMF
(q)
(S) (R)

Br KI , THF
(r) No reaction (N.R.)
reflux , 1 month
CH3
CH3
H2O , KI , heat
(s) (R)
(R)
(R)
H3C I
H3C Br
Chapter 14
Solutions Manual
Organic Chemistry. An Acid-Base Approach 2nd Edition
38.
(a) M (100) 100% M+1 (101) 6.66% M+2 (102) 0.42% C6H12O, 1 ring or π-bond.
(b) M (149) 100% M+1 (150) 11.46% M+2 (151) 0.62% C10H15N, 4 rings or π-bonds.
(c) M (96) 100% M+1 (97) 7.77% M+2 (98) 0.30% C7H12, 2 rings or π-bonds.
(d) M (96) 100% M+1 (97) 6.66% M+2 (98) 0.42% C6H8O, 3 rings or π-bonds.
(e) M (110) 100% M+1 (111) 8.88% M+2 (112) 0.39% C8H14, 2 rings or π-bonds.
(f) M (83) 100% M+1 (84) 5.91% M+2 (85) 0.15 C5H9N, 2 rings or π-bonds.

39.

CH3Br Chemical formula: CH3Br


Exact mass: 93.94
Molecular weight: 94.94
m/z: 93.94 (100.0%), 95.94 (97.3%), 94.95 (1.1%), 96.94 (1.1%)
Elemental analysis: C, 12.65; H, 3.19; Br, 84.16

94 95 96

40.
Br Chemical formula: C4H7Br2Cl
Exact mass: 247.86 M 248 44%
Molecular weight: 250.36 M+1 249 2%
m/z: 249.86 (100.0%), 251.86 (69.3%), 247.86 (44.1%), M+2 250 100%
253.85 (13.4%), 250.86 (4.4%), 252.86 (3.1%), 248.86 (1.9%) M+4 252 69%
Cl Br
Elemental analysis: C, 19.19; H, 2.82; Br, 63.83; Cl, 14.16 M+6 254 13%

Based on 75% 35Cl + 25% 37Cl + 2 each of 100% 79Br + 98% 81Br
C4+H7 = 55. 248 + mass based on lowest mass isotopes.
Higher mass peaks are 55+35+79+79 = 248
and 55+35+79+81 = 250; 55+35+81+81 = 252; 37+79+79 = 250
37+81+79 = 252; 37+81+81 = 254.
248 250 252 254

41. Butane, C4H10, M:M+1 = 100:4.44


Hexan-2-ol, C6H14O, M:M+1 = 100:6.66
Toluene (methylbenzene), C7H8, M:M+1 = 100:7.77
Triethylamine, C6H15N, M:M+1 = 100:7.03
Butanenitrile, C4H7N, M:M+1 = 10:4.81
42. The ratio of M:M+1 for C100H202 = 100:111

43. An M:M+1 ratio in a mass spectrum was 100:24. If M+1 = 24, then 24/1.11 = 21.6. If there is
experimental error, then this must be rounded to C22. This question is very ambiguous, but it is given
here for a reason. It shows that sometimes there is experimental error that casts doubt on the real
answer. In a laboratory, if possible, always obtain more data to confirm and/or verify a result.

44.
Chemical formula: CHCl3
Exact mass: 117.91
Molecular weight: 119.38
m/z: 117.91 (100.0%), 119.91 (95.9%), 121.91 (30.6%), 123.91 (3.3%), 118.92 (1.1%), 120.91 (1.0%)
Elemental analysis: C, 10.06; H, 0.84; Cl, 89.09

13C2HCl
CHCl3 C2HCl3 13CHCl
3 3

118 120 122 124 118 120 122


118 120 122 124 118 120 122 124
124

45. Based of the exact mass of the daughter ion, identify the fragment lost from the original molecule.
Suggest a cleavage that would lead to that daughter ion.
(a) M = 86.0732 and the daughter ion has a mass of 57.0341. Loss of 29.0391. Loss of ethyl
(b) M = 86.0732 and the daughter ion has a mass of 68.0626. Loss of 18.0106. Loss of water
(c) M = 114.1045 and the daughter ion has a mass of 58.0419. Loss of 56.0626. Loss of butyl
(d) M = 122.0368 and the daughter ion has a mass of 77.0392. Loss of 44.9976. Loss of CO2
46.
A π-electron is most easily removed

p
p

sp2 sp2
An electron is likely removed
from the π-bond.

s s

47.

O O O

M
CH4
O

48. If a radical cation formed from naphthalene has the structure shown, it is possible to delocalize the
charge over the π-bonds, and dispersal of charge density makes the radical cation lower in energy, and
more stable. Therefore, resonance delocalization explains the stability of this radical cation.

49. (a) For C6H12O2, there is 1 ring or π-bond. Since the C=O unit of the ester has a π-bond, this ester is
saturated.
(b) m/z 29 is C2H5, 43 is C3H7, and 71 could be C3H2O2 or C4H6O.

50. Yes! The energy is transferred to the masses (atoms) at each end of the spring (bond), and the more
energy that is applied to the spring, the more the spring (bond) must vibrate to dissipate that energy.
51. ν = (1/2π) (f/µ) = 0.159(f/µ), where µ = (mass1)(mass2) / mass1 + mass2
(a) k = 5.1 x 105 dynes cm-1 for C-H. µ = 12x1/13 = 0.923, so ν = 0.159(5.1x105/0.923) = 87.9 cm–1.
(b) k = 5.9 x 105 dynes cm-1 for C-H. µ = 12x1/13 = 0.923, so ν = 0.159(5.9x105/0.923) = 101.6 cm–1.
(c) k = 7.6 x 105 dynes cm-1 for C-C. µ = 12x12/24 = 6, so ν = 0.159(7.6x105/6) = 20.1 cm–1.
(d) k = 17.5 x 105 dynes cm-1 for C≡N. µ = 12x14/26 = 6.46, so ν = 0.159(17.5x105/26) = 10.7 cm–1.
(e) k = 12.3 x 105 dynes cm-1 for C=O. µ = 12x16/28 = 6.86, so ν = 0.159(12.31x105/6.86) = 28.5 cm–1.

52. Structure B does not have an OH group, and the IR spectrum most certainly does: the broad peak
between 3200–3600 cm–1.

OH O
This structure
is consistent
with the IR. OCH3 H O
A B

53. First of all, there are eight peaks, suggesting eight carbons. Note that there are two peaks at ~ 78
ppm (sorry for the scale). Structures A-C have six or five carbons? The peaks at 78 are due to solvent.
This leaves six carbon atoms, which automatically excludes C. There is a carbonyl peak at ~ 168 ppm,
consistent with an ester carbonyl. The telling peaks are at 130 and 138 ppm, which are clearly due to a
C=C unit. Therefore, structure B fits the NMR.
O
O
O
A B C O
O
O

54. At very dilute concentrations, there is minimal hydrogen-bonding, which leads to sharp absorption
peaks rather than the broad peaks usually associated with OH unit. At very dilute concentrations, each
of the two O—H bonds can be seen.

55. What IR absorptions would be expected for each of the following?


(a) 1-butyne : 2210 + 3300 cm–1 (b) 2-butyn-1-ol : 2210 + 3300-3500 cm–1
(c) 1,2-dichloroethyne: 2210 cm–1 (d) 3-cyanobutanoic acid: 2240 + 2500–3000 + 1730 cm–1

56. (a) Methylenecyclopentane and methylcyclopentene. IR, C=C at 1650 cm–1 1. NMR: 2 alkene H
for methylene cyclopentane vs one alkene H for methylcyclopentene + a singlet methyl
(b) Cyclopentanone and pent-3-enal. IR, 1725 cm–1 for cyclopentanone vs 2817 + 1730 + 1650 cm–1.
NMR: cyclopentane has only methylene groups, but pen-3-enal has the alkene protons at ~ 4.8–5.5 ppm,
a methyl group, and the aldehyde signal at ~ 9–10 ppm.
57. Possible structures are given. Other possibilities must include the given functional group, but there
are many structural possibilities as long no other functional groups are incorporated.

CN

(a) C4H7N

NH2

(b) C8H19N

(c) C5H10O O

58. (345)/x = 60/270, x = (345)(270)/60, x =1552.5 Hz. 1552.5/270x 106 = 5.75 ppm.
(350)/x = 60/270, x = (350)(270)/60, x =1575 Hz. 1575/270x 106 = 5.83 ppm.
(345)/x = 60/600, x = (345)(600)/60, x =3450 Hz. 3450/600x 106 = 5.75 ppm.
(350)/x = 60/600, x = (350)(600)/60, x =3500 Hz. 3500/600x 106 = 5.83 ppm.

59. The quartet at 4.3 pm is linked to the triplet at 1.4 ppm (ratio of 2:3). This is an ethyl group, and the
CH2 unit is either attached to Cl or O. If attached to Cl, the structure is chloroethane, which does not use
up all the atoms, so the structure must be OCH2CH3. The peak in the IR suggests a carbonyl, so it is
likely this is an ester, or COOCH2CH3. The other peak in the NMR is a singlet worth 2H, and it is
downfield at ~ 4 ppm. If we add up all the fragments, only the Cl remains, and if we have ClCH2, the
then structure of the molecule must be ClCH2CO2CH2CH3. The CH2 is connected to both a Cl and a
C=O, so it is further downfield. The downfield shift of the OCH2 is due to the electron withdrawing
effects of the ClCH2 group. The molecule is ethyl chloroacetate.
60. The IR suggests a carbonyl, and there is only one oxygen, so this compound is probably an aldehyde
or ketone. The formula suggests one ring or π-bond, which is accounted for by the C=O unit, so these
are not cyclic compounds and there are no C=C units. A is very symmetrical, accounting for seeing only
two carbon peaks, and the peak at ~ 14 ppm is a methyl, and the other is probably a CH2. The carbonyl
carbon is not apparent (note the peaks at 78 ppm are due to solvent).
It is not uncommon for carbonyl carbons, which normally resonates past 200 ppm, to be weak.
The symmetry in the NMR spectrum suggests this is pentan-3-one. The carbonyl carbon is apparent in
B at ~ 210 ppm, as are the other four carbons. This ketone is unsymmetrical. There are two different
methyl groups and two different methylene carbons, and one is more downfield than the other, so one
methylene is closer to the carbon than the other. Given this information, the likely structure for B is
pentan-2-one.
A = pentan-3-one. B pentan-2-one.

61. A spectrum with a singlet at 1.1 ppm (integrates to 3 H) and a singlet at 3.8 pm (integrates to 1 H)
indicates that the ratio is 1:3, 2:6, or 3:9. The predicted chemical shift for the OMe group in methyl 2,2-
dimethylpropanoate is about 3.5, whereas the methyl signal of the O=C-Me group is about 2.2. For the
given chemical shifts, only 1,1-dimethyethyl ethanoate fits this data.
Methyl 2,2-dimethyl propanoate 1,1-Dimethylethyl ethanoate
O
Singlet at 3.5 (3 H) O
O
Singlet at 0.9 (9 H)
Singlet at O
2.2 (3 H)
Singlet at 0.9 (9 H)

62. This methyl group is attached to a cyclohexane, and the molecule exists in two equilibrating chair
conformations. At low temperatures, below the activation barrier for conversion of one chair to the
other, both chair conformations will exist. At low temperatures, the methyl group in each chair will
have a different environment (one axial and one equatorial) so they will have different chemical shifts.
This difference can be seen in the 1H NMR, so there are two signals. At higher temperatures, the
equilibration is rapid, and the two different methyl signals are effectively in equilibrium, and they
coalesce into one signal (the average of the two separate signals).

CH3
CH3
H3C H
CH3
H H CH3
H
63. A M+2 = 98% of M indicates the presence of bromine.
F Cl Br

OH

64. A peak at m/z 179 relative to a parent ion at m/z = 208 indicates loss of 29 mass units, which
corresponds to an ethyl group. Therefore, all structures that have an ethyl group should be circled.
There is only one.
Cl Cl Cl Cl Cl
Cl Cl Cl
Cl Cl Cl Cl
Cl Cl Cl Cl

65. Any molecule that has a π-bond will exhibit magnetic anisotropy in the 1H NMR spectrum.
H
O
O
CH3

66. The IR data suggests the presence of an aldehyde. Therefore, the two aldehydes circled are
possibilities.
OH
O O O

H H
O

67. A singlet at 5.9 ppm, as well as a triplet at 1.0 ppm and a quartet at 3.5 ppm. Based on chemical
shift, the signal at 5.9 ppm suggests proximity to two functional groups because there are no π-bonds in
the choices. Of these. only the circled compound fits. The circled compound also has the quartet:triplet
characteristic of an ethyl group. The molecule is symmetrical, which is why there appears to be only one
ethyl group. In the other cases, only one has an ethyl group, and that one also has a single methyl that
should appear at ~ 3.5 ppm. The choice can be made solely on chemical shift, but the multiplicity of all
but the circled compound does not fit the given data.
t q s q t q m t
O O O O
O O O OH
t t t s t t s t m
s t s s
68. Peaks at 2850–2960 cm-1 in the IR indicate C—H, so any structure that does NOT have a CH
should be circled. there are two.
Br H3C O O
Br CH3

Br H3C
Br CH3 Cl3C CCl3 H3C CH3

69.
O O
O O
H

A B

(a) How many different signals will appear in the 1H NMR spectrum of A? Two different signals, one
for the CH2 and one for the identical tert-butyl groups.
(b) How many different signals will appear in the 1H NMR spectrum of B? Four different signals, one
for the aldehyde proton, one for the CH group, and the two tert-butyl groups will likely give slightly
different signals, although they will have very similar, if not overlapping signals.
(c) In A, the chemical shift of the CH2 will be ~ 3–3.5 and will integrate for 2 H, whereas the CH in B
will be slightly further downfield (~ 3.2–3.7 ppm) due to the presence of the carbon group, but it will
only integrate to 1 H. The most distinguishing feature is the aldehyde proton in B, which is missing in
A. The two tert-butyl groups in A will show up as one singlet, integrating for 18 H, whereas they may
be two slightly different singlets in B, each integrating for 9 H.

70. Peaks in the 7–8 ppm region of a 1H NMR spectrum indicate the presence of aromatic CH, so no
peaks in that region indicate there are no aromatic CH. The circled compounds have no aromatic CH.
Note that pentan-3-one has no benzene rings at all, so it clearly does not have aromatic CH.
Cl Cl
CH3 Cl Cl Cl

H3C CH3 O
Cl Cl
Cl Cl
71. Both questions are answered by comparison of the anisotropy diagrams shown.
(a) Ethene absorbs at ~ 5.4 ppm in the 1H NMR spectrum whereas the proton for ethyne absorbs ~ 2.3
because the two π-bonds in the alkyne orient the secondary magnetic field such that the protons of
ethyne are in the shielding porting of the anisotropy field, and are upfield. In the others, the protons are
oriented in the deshielding portion of the anisotropy field and are downfield.
(b) The aldehyde H—C=O unit appears at ~ 9.4 ppm because the signal is pushed down field by two
factors: inductive effects of the carbonyl deshield the proton, and the anisotropy of the π-bond also
deshield. The combination of these two effects pushes the aldehyde proton far downfield. In an alcohol,
the H of the alcohol H—C—O appears ~ 3.5 ppm because there is no anisotropy (no π-bonds), but there
is the inductive effect caused by proximity to the electron withdrawing oxygen atom. Since only one
effect is operative, the signal is not as far downfield as noted for the aldehyde.
ethene formaldehyde ethyne

H
H H H C
O
H H H
C
H

Ho Ho
Ho

72. Water will damage salt plates, so water or any solvent that contains water should be avoided.
CCl4 H2O aq. acetone THF hexane

73. A solvent that has protons cannot be used in 1H NMR because those signals will appear and either
“swamp out” signals for the sample, or obscure signals from the sample. Only solvents with no protons
may be used, and this usually means exchanging protons for deuterium in the solvent.
CH3OH D2O CDCl3 CH2Cl2 CCl4

74. 0 1 2 3 4 5 6

H O

2,2,-Dimethylpropan-1-ol

1H 13C 34S 35Cl 81Br 3H


75.
76. Based on this simplistic figure, there are CH, and the peak at ~ 4.4 microns (µ) is consistent with a
triple bond. Since this molecule has a formula of C5H8, it is a hydrocarbon, and the only possible
functional group is an alkyne with a C≡C.
2 3 4 5 6 7 8 9 10
ketone aldehyde alcohol ether

1° amine 2° amine 3° amine nitrile

acid alkyne alkene alkane

77. If we refer to Figure 14.1, the shorter frequency (cm) corresponds to higher energy. The unit in IR is
cm-1, or 1/cm, so 3200 cm–1 (corresponds to 0.0003 cm) is higher in energy than 180 cm-1 (corresponds
to 0.0005 cm).
700 cm-1 1000 cm-1 1800 cm-1 3200 cm-1

78. The molecular ion of C5H9Cl is based on the lowest mass isotopes, for 12C, 1H and 35Cl. This
corresponds to m/z 104 (60 for C, 9 for H and 35 for Cl). Use the isotope masses, not the mass from the
periodic table, which is the mass of all isotopes based on their natural abundance. Atomic Cl is 75:26
mixture of 35Cl and 37Cl, which gives 35.45 for the mass. In MS, use 35, not 35.45.
104 105 106 107

79. The molecular ion for CH3(CH2)5NH2 is m/z 101, and a peak at m/z 84 corresponds to 101-84 or
m/z 17, which is the m/z for ammonia (NH3). By analogy with an alcohol, a primary amine may lose
ammonia.

80. The mass of pentan-3-one is m/z 86, and that of methyl butanoate is m/z 102 due to the extra oxygen
atom. Analysis of the molecular ion of each compound should allow them to be distinguished. In the
IR, both will show a carbonyl at ~ 1725 cm–1 so MS is the best way to distinguish these compounds.

81. The only molecule with protons upfield of the OCH3 signal are in N-ethyl-N-methylethanamine.

Cl Cl
N
N
O
O
CH3
82. Compounds such as this have NO protons at all.
Cl
Cl Cl Br H3C O O
Br CH3

Br H3C
Br CH3 Cl3C CCl3 H3C CH3
Cl Cl
Cl

83. 2,4-Dimethylpentan-3-one.

84. 2,3-Dimethylbutane.

85. Cyclopentenone.

86. 3-Phenylpentan-3-ol.

87. 2,3-Dibromobutane.

88. N,N-Diethylpropanamine.

89. Hex-2-yne.

90. Isobutyronitrile.

91. Allyl vinyl ether.

92. 1-Ethylcyclopentene
Chapter 15
Solutions Manual
Organic Chemistry. An Acid-Base Approach 2nd Edition
28.

MgBr
(a) (b) (c) (d) MgBr
MgI
MgBr Ph

29.

Li
(a) (b) (c) (d) Li
Li
Li Ph

30.

Li
(a) (b) (c) (d) CH3Li
Li
Li Ph

31.
(a) MgBr Et2NH + Et2N–MgBr+

(b) MgBr +
NH2 NH–MgBr+

(c) MgBr NH + N–MgBr+

(d) MgBr OH + O–MgBr+

(e) MgBr +
:–MgBr+
(f) MgBr NH + N–MgBr+

Br
(g) MgBr + MgBr2

MgBr No reaction (perhaps some


(h) I elimination to 2-methylpropene)

32.
Li
(a) HNEt2 Li+-NEt2

Li
(b) NH2 NH- Li+

Li
(c) N-H N- Li+

Li
(d) OH O- Li+

Li
(e)
H : - Li+

Li
(f) NH N- Li+
2 2

Li CH2Ph
(g) PhCH2Br LiBr

Li Cu
(h) ).5 CuI
Li
Li
(i) I
33. Only allylic and benzylic halides give good yields in this reaction. The compound that is circled is
the only allylic bromide among the choices given.

Br Br Br Br

34.
Br Me2CuLi , ether CH3
(a)
-10 °C

Ph Ph
I Me2CuLi , ether CH3
(b)
-10 °C
Ph Ph
I Me2CuLi , ether CH3
(c)
-10 °C

35. Give the names of the final product, if any, for each of the following reactions.
(a) 2-iodopentane + 1. Mg/ether 2. acetylene Pentane + HC≡CMgI
(b) phenylmagnesium bromide + 1. CuBr/THF/-10 °C 2. 2-bromopentane. 2-Phenylpentane
(c) 2-bromobut-2-ene + 1. Li/THF 2. CuI/THF/-10 °C 3. iodomethane. 2-Methylbut-2-ene
(d) 2-bromobut-2-ene + 1. Mg/THF 2. but-1-yne. Butene + EtC≡CMgBr
(e) 3-bromocyclopentene + 1. Mg/THF 2. benzyl bromide. 3-Benzylcyclopentene
(f) butylmagnesium chloride + water. Butane
(g) 2-methylhexylmagnesium bromide + 1,2-dimethoxyethane. No reaction
(h) 1-iodopentane + 1. Li 2. 2-iodo-2-methylpentane. Probably no reaction
since a 3° RLi is less stable than a 1° RLi
(i) phenyllithium + 1. CuI/THF/-10 °C 2. 2-bromohexane. 2-Phenylhexane
(j) n-butyllithium + 1. 1-propyne 2. dilute aqueous acid. Prop-1-yne + butane
(k) n-butyllithium + N-methyl-1-aminopentane. N-Lithio N-methyl-1-aminopentane
+ butane
(l) methyllithium + 2,2,4,4-tetramethylhexane. No reaction

36. Propan-2-ol has an acidic OH unit, and this proton will react as an acid in the presence of the
Grignard reagent, which is a powerful base.

37. Although (2S)-bromohexane is chiral, upon reaction with Mg, the Grignard reagent is
configurationally unstable, so it will revert to a racemic mixture. Reaction with CuBr forms a
magnesium cuprate that reacts with iodoethane to yield the product, 3-methylheptane. This alkane
product is racemic because the Grignard reagent it is derived from is racemic, despite the fact that the
starting bromide was chiral. The point of this problem is to emphasize that a Grignard reagent formed
from a chiral halides will almost always be racemic.
Br MgBr
Mg , Ether 1. CuBr
(S) 2. EtI

38. Vinyl Grignard reagents are less stable and they are more difficult to form because a C=C—X bond
is stronger than a C—C—X bond (sp2 vs sp3 for carbon). THF is a stronger Lewis base when compared
to diethyl ether, and it provides more coordination with the halide which assists in the Mg insertion, and
after formation of the Grignard reagent,, the THF is better Lewis base, and has better coordination with
the Mg of the Grignard reagent, which helps to stabilize that product.

39.
1.NaNH 2
2-1-iodobutane Br
(a)
3. HBr

1. NBS , hν
2. Li
(b)
3. CuI
4. 1-bromopentane

1. HBr 2. Li
(c)
2. CuI 4. MeI

1. SOCl2
(d) OH MgCl
2.Mg , ether

1. BH3 , ether
2. NaOH , H2O2 Ph
(e)
3. PBr3
4. (PhCH2)2CuLi

40. Both are pyrophoric, which means that they react with oxygen in the air and with moisture in the
air. Based on the results, tert-butyllithium is much more reactive than n-butyllithium.

41. The positive head pressure of nitrogen excludes air and moisture from the reaction. Remember that
organolithium reagents react with both water and air. The reaction temperature should be maintained
between -78 → 0 °C, but at lower temperatures the reaction with an amine may be very slow. The
reaction is faster at the higher temperature. Amines are very weak acids, and increasing the temperature
raises the rate of the reaction. Addition of pent-1-yne to the LDA solution should generate the lithium
alkyne anion of pent-1-yne along with diisopropylamine. Subsequent addition of allyl bromide, leads to
an SN2 reaction of the alkyne anion nucleophile with the allyl bromide to yield oct-1-en-4-yne.
42. Octane is formed by reaction of n-butyllithium with unreacted 1-bromobutane (or 1-iodobutane).
The n-butyllithium is prepared by reaction of the halobutane and lithium metal, and as the butyllithium
is formed there is a competition between reaction of halobutane with lithium metal and reaction with the
newly formed n-butyllithium to give octane. This coupling reaction is known as the Wurtz reaction, and
the octane is a hydrocarbon so it is not removed from the reaction mixture.

Spectroscopy Problems. Do not attempt these until after you have studied Chapter 14.

43. Infrared cannot be used to see if bromododecane was converted to dodecane because the C—Br unit
does not have a distinguishing peak that is easily detected. Therefore, there is nothing to check. Proton
NMR may be used because the chemical shift of the CH2Br signal of the bromide will be downfield at ~
3.7 ppm, whereas in the alkane no signal will be downfield of 1.5–1.7 ppm. Therefore, 1H NMR can be
used exclusively as a diagnostic tool to distinguish the bromoalkane from the alkane.

44. Infrared cannot be used alone because the C—Br unit does not have a distinguishing peak that is
easily detected, and the internal alkyne unit of both molecules will show a peak at 2220 cm-1. Proton
NMR may be used because the chemical shift of the CH2Br signal of the bromide will be downfield
relative to the signal in the alkyne with an alkyl substituent. The bromide starting material and the
hydrocarbon product have different masses, so the molecular ion can be used to distinguish the two
molecules. In addition, the bromide will have a M+2 peak in the mass spectrum that is near 100% of the
molecular ion, and the hydrocarbon product will not.
Br CH2CH3
Et2CuLi

45.
Li 0.5 CuI
Br Li CuLi
2
PBr3
HO Br
ether , -10 °C
Chapter 16
Solutions Manual
Organic Chemistry. An Acid-Base Approach 2nd Edition

28.
O
OHC
(a) (b) (c)
O Cl
3-Ethenylcyclopentan-1-one (Z)-9-Chloro-5,5-dimethylnon-3-enal
1-Cyclopropylpentan-1-one

(d) (e) (f)


O
CHO
O
3,4,5-Trimethylcyclohexane-1-carbaldehyde
6-(3,3-Dimethylbutyl)octadecan-3-one
4,5-Dicyclopentylnon-6-yn-2-one

O
Ph
(g) (h)
OHC

6-Phenylhex-5-ynal
2-Cyclohexylcyclohexanone

29. There is less steric hindrance about the carbonyl carbon of an aldehyde than with a ketone, so
butanal should be more reactive in acyl addition reactions when compared with butan-2-one.

30.
O CHO Ph Ph
Ph
(a) (b) (c)
Ph
O
3,3,5,5-Tetramethylcyclohexanone 3-Ethyl-2-propylpent-2-enal 1,1,1,3-Tetraphenylpentan-2-one
Cl

CHO O
(d) (e) (f)
Br
H O
6-Bromo-2,2-dimethylhex-(5Z)-enal 2-Butyl-7-ethyl-5,5-dimethylnon-8-ynal 8-Chloro-3,4-dimethyl-
cyclooct-3-enone
CHO
Cl Cl
O
O
(g) (h) (i)

H
Ph
2,2-Dichlorocyclopentane- 1-Cyclopentyl- 4,5-Dimethyl-3-
1-carbaldehyde ethan-1-one (1-phenylpropyl)hexanal

31. The substituent does not have more carbon atoms than the ring, so it is named as a cyclopropane
derivative, and the name is 1-propylcyclopropanecarbaldehyde.

32.
O O O
(a) (b) (c)
CO2H
CHO CO2H
5-Oxohexanoic acid 6-Oxoheptanal 6-Oxoheptanoic acid

33.
O O O
(a) (b) (c)

(dibutyl ketone) (Methyl ethyl ketone) (Ethyl vinyl ketone)


Nonan-5-one Butan-2-one Pent-1-en-3-one
O
O
(d) (e)

(Dibenzyl ketone) (Phenyl propyl ketone)


1,3-Diphenylpropan-2-one 1-Phenylbutan-1-one

34. Intramolecular hydrogen bonding in 2-hydroxybutanoic acid weakens the O—H bond of the acid,
making it more acidic. The hydroxyl group in 4-hydroxybutanoic acid is too far away for effective
hydrogen bonding. Imagine a five-membered ring being formed for hydrogen bonding in the 2-hydroxy
versus a seven-membered ring for 4-hydroxy.
O O H
O O
H
O H
O
H

O S CH3C≡C:– HS C≡CCH3
35.

36. There is a great deal of steric hindrance around the carbonyl carbon in 3,3,5,5-tetraethylheptan-4-one
that hinders approach of a nucleophile, and makes it less reactive in acyl addition reactions when
compared to the unhindered heptan-4-one.
O
O

37. The tetrahedral intermediate is the same for both reactions. Once formed, ethoxide is a better
leaving group relative to the amide anion. For this reason, the ester is converted to the amide, but the
amide is NOT converted to the ester.
CH3
O O– N
EtO
– CH3 O
CH3 EtO CH3
N N
OEt
CH3 CH3

38.
CO2H
(a) CO2H (b) (c) HO C CO2H
Ph 2

3,3-Dimethylheptanoic acid 1-Phenylcyclohexanecarboxylic acid Hexanedioic acid


(Adipic acid)

CO2H CO2H

(d) (e)

3-Methyl-2-(1,1-dimethylpropyl)heptanoic acid 7,11,12-Triethyl-2,3-dimethylhexadecanoic acid


39.
O O O O
(a) (b) (c)
O O O

Butanoic 2-methylpropanoic anhydride Butyl 2,3-dimethylbutanoate 1-Methylethyl 2-methylpropanoate


(Butyric isobutyric anhydride) (Isopropyl isobutyrate)
O Cl O
O
(d) (e) (f)
N N

N-Ethyl-N,3,3-trimethylbutanamide 2-Butylheptanoyl chloride N, N,3-Trimethylbutanamide


O
NH2 O
(g) O (h) (i)
O O

Cyclohexyl ethanoate 2-Cyclooctylpropanamide Methyl decanoate


(Cyclohexyl acetate)
O O Cl Cl
(j) (k) (l) Cl
O Ph Ph O O
Propanoic anhydride 3,3-Diphenylpentanoyl chloride 4-Chlorobutanoyl chloride
(Propionic anhydride)

40. Iodide is a better leaving group than chloride, because it is a larger atom (weakens the C—X bond)
and the larger iodide ion has greater charge dispersal. If iodide is a better leaving group, then butanoyl
iodide should be much more reactive than butanoyl chloride. It is!

41. The alkyne anion is a better nucleophile. Reaction with the ketone will generate a C—C bond,
which is quite strong, whereas reaction with iodide will generate the weaker C—I bond. The alkoxide
intermediate from reaction with iodide should easily expel iodide because it is a good leaving group,
making the reaction reversible. These combine to make the alkyne anion a much better nucleophile.

42.
Ph
O O O
(a) N (b) (c)
Ph
O O
O
N,N-Diphenylhexanamide Cyclobutyl 3,3-dimethylhexanoate Dipentanoic anhydride
O
O
(d) (e)
O
Cl
Hexadec-(5Z)-enoyl chloride Ethyl oct-4-ynoate
O Ph Ph
O H
(f) (g) (h) N
Cl
N O
H O
N-Chlorobutanamide Butyl butanoate N-Cyclopropyl-4,4-diphenyldodecanamide
Ph
(i) O

O
4-Phenyl-3-cyclohexenyl pentanoate

43. Since n-butyllithium is a good nucleophile, acyl substitution will lead to the ketone. What is not
obvious from this chapter, is that the ketone is more reactive than the ester, and it will compete for
reaction with the butyllithium.
O BuLi O

OMe Bu

44.

O O O
O

45. The product or products are shown. No mechanisms are provided.


OH
CHO 1. HC≡C: –Na+
(a)
2. H3O+
1. KCN
(b)
2. dilute H3O+ HO CN
O
CHO NaCl
(c) No reaction (N.R.)

1. MeLi CH3
(d) O 2. dilute H3O+
OH
O
1. PhMgBr HO Ph
(e) 2. H3O+

O H+ OH
(f) No other products

O 1. MeMgBr OH
(g) CH3
2. H3O+
H H
O
HNEt2
(h) CO2Et
heat NEt2
O
O NaOMe
(i) OMe
Cl
1. BuLi HO Bu
CHO 2. H3O+
(j)

CH3MgBr
(k) + CH4
CO2 –MgBr+
CO2H
H2O
(l) CHO + CH4
CHO MeLi
46. In A, intramolecular hydrogen bonding is possible that enhances the acidity, whereas the carboxyl
groups are too far apart in B for such hydrogen-bonding.
O O

H OH OH
O

OH HO
A H O B

47. The oxocarbenium ion from pentan-3-one has two resonance contributors, whereas the
oxocarbenium ion from methyl pentanoate has three, and is more stable. The more stable (less reactive
= less acidic) oxocarbenium ion is formed from the stronger carbonyl base. Based on the stability of
these conjugate acids, the ester is less basic.
H H
O O

H H H
O O O

O O O

48. Protonation of the carbonyl oxygen is reversible. The oxocarbenium in is a very strong acid, which
means it is very reactive and the equilibrium for this reaction lies to the left, so Ka for this reaction
expected to be << 1.

HO3S
H
–O
O O 3S

49.
O H H H H
O H O O O

SYNTHESIS
There are no synthesis problems associated with this chapter
SPECTROSCOPY. You should read and understand Chapter 14 before attempting these
problems.

O
Pentan-2-one
50.

O
2-Methylpentanal

51. H

H
Hex-2-enal
52. O

O
Hex-3-enoic acid
53. OH

O
N,2,2-trimethylpropanamide
N
H
54.

O
2,2-Dimethylpropanoic acid
OH
55.

O
Hexanoyl chloride
56. Cl

O O

Isobutyric anhydride
O
57.
O

Pent-4-eneoic acid
58. OH
Chapter 17
Solutions Manual
Organic Chemistry. An Acid-Base Approach 2nd Edition
31.
OH NMe
Oxidation NMe
(a) (b) Reduction

OH
Br Reduction Oxidation
(c) (d)
OMe
CHO Br
Oxidation Oxidation
(e) (f)
CHO

32. The chromate ester derived from 3,3-dimethylpentan-2-ol is more sterically hindered, and it is more
difficult to remove the proton from the adjacent carbon. Therefore, the reaction is slower.
H
H

OH OCrO3H
OH OCrO3H
4-Methylpentan-2-ol 3,3-Dimethylpentan-2-ol

33.
(a) 1. CrO3 , aq.HCl , acetone

2. SOCl2
HO 3. EtOH , NEt3 O OEt
1. Hg(OAc)2 , H2O
2. NaBH4
(b) O
3. K2Cr2O7 , aq acetone
H
CH2Cl2
(c)
OH O
-2
N Cr2O7
H 2
1. 9-BBN
O
(d) 2. NaOH , H2O2
3. PCC

(e) CrO3 , aq. HCl , acetone

O
OH
(f) 1. PhMgBr , THF
2. hydrolysis
H Ph
3. PDC

O O

34. The answer to this question is not completely obvious from the discussion in the book. Collins
reagent is formed by mixing chromium trioxide in pyridine, and then adding this mixture to
dichloromethane. Pyridinium chlorochromate is isolated as an orange solid by mixing chromium
trioxide in aq HCl, and isolating the precipitate. It is likely that the Collins regent is more complex since
the equilibrium is different in pyridine than in water, and the HCl reacts to form a chlorochromate. They
are different, but certainly related.

35. The major products are


(a) 4,4-Diphenylhexanoic acid (b) Cyclohexanecarbaldehyde (c) Cycloheptanone.

CrO3 , aq. HCl


36. CrO3Cl –
N
N
H

37. Excess ozone will react with both C=C units, although at least some of the cleavage from only one
C=C is likely. Workup with hydrogen peroxide will generate the carboxylic acids.
1. O3
CO2H HO2C CO2H
HO2C CO2H
2. H2O2 CO2H

38.
Br Br
O
OH O–
a net oxidation
39. Alkyl groups are electron releasing with respect to the π-bond of an alkene. The more substituted
the alkene, the higher the electron density available for donation in the π-bond. The reaction with the
peroxyacid is driven by electron donation from the π-bond to the electrophilic oxygen of the peroxyacid.
Therefore, the more highly substituted alkene is the more reactive, and 3,4-dimethylhex-3-ene
undergoes epoxidation faster than does hex-3-ene.

40.
OH O
N H CrO3Cl –
(a)
CH2Cl2

N H CrO3Cl –
OH
(b) No reaction (N.R.)
CH2Cl2

1. OsO4 , t-BuOH CHO Via diol


(c)
2. HIO4 CHO

1. HBr Steps 1 and 2 yield


(d) 2. KOH , EtOH CHO + O
the same alkene shown
3. O3 , -78 °C
4. Me2S

1. BH3 , ether
(e) 2. NaOH , H2O2 Via 1° alcohol
CHO
3. PCC , CH2Cl2
1.Hg(OAc) 2 , H2O
2. NaBH4; then hydrolysis
(f) O
3. Na2Cr2O7 , aq acetone
H2SO 4

41. The trifluoroacetyl group is electron withdrawing, which makes the electrophilic oxygen of the
peroxyacid more positive via inductive effects. The more positive the oxygen, the more reactive will be
the peroxyacid.

42. The reaction occurs by electron donation from the π-bond to ozone in a 1,3-dipolar reaction. The
more electron rich the π-bond, the faster the reaction is expected to be. Since a tetrasubstituted alkene
has more electron releasing alkyl group, the π-bond is more electron rich, and it reacts faster.

43.

O OH2 OH
O
H OH OH OH
44.

PCC , CH2Cl2 H
H+
H
OH O
O
H

PCC
- H+
H H
OH OH O

45.
OH
CrO3 , H2SO 4
(a)
OH
H2O , acetone O
(b) 1. O3 , -78 °C
+ O
CHO
2. Me2S

dilute KMnO4
OH
(c)
H2O , NaOH
OH

1. KOH , EtOH OHC O


Br 2. OsO4 , t-BuOOH , t-BuOH
(d) 3. Pb(OAc)4 , H2O

1. 9-BBN , ether CHO


(e) 2. NaOH , H2O2
3. pyridinium dichromate
CH2Cl2
1. PBr3
(f) 2. NaOH , EtOH , heat
O
3. peroxyacetic acid
OH

Synthesis. Do not attempt these problems until you have read and understood Chapter 25.
46.
1. MeMgBr
2. H3O+
O 3. PBr3
O O
4. KOH, EtOH
5. O3 , -78 °C 6. Me2S

47.
1. PBr3
2. Me3N
3. Ag2O O
OH
4. 200 °C
5. O3 , -78 °C
6. Me2S

48.
1. 200 °C
2. BH3 , ether CHO
NMe3+ –OH 3. NaOH , H2O2
4. PCC , CH2Cl2

49.
OH
OsO4 , t-BuOOH
(a)
OH
OH 1. PBr3
2. KOH , EtOH CHO
(b)
3. O3 , -78 °C
4. Me2S CHO
1. Hg(OAc)2
MeOH
(c)
2. NaBH4
OMe
1. NaNH2
2. EtI
Br
(d)
3. HBr

Spectroscopic problems. Do not attempt these problems until you have read and understood
Chapter 14.
50.
O O
IR: Cyclohexanone has a C=O at
about 1725 cm-1 but the conjugated
C=O appears at about 1695 cm-1

3 2 1 0 6 4 2 0
PPM PPM

51. In the presence of the mercuric salts, the more stable secondary vinyl carbocation is formed, which
reacts with water to form an oxonium ion. Loss of a proton gives the enol, which tautomerizes to the
product, pentan-2-one. The singlet at 2.1 ppm is due to the methyl group adjacent to the carbonyl.

52. The second product of the epoxidation is trifluoroacetic acid, which is a very strong acid. As the
epoxide is formed, it reacts with the trifluoroacetic acid to generate a carbocation, which can rearrange
to a more stable oxocarbenium ion. Loss of a proton lead to the observed ketone. By adding a buffer to
the reaction, (e.g., sodium acetate), the trifluoroacetic acid product is buffered, which means that acid-
catalyzed processes are minimized.

F3CCO3H O
+ F3CCO2H
O

3,4-Diethylhex-3-ene 4,4-Diethylhexan-3-one

HO
O O OH
H
53. In the IR, the alcohol will have the broad peak at ~ 3300 cm-1, and this peak will disappear as it is
oxidized to the conjugated ketone, which will have a carbonyl peak at ~ 1695 cm-1. The alcohol will
have a C=C peak at ~ 1650 cm-1. In the 1H NMR, the ketone will have a single methyl signal at ~ 2.3
ppm. The methyl group adjacent to the alcohol-bearing carbon will be a doublet. Both of the molecules
will have alkene protons and an ethyl group.
OH
PDC O

Hex-3-en-2-ol

6 4 2 0 8 6 4 2 0
PPM PPM

54. In the IR, both will have a carbonyl peak at ~ 1725 cm-1, but the aldehyde will also have the
aldehyde proton signal at 2817 cm-1. In the 1H NMR, the aldehyde will have the signal at 9-10 ppm,
which is missing from the ketone. The ketone will have a singlet methyl signal at ~ 2.2 ppm that is
missing from the aldehyde.
O

H 3-Phenyl-1-pentanal
3-Phenylpentan-2-one

10 5 0 8 6 4 2 0
PPM PPM

55. In the IR, the aldehyde will have the aldehyde proton peak at ~ 2817 cm-1 and the carbonyl peak at
1725 cm–1. The carboxylic acid will have the carbonyl at ~ 1725 cm-1, but it will also have a broad
absorption at 2500–3000 cm-1. In the 1H NMR, the most obvious difference is the singlet at 9-10 ppm
for the aldehyde, but at ~ 12 ppm for the acid.
A simple alternative is to wash the mixture with an aqueous solution of NaHCO3. The
carboxylic acid will react in an acid–base reaction to give the carboxylate anion, which is water soluble.
The aldehyde can be separated and the acid regenerated by treating the aqueous solution with dilute
HCl.
H O
OH
O OH

10 5 0 10 5 0
PPM PPM

56. The signal for oxirane is shifted upfield because of the strain of the three-membered ring. The
strain causes the bonds to “bow” from linearity between the nuclei, which diminishes the effects due to
the proximity of the electron pairs on oxygen relative to the hydrogen atoms. The result is greater
shielding, and the signal moves upfield relive to methyl ether.

O O

4 2 0 3 2 1 0
PPM PPM

57. The aldehyde unit has the polarized C=O bond, and the electron withdrawing effect of the oxygen
leads to greater deshielding relative to the nonpolarized alkene, C=C. In effect, the bond polarization
and anisotropy effects are additive.

58. There will be few differences in the IR, but in the 1H NMR, butane-2,3-diol will show a doublet for
the methyl groups, whereas butane-1,4-diol has no methyl groups, only methylene.
OH

Butane-2,3-diol OH Butane-1,4-diol
HO
OH

4 2 0 4 2 0
PPM PPM

H
3,5,5-Trimethylhexanal
59. O
OH
OH
2-Methylpentane-2,4-diol
60.

O
1,2-Epoxyhexane
61.

O OH
2-Propylpentanoic acid

62.

Pent-2-enal (E or Z is OK)
63. O

O
HO 2,3-Dimethylsuccinic acid
OH

64. O
Chapter 18
Solutions Manual
Organic Chemistry. An Acid-Base Approach 2nd Edition

56. Both are pentane-2-one, which will react to yield the product.
O O

O O
Hexan-2-one Pentan-2-one Pentan-3-one Pentan-2-one

57. Grignard reagents are strong bases that will react with the acidic hydrogen of the OH unit in ethanol,
which is a strong acid in the presence of this powerful base.

58. Although (2S)-bromohexane is a chiral molecule, upon reaction with Mg, the Grignard reagent is
configurationally unstable at the carbon bearing the Mg, so the product will be a racemic mixture.
Reaction with pentan-2-one will give the alcohol, but since the Grignard reagent is racemic, the alcohol
is racemic and a mixture of diastereomers.
Br MgBr
Mg , ether 1. pentan-2-one OH
(S) 2. H3O+

59. Vinyl Grignard reagents are less stable and they are more difficult to form because a C=C—X bond
is stronger than a C—C—X bond (sp2 vs sp3 for carbon). The solvent THF is a stronger Lewis base
when compared to diethyl ether as the solvent, because it provides more coordination with the halide.
This increased basicity assists in the Mg insertion, and after formation of the Grignard reagent, the THF
is a better Lewis base, with better coordination of the Mg of the Grignard reagent, which helps to
stabilize that product.

60. The ketal from this reaction would be a nine-membered ring. The transannular strain in nine-
membered rings is so great that a cyclization reaction to generate such a ring will have such a high
activation barrier that it simply does not form.
61. Note that the C=C—N unit is essentially planar in an enamine. In B, the methyl group on the C=C
unit will have steric interaction with the isopropyl groups on nitrogen, which will hinder its formation.
In A, that particular methyl group can rotate away so there is less steric hindrance and this enamine will
form faster than B.

N N

A Steric B
hindrance

62. React each with 2,4-dinitrophenylhydrazine. The two derivatives will have different melting points,
and by looking up the melting points for these two derivatives in the literature, the two are easily
distinguished. A bit more research would show that the conjugated ketone will give an orange-red 2,4-
dintirophenylhydrazone whereas the nonconjugated ketone will yield a yellowish 2,4-
dinitrophenylhydrazone. The two compounds are easily distinguished using this method.
O2N
NO2
O H
O2N NHNH2 N NO2
N
Orange-red

O2N
NO2
O
HN NO2
O2N NHNH2
N

Yellow

O O– OSiMe3
KCN ClSiMe3
63. CN CN
64.
H Bu
H Bu N
N-Bu H+ OH2
N

OH2
H H
Bu H Bu – H NBu
–H+ H
N H+ N 2
O

OH OH

–H+ O

65. The enol form is stabilized by internal hydrogen bonding, as shown. Such stabilization is not
possible with simple mono-ketones.
H
O O
O O

66. The carbonyl is too sterically hindered and the nucleophilic carbon of the Grignard reagent cannot
come close enough for bonding. A slower reaction involves coordination of the carbonyl oxygen with
the magnesium, formation of ethene and transfer of hydride to the acyl carbon, leading to reduction of
the ketone.
If acyl addition cannot occur, coordination with the Mg—O
BrMg leads to transfer of a hydride to the acyl carbon, loss of ethene,
and reduction of the ketone
H
O BrMg+-O H
H2C=CH2

Steric hindrance prevents close approach


of the Grignard reagent to the acyl carbon

67.
Me2HC
Me2CHMgCl
OH
O
(a)
EtLi
OH
Et
HO CHMe2
Me2CHMgCl
O
(b)
HO Et
EtLi

CMe3 OH
CMe3 O Me2CHMgCl

(c) CHMe2
H CMe3 OH
EtLi

Et

Me2CHMgCl OH
O CHMe2
(d)
EtLi OH
Et
OH
O Me2CHMgCl
CHMe2
(e) OH
EtLi
Et

Me2CHMgCl

(f)
CHO Me2HC OH

EtLi

Et OH
Me2CHMgCl

(g) CHMe2
EtLi HO Ph
O Ph Et
HO Ph

68. Both have a π-bond and both will react as a Lewis base or a Brønsted–Lowry base. However, The
C=O unit is polarized and the C=C unit is not, which allows the carbonyl to undergo acyl addition in
which a nucleophile donates electrons to the electron-deficient carbonyl carbon, breaking the weak π-
bond and forming an alkoxide product.
69. In aqueous solutions 1,1,1-trichloroethanal forms the hydrate (chloral hydrate), which is quite stable
and there is a sizable concentration in aqueous media. If the carbonyl carbon is “tied up” as a hydrate,
acyl addition reactions do not occur.

70. Sulfuric acid is rather insoluble in organic solvents, whereas methanesulfonic acid is mostly soluble
in organic solvents. In addition, sulfuric acid is an oxidizing acid that may initiate unwanted side-
reactions or even decomposition of the starting material.

71.
Hemiketals Ketals

HO H3CH2CH2CO
O
(a)
H3CH2CH2CO H3CH2CH2CO
O OH OEt
(b)
OEt OEt

(c)
O MeO OMe
MeO OH

OH
O O
O
(d) O

72.
NaCN O–Na+ NaOEt O–Na+
(a) O (b) O
CN OEt
NaC≡CEt O–Na+ MeMgBr O–MgBr+
(c) O (d) O
C≡CEt Me
MeLi O–Li+ EtNH2 O–
(e) O (f) O
N H
Me
H
73. Give the complete mechanism for each of the following transformations.
OH OH
(a) HO OH -H+;+H + + H+
H H
O + H+ O O C3H7 CH3 C3H7 CH-H
O 3 +
H O
C3H7 CH3 C3H7 CH3 C3H7 CH3 HOR
HO
HO
H H C3H7 CH3 C3H7
O - H2O C3H7 CH3 H C3H7 CH3 - H+ O
O OH O
O + H+ CH3
C3H7 CH3 OH O O
O
HO
H OH
O H H N OH
(b) + H+ O O - H+
H C3H7 Me
C3H7 Me N
C3H7 Me C3H7 Me H CHMe2
H
H H
OH C3H7 Me - H+ C3H7 Me
+ H+ O - H2O

C3H7 Me N • •N
N C3H7 Me OH
H •• OH N H OH
H •• OH
H
N
H H OH
(c) O + H+ O O - H+

N
BASE H

H H
OH+ H+ H - H2O - H+ H
O H
N N N
•• N ••
••
Et Et H H H
O + H+ O - EtOH H O
(d) C3H7
+H2O
C3H7 H
C3H7 H- H+ C3H7 H + EtOH O
O O Et O C3H7 H
Et Et O
Et Et
-H+ OH OH H H
+H+ O O - H+ O
-HOEt
C3H7 H
+ H+ C3H7 H
-H+ O C3H7 H
O C3H7 H C3H7 H
Et Et H

74. Phenylglyoxal (A) forms a relatively stable hydrate because of the stabilization provided by the
adjacent electron withdrawing carbonyl group. No hydrogen atom is on the carbon adjacent to the
carbonyl carbon, so there is no mechanism that will allow loss of water. Therefore, A cannot easily form
an enol. In addition, formation of the carbocation shown is destabilized (a C+ next to a Cδ+ would
constitute a high energy intermediate. Formation of the hydrate from B is straightforward, and loss of
water to the enol and tautomerization back to the aldehyde is facile, so this hydrate is not very stable.
O O O
OH OH
CHO
A OH

OH OH
CHO
B OH
75. From a synthetic viewpoint, the ketal is formed from the ketone shown. Note that any synthesis
must account for the absolute stereochemistry of the groups in the acyclic ketone in order to generate the
ketal with the correct stereochemistry. Note also how the (R)/(S) configuration changes when the ring
system is opened to the cyclic ketone since there are now two OH units. The molecule is drawn in the
same perspective as B to show the stereochemical relationships, and then again in the extended
conformation.

H HO HPhH2CO OCH2PhO

B O HO
H O
O H
OH OH
PhH2CO H OCH2Ph PhH2CO H OCH2Ph

76. Formation of the ketal from propane-1,3-diol requires formation of a stable six-membered ring,
whereas formation of the ketal from octane-1,8-diol requires formation of a high energy nine-membered
ring. The transannular strain inherent to a nine-membered ring poses an energy barrier that is too high
for formation of this ketal.
O O

O O

77.

OCH2CH3 cat. TsOH


(a) O + 2 EtOH
H2O/THF
OCH2CH3
H3CO OCH3 cat. TsOH O
(b) + 2 MeOH
H2O/THF

O O
cat. TsOH
(c) + HOCH2CH2OH
H2O/THF
O
OCH2CH2CH3 cat. TsOH
(d) CHO
H2O/THF
OCH2CH2CH3 + 2 CH3CH2CH2OH
S SH
BF3 , H2O
(e) Ph PhCHO
THF
S SH
O O OH
Ph cat. TsOH
(f)
Ph H2O/THF
O Ph Ph OH
SCH3 BF , H O
(g)
3 2
O + 2 CH3SH
THF
SCH3

78.
H+ MeOH -H+
H OH
OH
O O
O H OMe
(a) Me
H+ H
OH2 -H+
-H+
O Me OMe
OMe
OMe OMe

Ph H+ Ph Ph Ph
H H2O OH -H+
OH
(b) O O
H H O H H OH
H
H

Et Et H
O + H+
H H
O - EtOH Me O
C5H11 +H2O
(c) C5H11 Me
C5H11 Me- H+ C5H11 Me + EtOH O
O O C5H11 Me
O Et O
Et Et Et
Et
-H+ OH OH H H

• •
+H+

• •
-HOEt O O - H+ O
+ H+ C5H11 Me C5H11 Me
O -H+ O C5H11 MeC5H11 Me C5H11 Me
Et Et H
OH OH
HO OH -H+;+H + + H+
O H H
(d) + H+ O O Bu H
O Bu H -H+
H O
Bu H
Bu H Bu H HOR
HO
HO
H Bu
H H Bu Bu O
O - H2O H H - H+
C3H7 H
O OH O H
O + H+ O
Bu H OH O
O
HO
OH OH
-H+;+H + + H+
O H H HS SH
(e) + H+ O O
S -H+
H S
HSR
HS
HS
H H H - H+
O - H2O S
S SH S
S + H+
S S
S SH
HS

79.

O Ph
O S
O Ph
O S
Ph
4-Phenyl-1,1-dimethoxy- 2,2-Dimethyl-4,4-diphenyl- 1,1-Diethylthiopentane
cyclohexane 1,3-dioxane

O O

O O

2,2-(Diisopropoxy)-3,4-dimethylhexane 2-Butyl-1,3-dioxolane

80.
H+ EtOH -H+
H SH SH
S
S
O H OEt
H Et
H+ -H2S EtOH
SH2 -H+
O Et OEt
OEt OEt OEt OEt
81. Although it may not obvious, the mixed OH-SH thiohydrate is known to be very unstable, and it
will undergo elimination and revert back to the ketone, just like a hydrate. Given this instability, it is
very unlikely that any of the thiohydrate product (HS-C-SH) will ever be formed under these conditions.

H+ H2S –H+
H OH
OH
O O
H
H3C H3C S SH
H3C H3C
H

OH2 SH2 SH

H3C SH SH H3C SH
H3C SH H3C

82. Yes. Dilute acid hydrolysis will convert the dioxolane to a ketone, and it should be possible to treat
A with BF3 under conditions that will convert the dithiolane to a ketone. In each case, the other
protected carbonyl will not be hydrolyzed.
O O O

O O
TsOH
H2O/THF BF3, H2O-THF

S S
O
S S
A

83. If the ether reacts with an electrophilic atom, the product is an oxonium salt. If an ether reacts with a
carbonyl, a trialkyl oxonium salt would be formed, that does not have a proton. The oxonium ion is
highly reactive, but formation of this ion is reversible and cannot proceed to product. In other words,
the oxygen of an ether is a remarkably poor nucleophile for acyl addition reactions.

84. The major product or products are shown. No mechanisms are provided.

(a) CHO NH2OH


CH=NOH
cat H+

(b) pyrrolidine , cat H+ N


O
O NH2
(c) N
cat H+

(d) PhNHNH2 , cat H+


O N-NHPh

CHO HO OH O
(e)
cat H+
O
Ph Ph Ph Ph
OH Ph
(f) Ph
cat. H+ BuO OBu
O
O NH
(g) O N O
cat H+

O
O O H2O + HOCH2CH2OH
(h)
cat H+ NO2
H
O 2,4-dinitrophenylhydrazine N N
(i)
cat H+

Ph NO2
1. MeCO3H
(j) Via epoxide
2. PhMgBr
3. H3O+ OH
85. Acid hydrolysis will lead to the sugar shown, and benzaldehyde. Along the way, cyanide is released
as a leaving group. Presumably, laetrile kills cells by exposing them to deadly cyanide, but it is unlikely
there is any selectivity for cancerous cells versus normal cells.
COOH COOH H
O O CN H+ O CN
HO HO O
HO HO HO HO
Ph Ph
A
CN CN CN
H2O -H+ HO
COOH
Ph H2O Ph
O OH Ph
HO
HO HO
H -H+ H
– –CN O O
H
Ph Ph

86.

OH OH OCH3 OH
(a) (b) (c) is possible
CN Ph Ph CN
CN CN

87. Compound B is not as susceptible to hydrolysis. For all practical purposes it is just an ether, and
acid hydrolysis of an ether is very difficult unless HI or HBr is used. Compound A is an acetal, and
subject to the hydrolysis mechanism discussed in this chapter.

88. The logical product is the enamine, formed by the usual mechanism. An alternative product arises
by interaction of the pendant hydroxyl group with the iminium salt (or another intermediate) to form the
cyclic N,O-ketal shown.

O O H OH OH OH
A N
N OH OH N OH2
H

N N N N
OH OH OH O
89. Hydrolysis of trioxolane eventually leads to the hydrate of formaldehyde, which is converted to
formaldehyde. All three C—O units of trioxolanes are converted to 3 equivalents of formaldehyde upon
acid hydrolysis.
H H H
O O H+ O O O O O O
H2
O CH 2 C
O O O OH2
A
H H
O O O O H2 - H2O O
H2 C formaldehyde
HO OH
O C H H
OH
H

90. (a) 3,4-Dimethylheptan-3-ol. (b) Phenylcyclohexanol (c) CH3CH2C≡C-+MgI + CH3—H.


(d) Dicyclopentylmethanol. (e) Butane. (f) 5,7-Dimethylundecan-5-ol.
(g) 2-Methylheptan-2-ol. (h) 4-Ethyl-1,1-diphenylheptan-1-ol. (i) Prop-1-yne.
(j) 1-Phenylhexan-1-ol. (k) 4-Phenylbutan-2-ol.

91. This multi-step a reaction is one in which each step is reversible. In an equilibrium, removing one
product from the reaction shifts the equilibrium towards product. If two products are formed and one is
removed, the equilibrium shifts to produce more of both products. Using Le Chatlier’s principle,
removal of water, which is a product along with the acetal or ketal, will shift the equilibrium and drive
the reaction towards the acetal or ketal.

92. The product or products are shown. No mechanisms are provided.


1. Mg , ether
2. pentan-3-one OH
(a) Br 3. H3O+

2 BuSH , cat BF3 SBu


(b) O
SBu
1. PCC , CH2Cl2
2. CH3CH2MgBr , ether
OH
(c) OH 3. H3O+
OH O
CrO3 , aq acetone
(d) H2SO 4

Br
1. Mg , ether
(e) 2. H2O , acetone

OH
CHO
1. CH3C≡C –Na+ , THF
(f) C≡CCH3
2. H3 O+

O HO
aq H+ , heat
(g) O
O HO
N
butan-2-one , cat H+
(h) CH3CH2NH2

MgI
1.
(i) O
2. aq H+ OH
1. OsO4 , t-BuOH
(j) 2. HIO4 CH=NNHPh
Via 1,2-diol
3. excess PhNHNH2 , H+ CH=NNHPh

Synthesis. Do not attempt these problems until you have read and understood Chapter 25.

93.
O 1. PhMgBr , THF Ph OH
(a)
2. H3O+

1. Mg, THF OH
2. cyclohexanone
(b) Br
3. H3O+

1. NaNH2 , NH3
2. 2-hexanone
(c)
3. H3O+
OH
1. Mg , THF
(d) 2. cyclopentanone
3. H3O+
I HO
94.
1. Hg(OAc)2 , HO
2. NaBH4
(a)
3. PCC
4. Me2CHCH2MgBr; H3O+ OH
1. PCC CH3
2. MeMgBr N
(b)
3. PCC
OH
4. MeNH2 , H+

1. O3 , -78 °C SEt
2. Me2S
(c)
3. excess EtSH , cat H+
SEt

1. PCC
2. MeMgBr
3. PBr3 NNHPh NNHPh
(d)
4. KOH , EtOH
OH
5. MeCO3H
6. MeMgBr
7. HIO4 8. excess PhNHNH2 , H+
1. Me2CHCH2MgBr
2. PCC
(e) CHO O O
3. ethylene glycol
cat H+

1. BuMgBr
2. PCC
(f) CHO
OH
3. PhMgBr

95.
1. Hg(OAc)2 , H2O
2. NaBH4 NMe2

3. PCC
4. Me2NH , cat H+

Spectroscopy Problems. Do not attempt these problems until you have read and understood
Chapter 14.

96. Differences between an imine and the alkene unit of an enamine are minimal. In the 1H NMR, an
enamine will have an alkene H between 4.5-5.5 ppm, whereas an imine will not. Two groups will be on
nitrogen in an enamine, and only one on the nitrogen in an imine. Depending of the nature of the groups
on nitrogen, there may be other differences in the 1H NMR.
97. There is a distinct ethyl group in the 1H NMR, and integration will reveal that there are two ethyl
groups. The collection of methylene groups at about 1.8-2.1 ppm suggests a ring, but there is no way to
absolutely correlate these peaks with a four-membered ring. The mass spectrum will show a weak
molecular ion, and a prominent M-18 peak, in accord with a tertiary alcohol. There will be a prominent
M-29 peak for loss of an ethyl group. There are characteristics of fragmentation for four-membered
rings, but they have not been discussed in this book, so the only reasonable thing to be gained from the
mass spectrum is the presence of ethyl groups and the suggestion of an alcohol.

OH

4 3 2 1 0
PPM

O
98.

C8H14O.

99. There are two rings or π-bonds, and one of them is a carbonyl. The position of the carbonyl
suggests a conjugated carbonyl. The absence of a peak at ~ 2800 cm-1 indicates the compound is not an
aldehyde, so it is a conjugated ketone. The absence of a peak at 2270 cm-1 indicates that it is not an
alkyne, which is consistent with the number of π-bonds, so it is indeed an acyclic conjugated ketone.
There is nothing to indicate the specific structure of the seven-carbon conjugated ketone. Two examples
are shown, but there are other possible isomers.
O
O
100. The conjugated ketone (cyclopent-2-enone) will show a carbonyl at ~ 1695 cm–1 whereas the non-
conjugated ketone (cyclopent-3-enone) will show a normal carbonyl at ~ 1725 cm–1. The 1H NMR of
cyclopent-3-enone is rather simple, showing only one type of alkene proton and the methylene protons
adjacent to the carbonyl. Cyclopent-2-enone shows several peaks, including two different alkenyl
protons and two different methylene groups, one further downfield than the other as it is next to the
carbonyl. Reaction of each of the ketones with 2,4-dinitriophenylhydrazine will give the 2,4-
dinitrophenylhydrazone product. The product derived from cyclopent3-one will be yellow-orange or
just yellow, whereas the product derived from cclopent-2-enone will be red-orange due to the
conjugation.
O O

6 4 2 0 6 4 2 0
PPM PPM

101. 1-Phenylbutan-2-one

102. Benzaldehyde diethyl acetal

4-Phenylbut-3-en-2-one

103.

Diisopropylamine
N
104. H

OH

1-Hexyn-3-ol
105.
106. 4-Methylpent-1-ene to 4-methylpentan-1-ol to 3,7-dimethyloctan-3-ol
OH Br
1. 9-BBN PBr3

2. NaOH
A B C
H2O2

MgBr O
Mg

D E OH

1-Phenylbutan-1-ol

107. OH
Chapter 19
Solutions Manual
Organic Chemistry. An Acid-Base Approach 2nd Edition
31.
CO2Me CH2OH C≡N CH2NH2
(a) (b)

OH OH
Reduction Reduction
Br O
(c) (d)
Reduction Oxidation

CHO CO2H O
(e) (f)
Oxidation
Oxidation

32. The OMe groups attached to Al in LiAlH(OMe)3 are electron withdrawing, so the Al—H bond is
less polarized (less δ-), and LiAlH(OMe)3 is weaker than LiAlH4. The electron releasing NaBHEt3
makes the B—H bond more polarized (more δ-), so it is a stronger reducing agent than NaBH4.

33. The Al—H bond in LiAlH4 is more polarized than the B—H bond in sodium borohydride, so Al—H
is more reactive with the acidic proton of water. Lithium aluminum hydride will react with water in an
acid-base reaction, which emphasizes the δ- polarization of the H (a hydride).

34. The product is the sodium salt of butanoic acid, CH3CH2CH2CO2-Na+. Sodium borohydride is not
a strong enough reducing agent to reduce the carboxylic acid.

35.
O O O O

O O O O
36. An amalgam is most commonly an alloy of mercury with another metals, but it has sometimes been
more broadly defined as a mixture of different elements.

37. Reduction of 5-oxooctanal with NaBH4 reduces the aldehyde first because it is less hindered and
more reactive. Excess reagent will reduce both the aldehyde and the ketone units, but if the
stoichiometry and reaction time are carefully controlled, it is possible to selectively reduce the aldehyde
in the presence of the ketone. Note that it is often difficult to do this with NaBH4.
CHO CH2OH
NaBH4

O O

38. Lithium triethylborohydride is LiBHEt3 and lithium trimethoxyaluminum hydride is LiAlH(OMe)3.


As noted in question 30, the OMe groups attached to Al in LiAlH(OMe)3 are electron withdrawing,
which makes The Al—H bond less polarized (less δ-), so it is weaker than LiAlH4. The electron
releasing NaBHEt3 makes the B-H bond more polarized (more δ-), so it is stronger than NaBH4.

39. The major product or products are shown. No mechanisms are provided.
O OH
1. NaBH4 , EtOH
(a)
2. aq NH4Cl

1. KI , refluxing acetone
(b) Br 2. KOH , EtOH
3. H2 , Pd-C
(c) 1. PCC , CH2Cl2
2. NH2NH2 , KOH
OH
– +
Br 1. MeC≡C: Na , THF
(d) 2. H2 , Pd-BaSO4
quinoline
1. PBr3
2. NaCN , THF CH2NH2
OH
(e) 3. 2 H2 , Ni(R) , EtOH

1.LiAlH 4 , ether
(f)
2. aq. NH4Cl

O OH
1. LiAlH4 , ether
(g) CO2Et 2. aq. H+ CH2OH + EtOH

Na , NH3
(h) EtOH

Na , NH3
(i) O OH
EtOH
O O
H2 , Pd-C
(j) H
Cl
H2 , PtO2
(k) O OH
MeOH

excess H2
(l) CH3 CH3
Ni(R) , heat , EtOH

40. Hydrogenation requires that the C=C unit of the alkene coordinate to the surface of the transition
metal catalyst. While cyclopentene readily coordinates to palladium and is easily hydrogenated 1,2-di-
tert-butylcyclopentene has a sterically hindered C=C unit that inhibits approach to and coordination with
the Pd, so the hydrogenation is much slower.

41. Hexan-2-ol

42.

LiAlH4
H H
O H3Al O H3Al O

43. The electron is transferred to the benzene ring at a carbon that generates a radical anion. The most
stable radical anion will NOT have the negative charge on the carbon bearing the OMe group in any of
the resonance contributors. The resonance contributors formed are those formed in this reaction, and the
product is the diene shown, with the OMe group of a C=C unit.
OMe OMe OMe OMe OMe
44. Triphenylmethane (Ph3C—H).

45. Hydrogenation is a surface reaction in that the hydrogen must coordinate with the surface of the
metal in order to break the H—H bond to form hydrogen radicals coordinated to the metal. Likewise,
the C=C unit must coordinate to the surface of the metal. The larger the surface of the metal, the more
coordination, and the faster will be the rate of the reaction.

46. Hydride reducing agents do not reduce nonconjugated alkenes. Therefore, if hex-5-enal is reduced
with NaBH4, the product will be hex-5-en-1-ol. Hydrogenation using a Pd catalyst has a greater affinity
for reduction of alkenes relative to carbonyl compounds. Therefore, treatment of hex-5-enal with one
equivalent of H2 and Pd-C catalyst, usually in ethanol or methanol, should give hexanal as the major
product, although some reduction of the aldehyde will undoubtedly occur to give hexan-1-ol. Reduction
of hex-5-enal to hexan-1-ol is easily accomplished using a large excess of hydrogen gas with a Pd or Pt
catalyst.

47.
O 1. NaBH4 OH
A
2. aq. NH4Cl B
CO2Et CO2Et

48. The major product or products are shown. No mechanisms are provided.

1. LiAlH4 , ether
(a) +
OH
CHO 2. H3O
O OH
1.NaBH 4 , EtOH
(b)
2. aq. NH4Cl

CHO CH2OH
(c) Na(0) , NH3
EtOH
Et
Et
(d) Na(0) , NH3
EtOH
OH Et OMe
O Et OMe
O
H2 , PtO2
(e) MeOH OH
H
H2 , PtO2
(f) Mixture of
MeOH
O OH diastereomers

H2 , PtO2
(g)
MeOH

O OH
O OH
OH excess H2 OH
(h)
PtO2 , MeOH

CHO CH2OH
(i) excess H2
Mixture of
O PtO2 , MeOH OH diastereomers

O 1. LiAlH4 , ether OH
(j)
2. H3O+

O 1. LiAlH4 , ether OH
(k) 2. H3O+
H
CO2Et 1. LiAlH4 , ether CH2OH
(l)
2. H3O+

O
OH
1. NaBH4 , EtOH
(m)
2. aq NH4Cl

O OH O OH
1.NaBH 4 , EtOH
(n) 2. aq NH4Cl

49. Give the product of each individual step where appropriate, and the final product for each of the
following:
(a) Hex-(2E)-ene. (b) Lithium salt of pentan-2-ol (lithium 2-pentoxide).
(c) Hept-(3Z)-ene. (d) Butan-1-ol and then butanal.
(e) 2-Phenylpentane. (f) Ethylcyclopentanol and then 1-bromo-1-ethylcyclopentane and then the
Grignard reagent and then ethylcyclopentane.
50. In this reaction the ammonia functions as an acid in the presence of the carbanion intermediate
(R3C– + H—NH2 → R3C—H + –NH2). Ethanol is a much stronger acid when compared to ammonia,
so ethanol is added to accelerate the protonation of the carbocation intermediate.

51. Transfer of an electron generates a ketyl, and it is possible for two of the ketyls to undergo a
coupling reaction via a radical coupling reaction to give the dimer shown. Hydrolysis leads to a vicinal
diol. This coupling reaction is known as the pinacol reaction or just pinacol coupling.
O OH
O H3O+

O O

O OH

52.
(a) Methanol + 4-phenylbutan-1-ol. (b) 3-Methylhexan-2-ol.
(c) 2-Methylpentanal. (d) Cyclopentylmethanol.
(e) 2-Methylpentane. (f) 3-Phenylpentane.

Synthesis. Do not attempt these problems until Chapter 25 is read and understood.

53. 1. Ozonolysis followed by treatment with dimethyl sulfide. 2. Dihydroxylation followed by


treatment with periodic acid. 3. Formation of an alcohol using oxymercuration or hydroboration,
followed by oxidation with PCC or PDC. 4. Epoxidation with peroxyacetic acid followed by reduction
with LiAlH4 and then oxidation with PCC or PDC.

54. Provide a synthesis for each of the following:

1. Br2 , CCl4
2. excess KOH , EtOH
(a)
3. NaNH2
4. EtI
5. Na, NH3 , EtOH

1. HBr
2. Mg , ether
(b)
3. CH3CH2CHO
OH
4. H3O+
1. HBr
2. Li
(c)
3. CuI , ether
4. EtI
OH NH2
1. SOCl2 , NEt3
(d)
2. NaCN , DMF
3. LiAlH4
4. H3O+

1. Sn , HCl Br
Cl 2. Me2CHMgBr 3. aq NH4Cl
(e)
4. PBr3
O
1. LiAlH4
2. H2O 3. PBr3
(f) CO2Et
4. HC≡C:-Na+
5. NaNH2 6. MeI
7. H2 , Pd-BaSO4, quinoline

1. BH3 , ether
2. NaOH , H2O2
(g)

1. LiAlH4
2. H2O
(h) CO2H
3. PCC
4. EtMgBr OCH3
5. H3O+
6. NaH and then MeI

Spectroscopic problems. Do not attempt these problems until Chapter 14 is read and understood.
55.

Little difference in IR

HO HO
3-Phenylpent-2-en-1-ol 3-Phenylpent-4-en-1-ol

Ethyl group

8 6 4 2 0 8 6 4 2 0
PPM PPM
One alkene H 3 Alkene H

56.
IR: the alcohol has a broad
peak at 3300 cm –1 and the
aldehyde will have a peak at O
OH 1725 cm –1 and the aldehyde
H at 2817 cm –1
4-methylpentan-1-ol 4-methylpentanal H

4 2 0 10 5 0
PPM PPM
aldehyde H at 9-10 ppm is absent in the alcohol
57.
O IR: Ester has a carbonyl at
~ 1725 cm-1 and the
alcohol has a broad peak OH
OCH2CH3 at ~ 3300 cm-1

4 2 0 4 2 0
PPM PPM
The ester has an ethyl group, with the CH2 group connected to O. The alcohol has
a CH2 group attached to O, but it is not part of an ethyl group. The broad OH
protn will also show up in the alcohol : here at ~ 3.8 ppm

58. The aldehyde proton is pushed downfield by a combination of electron withdrawing inductive
effects of the C—O bond plus the anisotropy of the carbonyl π-bond. The alkene has only the downfield
shift due to anisotropy of the π-bond, but there are no inductive effects.

59.
OH
No difference in the IR
OH

4 2 0 4 2 0
PPM PPM
A doublet methyl

60.
A B C

O
61. OH
A B H C

62.
O OH Br

A B C
CN CHO

D E

CH2NH2

F
Chapter 20
Solutions Manual
Organic Chemistry. An Acid-Base Approach 2nd Edition
95.

O
O
OH
(a) (b)
OH

2,2-Diethylcyclobutane-1-carboxylic acid 16-Phenylhexadecanoic acid

O OH O
O
(c) (d) (e) CN
OH
OH
Br OH
4-Bromo-3-cyclopropyl-2-
1-Butyl-1,4-butanedioic acid 3,3-Diethyloctanenitrile
hydroxyhexanoic acid

96.
O
Br
CO2Me O
(a) (b) (c)
Br Br O

Methyl 3,5-dibromohexanoate 3-Methylbutan-2-yl 3,3,5-Trimethylheptanoyl bromide


cyclopentanecarboxylate
Ph Cl O

(d) O (e) (f)


N
CONMe2
O O Ph CHMe2
3-Methyl-2-phenylbutanoic 3- 3-Chloro-N-ethyl- (1R,2 R)-N, N,2-Trimethylcyclohexane-
phenylpropanoic anhydride N-isopropylpentanamide 1-carboxamide
(use of cis- is OK)

O CN
(g) (h) (i)

O O CO2Et

Cyclopentanecarboxylic Ethyl 3-isobutyl-2,2,5-trimethylhexanoate 2,2-Dimethylhex-(3E)-enenitrile


1,1-dimethylethanoic anhydride
CN
(j)

2-Ethyl-2,5-dimethylhexanenitrile

NH2 O O
O
97. N
H O N
O

98. PBr3 SOCl2 CH3Cl PCl5 NaCl

99. The byproduct of the reaction is the strong acid trifluoroacetic acid. The buffer is added to prevent
this acid from reacting with the product and causing unwanted secondary reactions.

100. An alcohol reacts with HCl to form an oxonium ion, and chloride ion can displace water to form
the chloride in a substitution reaction. Reaction of the acid with HCl forms an oxocarbenium ion, which
is more stable than the oxonium in derived from an alcohol. In addition, chloride ion would have to
react via an acyl substitution reaction in which chloride ion displaces hydroxide, and chloride is not a
strong enough nucleophile to do that, given that hydroxide is a very poor leaving group.

101. NH3 , 25 °C Me2NH , 25 °C 1. SOCl2 2. NH3 1. MeOH , H+ 2. NH3 , heat Me2NH , 250 °C

102.
Ph O
Br 1. Mg
O
2.
Ph Cl

Ph
103.
Ph
OH

104.
H

Ph Ph OH2
Ph
Ph Ph
OH OH2 Ph
105. The major product or products are shown. No mechanisms are provided.
O SOCl
O
2

(a)
OH Cl

Me2NH , heat
(b)
CO2Et
O NMe2

cat H+ , butanoic acid


(c) OH O
heat , drying agent C3H7

O O O
cat H+ , H2O
(d)
heat OH
O HO
CN
1. HBr
(e)
2. NaCN , DMF

O
CO2H 1. SOCl O
(f) 2 O
2. CH3COOH

CO2Et 1. aq NaOH CO2H


(g)
2. dilute H3O+ + EtOH

O O
EtOH , reflux
(h)
cat H+
NEt2 NEt2
O O

(i) OH CH3CH2NH2
O –+NH3Et
20 °C

Cl 1. BH3 , ether Cl CN
2. NaOH , H2O2
(j) 3. SOCl2
4. NaCN , DMF
106. The adjacent methyl groups sterically hinder approach to the carbonyl carbon in 1,1-dimethylethyl
butanoate relative to methyl butanoate. Since reaction with hydroxide or with water under acid
conditions requires a nucleophilic attack at the acyl carbon, if that carbonyl is sterically hindered the
reaction will be much slower.

107. The product was methyl butanoate, from reaction with methanol. Methanol is more reactive than
isopropyl alcohol, and only a five fold excess of isopropyl alcohol is probably insufficient to
compensate for the difference in rate. The solution is to use isopropyl alcohol as the solvent, with no
methanol.

108. The bicarbonate is a base, and it reacts with any unreacted hexanoic acid. This reaction generates
the carboxylate anion of ethanoic acid, which is water soluble and easily washed away from the ester
product.

109. Because the acid catalyzed reaction is reversible, removal of the water product is necessary by the
use of a dehydrating agent or removal via azeotropic distillation. Either way, the reaction may be
incomplete and with higher molecular weight esters both of these procedures are problematic. Drying
agents are often inefficient, and some esters are sensitive to heat. Making the acid chloride is usually a
high yield procedure and the subsequent reaction with an alcohol is rapid and proceeds in high yield.
Using this procedure there are usually no other products, and any unreacted acid chloride is treated with
water and the resulting carboxylic acid is washed with sodium bicarbonate to remove it.

110. In one sense this is a silly question, but the point is to say that an ester is more reactive and easily
hydrolyzed, whereas an amide is less reactive and much more difficult to hydrolyze. This finding means
that the amide is more robust and many chemical processes may occur without disrupting the amide
bond.

111. In both reactions the tetrahedral intermediate shown is the intermediate. Ethoxide is a far better
leaving group when compared to a dialkylamide. Once this intermediate is formed, loss of OR will
always produce the amide, regardless of conditions for how the intermediate was formed.
–O
OR
R2N R

112. The acetyl chloride is so reactive that it reacts with the water in the air to form acetic and
hydrochloric acid. The fuming occurs when gaseous HCl is produced in the presence of moist air,
which leads to an aerosol of hydrochloric acid.

113. Washing with aqueous bicarbonate will convert all of the unreacted carboxylic acid to the salt,
which is removed by washing with water, leaving behind the neutral amide. An acid chloride is very
reactive, so washing an acid chloride with aqueous bicarbonate or acid will hydrolyze the acid chloride
to the corresponding acid, which defeats the purpose of making the acid chloride in the first place.

114. Thionyl chloride, oxalyl chloride, phosgene, phosphorus trichloride, phosphorus oxychloride,
phosphorus pentachloride.
115. The second product in this reaction is HCl. Triethylamine is added as a base. Reaction of
triethylamine with HCl gives triethylammonium chloride (HEt3N+Cl-). Removing one product in this
way drives the reaction towards the desired ester product. The water soluble ammonium salt is easily
removed from the ester by simply washing with water.

116. The major product or products are shown. No mechanisms are provided.

1. SOCl2 O
(a) CO2H
2. Me2CHOH , NEt3
O
1. aq NaOH ; H3O+
2. Me(Bu)NH NMe(Bu)
(b) CO2Me
3. 200 °C
O
CO2H 1. PCl
5
2. butanoic acid O
(c)

O O O O
1. EtOH , NEt3
2. MeOH , cat H+ Via transesterification
(d) Cl OMe

1. SOCl2
O
2. ammonia Via dehydration of 1° amide
(e) CN
3. P2O5
OH
O
1. Ac2O
(f) 2. cyclopentanol , cat H+ O
CO2H
Via anhydride
OH O
(g) CO2H
DCC O

diethylamine , reflux
(h) CO2Me CONEt2

EtOH (solvent)
(i)
cat H+ , reflux
CO2H CO2H CO2Et CO2Et

1. 6N HCl
(j) CN CO2Et
2. EtOH , cat H+
117. When phosgene is exposed to water (moist air) it reacts as a highly reactive acid chloride,
producing HCl gas and eventually carbonic acid (HOCO2H), which is unstable and decomposes to
carbon dioxide and water. The whitish gas is generated by exposure to moist air, which will produce an
aerosol of HCl, and is highly corrosive, and dangerous if inhaled. Note also phosgene may be inhaled
without hydrolysis. Inside the body, phosgene can react with the amine units of proteins, forming cross-
linked urea structures. Such reaction products produce many problems in humans once inhaled.

118. When thionyl chloride is exposed to water, the hydrolysis produces are sulfur dioxide (SO2) and
HCl. It is likely that the gaseous HCl, which when exposed to moist air forms an aerosol of
concentrated HCl. Since HCl is corrosive, it is likely to be responsible for the damage.

119.

δ+ δ+
N C N N C N N C N
H H O
O
O
H
O O O
O
O H
N C N O O
H O +
H N O
O H C N
H
O O
O

120.

O
heat
A cat H+ - H+
HO OMe
O O
H+

O H - H+ H+ O O H
- MeOH
OH OH OH
HO OMe O O
OMe O Me O
O
H Me
H

121. A five-membered ring is lower in energy than a seven-membered ring, due to diminished
transannular interactions and less torsional strain in the five-membered ring. As the lactone ring is
formed by cyclization, the transition state assumes the conformation of the ring being formed. If
formation of the five-membered ring is lower in energy in the transition state, then formation of the five-
membered ring lactone is faster than formation of the seven-membered ring lactone.
122. Give the major product for each of the following reactions.
O O
EtOH , cat. H+
(a) OEt
O
O O
OH
1-aminobutane
(b)
O reflux N-Bu
O
H3O+
(c) OH CO2Me
AcO CO2Me
O
O O Spontaneously cyclizes

MeNH2
(d) O N-Me
heat

O O

123. It is difficult to form a 17-membered ring because it is difficult for the distal OH group to attack
the acyl carbon, so the more common product is the ester shown. In other words, the intermolecular
ester-forming reaction is more favorable than the intramolecular lactone-forming reaction.

O
O
O
HO
OH HO O
O O

OH

124. The amide has resonance contributors that delocalize the electron pair on the oxygen, which
effectively shortens the C—N bond length
O O

R NH2 R NH2
125. 4-Aminobutanoic acid has a basic amine unit and an acidic COOH unit. An internal acid–base
reaction occurs to form the zwitterion shown.
OH O
H2N H3N
O O

126.
O O
(a) NH H2N
OH O
O
O
(b ) NH N
Cl

O
O
(c) NH N
OEt

(d) NH H–Cl NH2 Cl

O O O
(e) NH + CH3CO2H
N
O

(f) NH NH2 No reaction (N.R.)

127.
O
Cl O
Cl H
N
O NH
n
NH2 O
H2N
128.
HO2C CO2H SOCl2 (S)
(S) EtOH EtO2C N O
O O NH2 A H

O O
Cl (S) Cl EtO
O
NH2 Cl Cl (S)
(S)
NH O Cl (S) NH
NH O
Cl
O

129.
O O
1. propylamine
2. 250 °C N
OH H

O O
propylamine

Cl N
H
O O
propylamine
heat N
OMe H

130. The leaving group for acetic anhydride is acetic acid whereas the leaving group from the imide is
the amide, acetamide. In acyl substitution reaction, the amide is a poor leaving group, and reactions
with the amide simply do not give the acetylation reactions observed with the anhydride.
Me O Me H
Me N Me
versus
O O O O

131.
O
CO2H CO2H CO2H
Heat Heat
(Z) (Z) O (E) (E)
CO2H HO2C HO2C
O
Maleic acid Furan-2,5-dione Fumaric acid
(Maleic anhydride)
132.
O O
O OH
Ph Ph
(a) (b) Ph (c) Me
Me
O OH
O
(d) (e) (f)
Ph
Ph

133. The Cl is a good leaving group and acyl substitution with an acid chloride is much faster than acyl
addition to the ketone. In other words, the acid chloride is much more reactive.

134.
O
CO2H BrMg+ -C≡CCH2CH3
(a) (b) (c) (d)
O

CO2- +MgCl PhCONEt2


(e) (f) (g) (h)
HO
HO

(i) (j) (k) CN


O O
OH

135.
H OH
O H
+ H+ O O + MeOH - H+
Me
C3H7 OH C3H7 O
C3H7 OH C3H7 OH OH
H
OH
OH H O
+ H+ - H2O O O–H - H+
C3H7 OMe
C3H7 OMe O C3H7 OMe
OH H H C3H7 OMe C3H7 OMe

136.
O
RCO3H O

O
1. BH3 , ether 2. NaOH , H2O2
3. PCC O
4. Me3CMgBr 5. H3O+
6. PCC 7. MeCO3H O

137 The major product or products are shown. No mechanisms are provided.
OH O
1. MeMgBr , ether
(a) + 2. aq H+
N 3. cyclopentanecarboxlic acid H
H CrO3 Cl H+ catalyst
MeO HO
(b) excess H2O in THF
cat H+ O
O
1. SOCl2 O
2. Mg , ether
OH
(c)
3. 0.5 ethyl butanoate
4. H3O+
1. SOCl2
(d) CO2H CO2CHMe2
2. propan-2-ol

OH 1. CrO , H SO, aq acetone O


3 2 4
(e)
2. SOCl2
3. Ph2CuLi , THF , 0 °C Ph
1. SOCl2 O
(f) 2. MeNH2
CO2H
NHMe

(g) 1. SOCl2
SO2H SO2OCH2CHMe2
2. Me2CHCH2OH
O
1. KCN , DMF
(h) 2. H3O+ , heat
Br NMe2
3. Me2NH , DCC

Me3CO3H
O
(i) O
O

O O
Me3CO3H + MeCO2H
(j)
O
138.
OH
H O OH + H2O
OH
O + H+ - H+ + H+
R NMe2
R R NMe2 O R NMe2
NMe2 NMe2 OH
H H

OH H
-NHMe2 OH - H+ O
Me O
R N Me
OH2 R OH R OH OH
H

139. Conversion of octanedioic acid to the corresponding anhydride requires formation of a nine-
membered ring, whereas succinic acid generates a five-membered ring anhydride. The transannular
interactions of the nine-membered ring raise the activation barrier for cyclization to that ring so high that
it does not easily form. Formation of the lower energy five-membered ring is energetically easy.
O O
CO2H
CO2H
O O
CO2H
CO2H
O O

140.

1. O3 CO2H EtOH CO2Et


2. H2O2 CO2H H+ CO2Et
141.
O O
O
2-phenylethylamine
N -H+
O
N
cat H+ Ph
H Ph
O O O
O O OH
H+ N
Ph
-H2O
N O
O H O H
Ph
+RNH2 HO
H+ OH N
O -H+ O H+
H O H Ph
O Ph -H+ H2O
N N O OH
H H
O Ph
O H H N
O Ph
O H HO OH

142.
O O H H2O OH HO OH
H+ + H2O -H+
O O O O

O O O O
OH H O
HO O
H+ -H+ OH
OH
O H OH
OH

O O O

143.

O O O
(a) H3C S OEt (b) H3C S NEt2 (c) O (d) H3C S OH
H3C S O
O O O
O
144.
SO3H

NaOH SO3–Na+

145.
O O
HO P O O P O O O

OH OH O P O O P O
OH O

O
146. O
S
O O

O EtOH O HNMe2 O
EtO EtO EtO
147. P P P
EtO Cl EtO OEt EtO NMe2

NH2

N
N
O
148.
-O P O N N
O
O- H H
H H
OH OH

Synthesis Problems. Do not attempt these problems until you have read and understood Chapter
25.

149.
O O
CO2H cat H+ MeNH2 , heat
O N-Me
heat
CO2H
O O
150.
SOCl2
(a) SO2OH SO2Cl
EtOH
(b) (a) then SO2Cl SO2OEt

NH3
(c) (a) then SO2Cl SO2NH2

NaOH
(d) SO2OH SO2O –Na+

151. Show a complete synthesis for each of the following from the indicated starting material.
1. HBr
2. Mg , ether
3. propanal
(a)
4. PCC
1. 9-BBN , ether
2. NaOH , H2O2
(b)
3. PCC 4. EtMgBr
5. H3O+ 6. PCC
7. EtMgBr 8. H3O+
O O
1. H3O+ , heat
(c)
2. SOCl2
NH2 3. EtOH OEt

1. NaBH4 ; aq NH4Cl Ph
CHO 2. PBr3 3. Mg , ether
(d) Ph
4. benzophenone ; H2O
5. PBr3 6. KOH , EtOH
1. H3O+ , heat
(e) CN CO2Et
2. SOCl2
3. EtOH
1. BH3 , ether
2. H2O2 , NaOH Cl
(f)
3.Jones oxidation
4. SOCl2 O
Br
O 1.NaBH 4
2. H3O+ 3. PBr3
(g)
4. Mg , ether
5. Me2CHCHO
6. H3O+ 7. PBr3
1. Me2CHMgBr
2. H3O+ , heat O O
(h) CN
3. HOCH2CH2OH
cat H+

Spectroscopy Problems. Do not attempt these problems until you have read and understood
Chapter 14.

152.
O
CO2Et IR: two bands at about
IR: carbonyl at 1725 cm–1 NH2 1650 and 1540 cm–1

4 2 0 8 6 4 2 0
PPM PPM
NMR shows OEt, with triplet at 4.2 pm

153. Extensive hydrogen-bonding of the carboxylic OH effectively makes it more positive (large δ+),
and the more positive the proton, the less electron density, and the more deshielded. The range of
chemical shifts are used because carboxylic acids undergo different amounts of hydrogen-bonding based
on structure of the acid, concentration and the solvent used.

154. The amide I band is at 1630-1695 cm–1 and the amide II is at 1500-1560 cm–1. The amide I band
is associated with the iminium unit (C=N) and the amide I with the carbonyl (C=O). Since both forms
of the amide are usually present, we see both bands in the IR.
O O

R NH2 R NH2

155. Using IR, there must be a focus on the carbonyl region. These are the peaks one should
examine.
C=O aldehydes, ketones, acids, esters : 1690–1760 cm–1 (5.68–5.92 µ)
RCO2H: 2500–3000 cm-1 (3.33–4.00 µ) and the OH at 2500–3000 cm-1 (3.33–4.00 µ)
Acid chlorides : 1802 cm-1 (5.55 µ)
Anhydrides: two bands at 1818 cm-1 (5.50 µ) and 1750 cm-1 (5.71 µ)
The 1H NMR can be used. The 1H NMR of a carboxylic acid will show the acid proton, which
will be removed if the sample with treated with D2O. An acid chloride will look much like the acid, but
no acidic proton will be seen. An anhydride should show signals for both acid portions of a mixed
anhydride, or it will show a symmetric pattern for a symmetrical anhydride.
156. To distinguish a primary, secondary, or tertiary amide, examine the N—H region of the IR.
Examine the 3300-3500 cm-1 region. The primary amide should have two peaks, the secondary amide
one peak and there will be no peaks in that region for the tertiary amide.

157. Add the unknown to a solution of aqueous sodium bicarbonate. Hexanoic acid will react to form
the sodium salt, which is soluble in water (it will dissolve). The ester (ethyl butanoate) is insoluble in
the aqueous medium and will not react with the bicarbonate, so it will show up as an oil in the water.
There are also differences in the odor. Relatively low molecular weight carboxylic acids tend to have a
sharp odor, whereas esters tend to have a relatively sweet or pleasant odor.

158.
IR: 1725 cm–1 for the C=O of the
ketone, but the alcohol will have
O a broad peak at about 3300 cm–1 OH

3 2 1 0 4 2 0
PPM PPM
NMR: ketone shows two ethyl groups, The NMR shows three ethyl groups,
and two different CH2 resonate at and the only peak past 1.6 ppm is
about 2.3-2 ppm. the OH at about 3.8 ppm

159. The IR should show a broad band between 2500–3000 cm-1, and a carbonyl at ~ 1725 cm-1. The
1H NMR should show a signal between 1215 ppm for the COOH proton.

160. In the IR, the CN of the nitrile will show a moderate and sharp peak at 2210-2260 cm-1, whereas
the ketone will show a strong C=O band at 1725 cm-1. As the reaction proceeds, the nitrile peak should
diminish and eventually disappear, whereas the carbonyl peak from the product will appear and then
increase in strength as the reaction goes to completion.

O
3-Methylpentanoic acid
161. OH

O O

N N-Benzylacetoacetamide
H
162.
O O
Isobutyl formate

163. H

O O

O O
Diethyl allylmalonate

164.

O O
O
cis-dimethyl maleate

165. (Z) O

O O
Diisobutyryl anhydride
O
166.

Br OH 2,3-dibromopropionic acid

167. Br

O
Isovaleryl chloride
168. Cl
Chapter 21
Solutions Manual
Organic Chemistry. An Acid-Base Approach 2nd Edition
83.
O Cl
O N+ Cl Cl
HO Cl O-
(a) (b) (c) (d)

Cl Cl
N+ Cl
O O-
1,3,5-Trimethylbenzene m-Chlorophenol 3,5-Dinitroanisole Hexachlorobenzene
O
O O
OH
OH S
(e) (f) (g) OH (h)
OH O
Br O
I C N
O
4-Bromophthalic acid 4-Iodobenzenesulfonic acid 2-Cyanobenzoic acid Ethoxybenzene
O OH O (Phenetole)
O Br
N+ N+
-O O-
Cl
(i) (j) (k) (l)
C
Br N
OH
4-Bromo-3'-chlorobenzophenone 2,6-Dinitrohydroquinone o-Bromobenzonitrile 1,3-Dimethylbrenzxene
(m-xylene)

O
(m) (n) (p)
N
H
1,2-Di(2-methylphenyl)ethene N-Acetyl-3-methylaniline 2,2-Dimethyl-4-phenylhexane
(2,2'-Dimethylstilbene)

84.
NHMe
Br Cl OH
Me
HO2C CO2H C3H7
(a) (b) (c) (d)

Cl Me C3H7
Me
N,2,4-Trimethylaniline 2-Bromoisophthalic acid 1,3-Dichloro-5-methylbenzene 2,5-Dipropylphenol
SO3H
CHO CO2H

(e) (f) (g) O (h)

CN
F NO2 NO2 Br
Et
3-Fluorobenzaldehyde 3,5-Dinitrobenzoic acid 1-(2-Bromophenyl)ethan-1-one 3-Cyano-4-ethylbenzene-
sulfonic acid
CN I OH Me NO2
Me Br
(i) (j) (k) (l)

Et OH Br OMe
5-Ethyl-2-methylbenzonitrile 5-Iodobenzene-1,3-diol 1,2-Dibromo-3-methylbenzene 1-Methoxy-3-nitrobenzene

85.
O O
OCH3 OCH3 Br Br
Br NO2 HN HN
Br
NO2
(a) (b) (c) (d)

Cl
Br Br
NO2

86.
O OCH3

(a) (b) Ph (c) OCH3

O
NO2

(d) (e) (f)

(No reaction)
Deacativated aromatic rings
do not undergo Friedel-Crafts
reactions.
87. Both OH and the carbon group of the ethyl unit are electron releasing. Electron-releasing
substituents stabilize positive charge at the ipso carbon, which makes that intermediate more stable and
accelerates the rate of the reaction. Therefore, these groups are activating substituents in electrophilic
aromatic substitution.
OH CH CH NO O SO H
2 3 2 3

88.
NH2 NH2 CH3 NO2
H H H

Cl H Cl Cl
Cl

89.

O N
H

Cl

90. The major product or products are shown. No mechanisms are provided.

NO2
HNO3
(a)
H2SO 4

NO2 NO2
Br2 , AlCl3
(b)

Br
CH3 O H3C
CH3 O

Cl C3H7 C3H7
(c)
AlCl3 O
OCH3 OCH3
H3CO
Cl2 , FeCl3 Cl
(d)
Cl

1. oxalyl chloride
OH Ph
(e)
2. benzene , AlCl3
O O

1. H3O+ , heat
2. SOCl2
(f)
CN 3. 1-phenylhexane , AlCl3 O
O C6H13
C6H13
OCH3
OCH3 OCH3
Cl
Cl2 , AlCl3
(g)
NO2
NO2 NO2

CH3 Cl
CH3
Me3CCl H3C
CMe3
AlCl3
(h)
CMe3

1. Br2 , AlCl3
2. Mg , THF

(i) 3. 3-methylcyclopentanone OH
4. aq H+

SO3H
SO3 , H2SO 4
(j)
91. The electron-releasing groups stabilizes the positive charge on the ipso carbon in electrophilic
aromatic substitution, which makes those intermediates lower in energy, and the rate of their formation
is faster.
Br OH CH3 O NO2

92.
OMe OMe OMe OMe OMe
Cl Cl

Cl Cl Cl

93.
Cl Cl Cl Cl
Cl
Cl+

Cl H Cl H Cl H Cl H

94. The methoxy group is strongly electron releasing. The methyl group releases electron density to the
oxygen, which leads to a large δ- dipole, so it will stabilize a positive charge on the ipso carbon to a
greater extent. The electron-releasing capability of OMe is so great that the benzene ring will react
without the need for a Lewis acid to generate X+.

95.
NO2 NO2 O2N NO2 NO2
NO2
Cl Cl
Cl

Cl Cl NO2 O2N
96.

H+
H
OH H+ OH2 - H2O - H+

97.The sulfur atom in sulfonic acid is more electrophilic and more positive than the carbon atom in
acetic acid. Therefore, benzene reacts more readily. In addition, the sulfur atom is larger and can
accommodate another ligand more readily than carbon.

98.

Br AlCl3

E+Z
AlCl3Br

99.
OCH3 OCH3 OCH3 OCH3
H H H H
(a)
Br Br Br Br

OCH3 OCH3 OCH3 OCH3


H H H H

(b) Br Br Br Br

OCH3 OCH3 OCH3 OCH3


O O O O

NH NH NH NH
(c) H H H H
Br Br Br Br

H H H
(d)
Br Br Br

100. Reaction of the diene with the Lewis acid generates an allylic cation. The C=C unit of another
molecule of the diene is simply more reactive than the C=C unit of a benzene ring. Therefore, the diene
reacts preferentially to form a polymer.

101. An alkyl substituent on benzene is activating, because the carbon group is electron-releasing
relative the C+ of an arenium ion. Boron is less electronegative than carbon, so a PhB group should be
less electron-releasing than an alkyl group. Triphenylborane may undergo electrophilic aromatic
substitution, but it should be slightly slower than a methyl group, and it will be an ortho/para director.
Trisubstituted boron derivative are Lewis acids in their own right. It is not clear if the reaction will be
autocatalytic.

102. The carbonyl unit of the ester in phenyl acetate will withdraw electron density from the oxygen
atom, making it lower in electron density, and a weaker activating substituent. Anisole has an electron
releasing methyl group on oxygen, increasing the electron density and making it a stronger activating
substituent. Therefore, phenyl acetate will react slower than anisole in a reaction with bromine and
ferric bromide.

103. The π-bond of the ethylene unit is more electron rich, but a major reason is that another resonance
contributor can be drawn for the arenium ion, as shown. This extra resonance contributor is not possible
by reaction with ethylbenzene.

H H H H

Br Br Br Br
104.
OCH3
OCH3 CO2H CO2H
O2N Cl
(b) (c)
(a)
NO2
NO2 HO3S SO3H
NO2
NO2 NO2 OCH3 O
NO2
Cl
(d) (e) (f)

Cl Cl
Br NO2
Cl O
O OCH3
HN OCH3
O HN
(g) (h)

105. The oxygen atom of the OMe unit may coordinate to the Br+X- complex, which “holds" the
electrophilic Br+ closer to the ortho carbon.

106.

OH
NH2
(a) (b (c) (d) (e)

CO2H NH2
(f) (g) (h) (i)

ortho +para
O
NO2
(j) (k) N (l)
H

O
Cl CN
(m) OBu (n) (o)

107. Electron transfer generates a radical anion, but electron transfer will occur so that the negative
charge will not be on the carbon bearing the oxygen. Hydrogen transfer from the solvent leads to the
radical, and a second electron transfer leads to another carbocation. A second hydrogen transfer leads to
the alkoxide. Protonation in aqueous acid yields the alcohol, but the C=C unit isomerizes to form an
enol, where the C=C units are connected (this is a conjugated system; see Chapter 23), which is more
stable. The enol tautomerizes to the carbonyl, so the final product is cyclohexenone.

OH O– OH OH O

O– O– O– O–
108.

(a) (b) (c)

22 π-Electrons 6π-Electrons
aromatic aromatic 6π-Electrons
not aromatic

(d) (e) (f) (g) (h) (i)

4π-Electrons 2 π-Electrons 4π-Electrons


4π-Electrons 8π-Electrons not aromatic 10 π-Electrons
not aromatic not aromatic aromatic
not aromatic aromatic

(j) (k) (l) (m)

6π-Electrons
not aromatic 18 π-Electrons
aromatic 20 π-Electrons
not aromatic 30 π-Electrons
aromatic

(o)
(n) 14 π-Electrons
aromatic

18 π-Electrons
aromatic
109. If reaction with Br+ places a positive charge on the ipso carbon in one of the resonance
contributors, then reaction at that carbon will be faster, and all such substitution products will be formed.
The Br is shown at each possible different position, and a * is shown to indicate the position of the
positive charge for each resonance contributor.

*OMe *OMe
OMe OMe
H *
* * *
Br * * H
H
* * * Br * Br
* *
* * *
*
* *
* *
OMe * OMe * OMe * OMe
* *
* * * *
* * * *
* * * *
H
H
* * * *
* *
* * Br Br
* H * *
*
* * Br Br H
Therefore OMe
OMe OMe OMe
Br

Br

Br

Br

110. The OMe unit is electron releasing and activating (reacts faster). Therefore, the ring bearing the
OMe will react faster than the other ring, which is another way of saying that the charge can be
delocalized on the oxygen of OMe, which of course occurs only in the ring bearing the OMe.
111.
OCH3
OCH3
OCH3

H
H
Br
Br OCH3
OCH3
OCH3

H
H
H Br
Br OCH3
Br OCH3
OCH3

H
H
H Br
Br
Br

112. The arenium ion for attack at each carbon is shown. A * is used to show the position of the
positive charge in each ion. It is clear that attack via Ca leads to an arenium ion in which the positive
charge is on the ipso carbon, which allows the charge to be delocalized on the oxygen. This
intermediate is more stable than the arenium ion via Cb, where the positive charge never appears on the
ipso carbon. Therefore, the product resulting from reaction via Ca is preferred.
H Cl

* * OMe
* *
1
* *
* *
Attack via
b Cb * *
* * *
OMe
1
* *
* *
Attack via
* OMe
a
Ca * * 1* * *
* *
OMe * *
H Cl
* * * *

Cl
113. The major product or products are shown. No mechanisms are provided.
OMe
OMe OMe
HNO3 , H2SO 4 NO2
(a)

NO2
Br2 , FeBr3
(b)

Br
NO2 O
NO2 NO2
acetyl chloride
(c)
AlCl3 O

OMe OMe

Cl2 , AlCl3
(d)
Cl
OMe OMe
NO2 Me NO2 Me NO2 Me
Br
Br2 , AlCl3
(e)
Br
Me Me Me
Me Me

Na , NH3
(f)
EtOH

Me Me
114.
OH CN Br Cl

(a) (b) (c) (d)

I N=NPh

(e) (f) (g)

115. The nitro group is electron withdrawing, which stabilizes the carbanion intermediate more than the
bromine. Remember two things. First, the nucleophilic aromatic substitution places a negative charge
in the ring after reaction with the benzene ring and not a positive charge, such as is formed during
electrophilic aromatic substitution. Second, the bromine is polarizable so if the negative charge is on the
ipso carbon adjacent to a bromine atom, Br will take on a positive dipole, which is stabilizing.
However, the nitro group is more stabilizing than the bromine.

116. At elevated temperatures, the hydroxide ion will attack the carbon bearing the sulfonate anion,
because SO3 is a leaving group under these conditions. This means that the reaction occurs via
nucleophilic aromatic substitution, with a negative charge in the intermediate. The nitro group at C4
will stabilize this charge whereas there is no resonance contributor where the negative charge appears at
C3. Therefore, the 4-nitrobenzenesulfonic acid will react faster to yield the product, 4-nitrophenol.

117. Give the major product of each of the following and show the intermediate product for each step:
HNO3 NO2 NH2 N2+BF4– F
H2SO 4
(a)
H2 . Pd-C NaNO2 , HBF4 150 °C

Me Me Me Me
H2 , Pd-C
(b)
Br2 , FeBr3 HNO3
(separate paraBr HSO4 Br NO2 Br NH2
product) Me Me
NaNO2 , HCl CuBr

Br N2+ Cl – Br Br
Br NHAc Br NH2 Br N2+Cl – Br OH
(c)
aq NaOH ; NaNO2 , HCl 160°C ,
aq HCl aq H2SO 4
NH2 N2+Cl – OH O –Na+
O
(2R)-iodopentane
(d)
NaNO2 , HCl 160°C , NaH , THF
aq H2SO 4

NO2
NH2
HNO3 , H2SO 4 H2 , Pd-C
(e)

N2+Cl –
NaNO2 , HCl
1-aminonaphthalene NH2
N
N

Br Br Br
Br HNO3
NaNO2 , HCl
Br2 , FeBr3 H2SO 4 H2 , Pd-C
(f)

–Cl+N
O2N H2N 2
Br Br Br MgBr OH
160 °C
MeI Mg , THF cyclopentanone
aq H2SO 4 NaH

HO3S +Na–O MeO3S


3S MeO3S
MeO3S
OH O
I MgI
PCC
(g)
Mg , THF butanal
O O
O

CuBr
HNO3 , H2SO 4 NaNO2 , HCl

N2+Cl – Br
NO2

118.
OH
CO2H
SO2OCH2CH=CH2
(a) (b) (c)

NMe2
(d) Via benzyne
NMe2
119.

NH2 NH2
(a) (b)

NH2 NH2
OMe NH2
OMe OEt OEt

NH2
(c) (d)

NH2 NH2
NH2 NH2
NH2
(e)
NH2

120. There is only one; methyl cyclohexyl ether. O


CH3
121.

OH OH O OH O

122.
OMe NO2
OMe NO2

Na , NH3 Na , NH3
EtOH EtOH
123. (i) Nitrobenzene + H2/Ni(R) This combination gives aniline
(ii) HCl/NaNO2. This mixture converts aniline to benzenediazonium chloride
(iii) KF, THF. This reagent converts the diazonium salt to the final product, fluorobenzene.

124.
Me

N2+OAc– N

Ph2NMe N
N

MeO OMe

1,3,5-Trimethoxybenzene N
N
MeO

125. In these reactions, the leaving group is dinitrogen (N2), from the Ar-N2. It is essentially a
nucleophilic aromatic substitution, so an electron-withdrawing group (e.g., nitro) will stabilize the
carbanion intermediate and an electron-releasing group such as OMe will destabilize the intermediate.
Based on this analysis, 4-nitrobenzenediazonium chloride should react faster.

126. Benzylamine, PhCH2NH2.

127.
Br MgBr CO2H CO2Et CH2OH
Mg 1. CO2 LiAlH4
EtOH
H+ + EtOH
2. H3O+

128.

CHO CH2OH CH2Br


H2 MeC≡C: – H2
PBr3

PtO2 Lindlar
129. The product formed by this reaction is phenol (PhOH), which is rather acidic (pKa ~ 10). As
phenol is formed, it will react with the NaOH to generate the phenoxide anion, PhO-Na+. Therefore at
least 2 equivalents of NaOH are required: one to react with the chlorobenzene to form the phenol, and
the second will react with the phenol product. If only 1 equivalent of NaOH is used, only about one-half
of the chlorobenzene will react.

130. The mechanism is nucleophilic aromatic substitution (SNAr), in which water attacks the carbon
bearing the diazonium salt. Following proton transfer, loss of nitrogen leads to phenol.
If the diazonium salt is heated with CuBr, the product is bromobenzene, PhBr.

- NH - NH
2 2

NH2
N2+ NH2
131.

132. Since iodide is a better leaving group than chloride, and this is a substitution reaction, where it is
assumed that iodobenzene will react faster than chlorobenzene.

133.
Cl

Cl Cl
Cl
B

Under these conditions, alkynes maybe reduced to the alkene (A) and the benzene ring may be
reduced via Birch reduction to B. However, it is much more likely that the alkyne anion will form faster
via the acid–base reaction.

134.

Na , NH3

(E)-1,2-diphenylethene
(trans-stilbene)
SYNTHESIS. Do not attempt these problems until Chapter 25 has been read and understood.

135.

Br Br Br
Br
Br2 propanoyl
chloride 1. NaBH4 PBr3
FeBr3
2. aq NH4Cl
AlCl3

O HO Br
separate out
the ortho
product

136. The major product or products are given. No mechanisms are provided.
1. Br2 , FeBr3
(a) Cl Br
2. Cl2 , AlCl3
(separate ortho)

1.HNO 3 /H2SO 4 OEt


2. Br2 , AlCl3 3. H2 , Pd
(b)
4. HONO 5. H3O+ , heat
6. NaH then EtI
Br
F
1.HNO 3 /H2SO 4
2. Br2 , AlCl3 3. H2 , Pd
(c)
4. HBF4 , NaNO2
5. 150°C
Br

1. Br2 , FeBr3
2. SO3 , H2SO 4
(d) Br SO3H
seaprate ortho
F
1. propanoyl chloride, AlCl3
2. HNO3 , H2SO 4 3. MeMgBr ; H3O+
(e)
4. H2 , Pd 5. HBF4 , NaNO2
6. 150°C
OH
CH2CH2CH2CH3
1. butanoyl chloride , AlCl3
2. HNO3 , H2SO 4
(f)
3. N2H4 , KOH 4. H2 , Pd
5. HONO 6. H3O+ , heat
OH
OH
1. HNO3 , H2SO 4
2. H2 , Pd 3. Ac2O
(g)
4. HNO3 , H2SO 4 (separate ortho)
5. HONO 6. CuCN
7. H3O+ 8. HONO 9. H3O+ , heat
CN

1. propanoyl chloride , AlCl3


(h) 2. Me2CHMgBr
3. H3O+ 4. Br3
5. KOH , EtOH

1. HNO3 , H2SO 4 Br Br
(i) 2. Br2 , AlCl3

3. H2 , Pd 4. HONO
5. CuBr

137. Syntheses are shown for each compound from a selected starting material.
O F
1. HNO3 , H2SO 4 Assume hydrogenation of NO2
2. H2 , Pd is faster than hydrogenatin of C=O
(a)
3. HBF4 , NaNO2
4. 150°C

O
NO2 Cl
1. Cl2 , AlCl3
2. H2 , Pd
(b)
3. HBF4 , NaNO2
4. 150°C
F
CO2Me (R)
1. Br2 , AlCl3
2. LiAlH4 ; H2O O
(c)
3. NaH , THF
4. (2S)-bromobutane
Br
CN 1.CH 2=CHCH2MgBr NH2
2. H3O+
(d)
3. PBr3 4. NaN3 , THF
5. LiAlH4 ; aq. H+

NHAc 1. HNO3 , H2SO 4 - separate ortho


2. Me3CBr , AlCl3 3. H3O+
(e)
4. HONO 5. H3PO2
6. H2 , Pd-C 7. HONO
8. H2O , 150 °C
OH
1. HNO3 , H2SO 4
OEt
O
2. N2H4 , KOH
3. H2 ,Pd
(f)
4. HONO 5. H3O+
6. NaH 7. EtI

Spectroscopic problems. Do not attempt these problems until Chapter 14 has been read and
understood.

138.
NH2 Little difference in 1 H NMR, except OH
the phenol will have only 1 H that exchanges
with D2O, whereas the amine will have 2 H
IR: phenol OH at 3300 cm-1 is
broader and more intense
Phenol is soluble is aqueous NaOH because phenol is acidic (pKa, 10), whereas aniline is
insoluble in aqueous NaOH.
139. The only real difference is in the aromatic region of the 1H NMR, ~ 7 ppm. 4-Methylanisole is
more symmetrical and will have only two identical signals, as shown, whereas 2-methylanisole has more
signals and the aromatic region is a bit more complex..
OCH3

OCH3

8 6 4 2 0 8 6 4 2 0
PPM PPM

140. Aniline will have amine NH at ~ 3300 cm-1, and since it is a primary amine, there will be two
peaks (a doublet). N-Acetylaniline will have one peak in the NH region, but will have the amide I and
amide II peaks at 1630–1695 and 1500–1560 cm-1, respectively

141. Benzonitrile is the expected product. The only discernable difference is in the IR. Benzonitrile
will have the nitrile peak at ~ 2260 cm-1, whereas benzamide will have the amide I and amide II peaks at
1630–1695 and 1500–1560 cm-1.
O
CN
NH2

8 6 4 2 0 8 6 4 2 0
PPM PPM

142. The aromatic region for methylnaphthalene will integrate to nine protons, whereas the aromatic
region of toluene will integrate to five, relative to the three-proton singlet for the methyl group.

143. There are no protons, so there will be NO signals in the 1H NMR.

CHO
144.
Hexaethylbenzene

145.

N
C
2,6-Dimethylbenzonitrile

146.

4-Isopropylphenetole

147. O

OH
4-Methylphenol

148.

Cl
1,4-Dichlorobenzene

149. Cl

N-Ethyl aniline
N
150. H
Chapter 22
Solutions Manual
Organic Chemistry. An Acid-Base Approach 2nd Edition
83.
O O
Ph Ph

O O

pKa , 19 pKa , 19–20 pKa , 20–21 No α-proton

84. The pKa of acetone is ~ 19. The carbon of the cyano group is δ+, which makes acetonitrile less
acidic (pKa, 24). The nitro group is more electron-withdrawing and the charge can be delocalized on the
nitro group, which makes it even more acidic, pKa = 11.

85. Ammonia is a protic solvent, so the enolate anion will react to regenerate the ketone. These are
thermodynamic conditions.

86. The enolate stabilized by two adjacent carbonyl groups is more stable than the enolate anion
stabilized by only one carbonyl. The presence of the methyl group, which is electron releasing, will
provide further stabilization to each enolate anion.
O O O O O
O
> > >

N N
87. N N C C N N
C C C C

88. Aprotic solvents are compatible with kinetic control conditions and protic solvents are compatible
with thermodynamic control conditions.
Protic
Protic O O Protic Protic
MeOH O NH3 Me2NH
Aprotic O OH OEt
Aprotic Aprotic
89. The reaction with LDA generates the kinetic enolate, and hydrolysis will simply protonate the
enolate anion to regenerate the original ketone. Because these are kinetic control conditions, the
stereochemistry at the adjacent methyl group is not affected.
H H
O O
1. LDA , THF , –78 °C
2. dilute aq H+
CH3 CH3
H H
OTs OTs

90.
Ph OH O
PhCHO C3H7CO2Et

O O
O I O
(S)
(R)

O
O

91.

O
O
(a) N Li N H
Li+

O
O
(b)
N Li+ N
Li O H
O
(c) Li-NH2 NH3
Li+
O
O
(d) NaH Na+ H-H
92.
(a) butan-2-one (b) butanal (c) 2,5-dimethylhexan-3-one
O H O

O
H H H

(d) 3-methylbutan-2-one (e) 4-methylheptan-3-one (f) 3,3-diphenylpentan-2-one.

O O

H H
O H

93.
O O
O O O
(a) (b) (c)
H
Kinetic Thermodynamic Kinetic & thermodynamic Kinetic Thermodynamic
since only one enolate anion
is possible
O O Ph Ph

(d) (e) (f)

Me Me O O O
Kinetic Thermodynamic Kinetic Thermodynamic Kinetic & thermodynamic
since only one enolate anion
is possible

94. These are kinetic control conditions. The Me2NLi is a strong base that generates Me2NH as the
conjugate acid, which is a much weaker acid than the ketone. This combination favors the forward
reaction but not the reverse (Ka is larger). The aprotic solvent THF does not have an acidic proton to
react with the enolate anion, which also favors a larger Ka. The low reaction temperature allows the
forward reaction to occur, but provides less energy to overcome the activation barrier for the reverse
reaction. The short reaction time does not give the reaction time to equilibrate: reaction of the strong
base and the ketone is rapid but reaction of the enolate anion with the weak acid (the amine) is slow.

95. Kinetic conditions are preferred. The same enolate anion is generated by either set of conditions,
but under equilibration conditions (thermodynamic), there is unreacted aldehyde in the equilibrium,
which may react in an aldol condensation of the aldehyde and the enolate anion (self-condensation).
Under kinetic control conditions, the aldehyde is effectively converted entirely to the enolate anion, so
there is little chance of self condensation (self-condensation is minimized).

96. Removal of the α-proton generates a resonance-stabilized enolate anion. Removal of the aldehyde
proton places a negative charge on the carbonyl carbon, which is normally polarized δ+C=Oδ-, and such
a carbocation is not resonance stabilized and is indeed very high in energy. It does not form, which
means that a base will NOT remove the aldehyde proton.
97. The aldol condensation products for each reaction is shown.
O
O
1. LDA , THF , -78 °C
(a) 2. cyclopentanone

3. hydrolysis
HO
HO
1. LDA , THF , -78 °C
2. 3-phenylpentanal
(b) CHO 3. hydrolysis Ph
CHO
O 1. NaOEt , EtOH , reflux
O
(c)
2. hydrolysis

O
O 1. LDA , ether , -78 °C OH
2. 4-ethyl-2-methylheptan-3-one
(d) 3. hydrolysis

Ph Ph HO
OH O
1. KOt-Bu, t-BuOH , 80 °C
(e)
O
2. hydrolysis

1. NaOEt , EtOH , reflux


2. cyclopentanone OH Thermodynamic, so there will be the
(f) self-codensation product of butan-2-one,
3. hydrolysis
and perhaps of cyclopentanone
O

1. NaOEt , EtOH , reflux O O


O hexan-2-one
(g) + other aldols
2. hydrolysis

OH
1. LiN(iPr)2 , THF , -78 °C O
O 2. hexan-2-one
(h) 3. hydrolysis

OH

98. Benzaldehyde does not have an α-proton, so formation of an enolate anion is impossible. The only
enolate anion that can be formed is from benzophenone, and it will react with benzaldehyde because the
aldehyde is more reactive to acyl addition than a ketone.

99. 3,5-Diethylheptan-4-one has a great deal of steric hindrance at the α-carbon atoms, so reaction with
LDA will be rather slow, particularly at the low temperature.
100. Under these thermodynamic conditions, the enolate anion from pentanal with react with itself in a
self condensation to yield the aldol product shown. To obtain the mixed aldol, kinetic control conditions
must be used, treating the aldehyde with LDA in THF at -78 °C, and then adding the hexanal.
1. NaOEt , EtOH CHO
reflux
CHO
2. hexanal
HO
1. LDA , THF CHO
-78 °C
CHO
2. hexanal
HO
101.
O
O
1. LDA , THF , -78 °C

(a) 2. hydrolysis

O CHO
HO
OH
1. NaOEt, EtOH, heat
(b) OHC CHO
2. hydrolysis

O O OH
1. LDA , THF , -78°C
(c) 2. hydrolysis O

CHO
O OH
(d) 1. LDA , THF , -78°C
CHO Ph
Ph 2. hydrolysis

102. Reaction with LDA generates the enolate anion from the 1,3-diketone unit, as shown, because that
is the more acidic proton. Intramolecular aldol condensation generates a six-membered ring as shown.
OH O
H3 O+
LDA , THF

O O O O O O O
103. Under thermodynamic control the more substituted enolate anion will be formed, and subsequent
internal aldol condensation will form the six-membered ring with the distal carbon rather than the four-
membered ring from reaction with the closest carbonyl.
O

O O O Me OH
O
1. NaOEt , EtOH , reflux
Me Me 2. hydrolysis Me
Me Me
Me Me

104. Sodium methoxide can react with the ethyl ester to form the methyl ester. The expected Claisen
product is the keto ethyl ester, but the keto methyl ester can also form if the methyl ester is generated as
suggested.
O O
O O
MeO OEt
OMe OEt
O O

105.

O MeO2C CO2Me O
EtO2C CO2Et Ph
(a) (b) (c) (d)
HO
Ph EtO2C CO2Et CO2Et
CO2Et

O
CO2Et HO2C
(e) O (f) CO2Et (g)

CO2Et Ph
O
CO2Et

106.
O
O CO2Et
1. LDA , THF , -78 °C Ketone is more acidic
(a) than the ester
2. mild hydrolysis

O
O
H CO2Et H
Ketone is more acidic
1. LDA , THF , -78 °C than the ester
(b) 2. mild hydrolysis
CHO
CHO
H H

(c) EtO2C 1. LDA , THF , -78 °C


CO2Et 2. mild hydrolysis
CHO
Aldehyde α-H is most acidic, and formation
of 6-membered ring is energetically
CO2Et more facile than forming a 9-membered
ring
CHO
O

107. The product or products are shown. No mechanisms are provided.


1. NaOEt , EtOH , reflux
CO2Et
(a) CO2Et
2. hydrolysis

O
1. LDA , THF , -78 °C CO2Me
(b) CO2Me 2. methyl pentanoate
3. hydrolysis

1. LDA , THF , -78 °C


2. 2-methyloctan-4-one
(c) CO2Et OH
3. hydrolysis
CO2Et
1. NaOEt , EtOH , reflux
CO2Et Me3C-CO2Et No new products, only
(d) starting materials
2. hydrolysis
CO2Et Ph CO2Et
O
CO2Et 1. NaOEt , EtOH , reflux CO2Et
(e) ethyl 3-phenylpropanoate OEt
2. hydrolysis O
Ph O Ph O Ph O
O
1. 2 eq. SOCl
2
CO2H 2. excess MeOH , NEt3
(f)
3. NaOMe , MeOH , reflux
CO2H 4. hydrolysis CO2Me

1. DCC , EtOH CO2Et


2. LDA , THF , -78 °C
(g) CO2H
3. ethyl cyclopentanecarboxylate
4. hydrolysis
O
O
CO2Et CO2Et 1. NaOEt , EtOH , reflux
(h) EtO2C
2. hydrolysis

108. The major product or products are shown. No mechanisms are provided.

O
1. LDA, THF, -78 °C
2. ethyl 2-methylbutanoate
(a) O
3. hydrolysis
O
1. NaOEt, EtOH, reflux, PhCHO CO2H
(b) CO2Et
2. vigorous hydrolysis

Ph
1. LDA, THF, -78 °C
2. pentan-2-one

(c) 3. vigorous hydrolysis OHC No β-H for elimination, so


CHO no conjugated aldehyde
HO
1. NaOEt, EtOH, reflux
CHO 2. vigorous hydrolysis CHO
(d)

CHO 1. LDA, THF, -78 °C


(e) CHO
2. vigorous hydrolysis
CHO

1. LDA, THF, -78 °C


2. ethyl butanoate O
(f) O
3. 6 N HCl

109.
PPh3+ Br– PPh3+ Br–
(a) (b) (c) Ph3P
(d) (e) I (f) PPh3
P
CH3

110.

O O
(a) (b) (c) (d)
Bu Br Ph P OEt
3
Ph

Ph
(e) PPh3 OH (f) (g)

111.

(a) (b)
O PPh3 O PPh3 O PPh3 O PPh3

(c) (d)
O PPh3 O PPh3 O PPh3 O PPh3

112.
SMe2 Br SMe2 Br SMe2

O SMe2
SMe2 Br O
+ Me2S
O

113. Diphenylphosphine will react with 2 equivalents of an alkyl halide, so there will be two sites for
generation of an ylid. Triphenylphosphine can react with only 1 equivalent of an alkyl halide, so there
will be only one site for generation of an ylid.
114.
aq. OsO4 OH O
t-BuOOH 2 PDC

OH O O
O
1. LDA , THF , –78°C

2.
Br O
O

115. If the carbonyl oxygen attacks the acidic proton, electron transfer
must dump electrons on carbon to form a carbanion, which is very high in
energy and does not form under most thermolysis conditions. The lack of a
stable product means there is no place to “dump” the electrons if CO2
departs, and the energy requirements for this reaction are such that it does
not occur.

116.
CO2H N Ph CO2H

CO2H PhCHO CO2H

CO2H CO2– HN CO2– HN


H
N H Ph
CO2H H CO2H CO2H
N O

HO H2O CO2H - H2O


O CO2– H3O+ Ph CO2H
CO2– HN

Ph CO2H Ph CO2H
Ph CO2H CO2H

117. The π-bonds of the benzene ring may react a weak base, and loss of carbon dioxide generates a
carbanion. Although carbanions are difficult to form, this one is resonance stabilized, so the energy
requirements are feasible, but high temperatures are required. Loss of a hydrogen atom from the
intermediate leads to benzene.
O
- O=C=O
O
H
H
H
O
NaOEt O OH
EtOH H3O+
CH2(CN)2 H Ph
118. –CH(CN)
2 Ph Ph
H H
CH(CN)2 CH(CN)2

119.
NC NC
NC O LDA NC O CN H3O+ CN
OH OH
NC NC

120. The thermodynamic enolate anion leads to a product in which there is no β-hydrogen atom, so
elimination is not possible. The product from the kinetic enolate anion, however, does have a β-
hydrogen and elimination proceeds as shown.
O
O–
O
No β-hydrogen, so
Ph no elimination
Ph Ph
OH
Ph
O– O O
O Ph Ph
HO

Ph Ph Ph Ph

121. The carbanion derived from dinitromethane is extensively delocalized due to the nitro groups, with
three resonance contributors. The stability of this carbanion makes deprotonation very facile. This
means that the proton is rather acidic and will be removed with a relatively weak base (e.g., carbonate).
It is likely that dehydration is very facile because the C=C unit will be conjugated to the two electron-
withdrawing nitro groups, which will facilitate dehydration.
OH
O2N CHO +
O2N Na2CO3 H 3 O NO2

O2N
O2N NO2
O O O
O N N N N
O N O N
O O O O O
O
122. The enolate anion is a base as well as a nucleophile. Reaction as a nucleophile yields the
calculated product, but reaction as a base with the alkyl halide leads to the alkene via an E2 reaction. A
mixture of (E) (Z)-isomers are formed because the β-carbon of the alkyl halide is not chiral.
O
E2 Br
(E)

(Z) (R)
O– (S)

SYNTHESIS. Do not attempt these problems until Chapter 25 has been read and understood.

123.
O Ph3P=CH2
(a)
H
O Ph3P
(b)
H (E)
H
Ph3P=CH2CH3
(E)
(c)
O

O
Ph3P=CH2
(d)

H Ph3P=CHCH2Ph
(e) (E)
O Ph

Ph3P
(E)
(f)
O

Ph3P
(E)
(f)
O
124.
CHO O O
O OH CO2Et
(a) (b) (c) (d) Ph

125. A synthesis is shown for each disconnection.


1. LDA , THF , -78 °C
2. methyl butnoate
(a)
O O O
1. LDA , THF , -78 °C
O
O 2. 3-pentanone
(b)
3. aq. H+
OH
1. excess EtOH , H+
2. NaH 3. EtI
(c) CH2(CO2H)2
4. NaH NH2
5. Me2CHCH2Br
6. H3O+ 7. 200 °C O
8. SOCl2 9. NH3

1. EtOH , H+ CO2H
(d) CO2H 2. LDA , THF , -78 °C

3. 2-methylbutanal O
4. PCC
5. aq. H+
1. LDA , THF , -78 °C CN
CN 2. pentanal
(e)
3. PCC
O
1. H3O+ Ph
2. SOCl2
3. Et2CuLi
(f) CO2Et
4. PhCH=PPh3

Spectroscopy Problems. Do not attempt these problems until Chapter 14 has been read and
understood.

126. The IR will show no difference, but the 1H NMR for a methyl ester will show a single that
integrates the 3H at ~ 3.5 ppm, whereas an ethyl ester will show a quartet at ~ 3.6 ppm that integrates to
2H and a triplet at ~ 1.2 ppm that integrates to 3H.
127. It really depends on the position of the keto units and the ester group, as that will determine the
chemical shift and multiplicity in the 1H NMR. The carbon of a ketone and an ester are very similar, so
it is unlikely this can be used to distinguish them. The most reasonable difference is the OCH signal of
the methyl ester, at ~ 3.5 ppm, which will be lacking for the diketone.

128. There is little difference in the 1H NMR. A nitrile will show a medium band at 2220 cm-1 in the
IR, whereas the ketone will show a strong band at 1725 cm-1. The imine is likely difficult to isolate, but
imines show a band at ~ 1660 cm-1 for the C=N unit.

3 2 1 0
PPM

CN
NH O

3 2 1 0
3 2 1 0 PPM
PPM

129. The band at ~ 3340 cm-1 is likely due to the O—H absorption of the tertiary alcohol product.

O
O–

OH

130. The lack of a signal at 2600 cm-1 indicates it is not an acid, and the signal at ~ 1670 cm-1 indicates
the presence of a C=C—C=O unit. Therefore, it is likely that the initially formed alcohol lost a
molecule of water via dehydration to yield the alkene shown.
CO2Et Ph CO2Et CO2Et
1. LDA 2. PhCHO Ph
2. dilure aq. H+ CO2Et
CO2Et HO CO2Et

131. The prominent COOH signal in the IR and the presence of an methyl group in the 1H NMR are the
most salient feature. The acid will not have the methyl group at 3.5 ppm, but will have a COOH signal
~ 12 ppm. There is an extra carbon atom in the 13C NMR
O IR: 1725 cm-1 O IR: 1725 cm-1
and broad band
CO2Me CO2H 2500–3000 cm-1

13C

200 100 0 200 100 0


PPM PPM

1H

4 2 0 10 5 0
PPM PPM

132.
IR: Broad peak at 3300 cm-1 for O—H
HO Ph mass spectrum: Weak or absent M, but
a significant M-18 peak.

8 6 4 2 0 150 100 50 0
PPM PPM

133.
Ph

Kinetic Thermodynamic
O Ph Very little difference
in the infrared
O

8 6 4 2 0 8 6 4 2 0
PPM PPM
Ethyl group Two methyl groups + CH at about 2.8 ppm
134.
O O

Little difference in the IR

3 2 1 0 3 2 1 0
PPM PPM
The presence of the propyl group is
obvious in the 1H NMR

135.
O
HO O
O

O OH
Not much difference 1H NMR: Singlet methyl 1HNMR: 2 different
in the IR CH + CH2 adjacent to C=O CH2 adjacent to C=O
O O

O
O O

OH
O OH
More complex 1 H NMR
More complex 1 H NMR
2 propyl groups
2 propyl groups + 2 singlet methyls next to C=O
2 different singlet methyl groups,
one next to C=O, the other β to OH

Hexane-2,5-dione

136. O
Methylenecyclopentane
137.

O O

Ethyl 3-oxopentanoate
138. O

O
HO
4-Hydroxy-4-methylpentan-2-one
139.

140.

OH O O – O
A B C D O E OH

141.
CO2H CO2Et CO2Et CO2H

CO2H CO2Et CO2Et CO2H CO2H


A B C D E
Chapter 23
Solutions Manual
Organic Chemistry. An Acid-Base Approach 2nd Edition
45.

(a) (b) (c) (d)

2E,5Z-Nonadiene 1,2-Diethylcyclohexadiene Hex-1-en-3-yne 2,3-Dimethyl-1,3-butadiene

O H

(e) OH (f) O (g)


2,4-Pentadienoic acid 3E,5E-Dodecadienal 1,5-Cyclooctadiene

(h) (i)

1,4-Diphenyl-(1 E,3E)-butadiene 2,3,4,5-Tetramethyl-2,4-hexadiene

46. It is clear from the structures that the s-cis conformation of the (2Z,4Z)-diene has severe steric
interactions for the methyl groups that raises the energy of the conformation. The methyl groups are far
apart in the (2E,4E)-isomer, which makes this conformation lower in energy relative to the s-cis
conformation of the (2Z,4Z)-isomer.

Hexa-(2E,4E)-diene Hexa-(2Z,4Z)-diene

47. HC≡C-C≡C-CH3. No! The alkyne units are linear, so s-cis and s-trans isomers are not possible.

48.
O
O NC
(a) (b) (c) (d) O
O

Ethyl benzoate Hex-3E-en-2-one Pent-2-ynenitrile 2,7-Diethylcyclo-


hept-2-en-1-one
O O
O O
(e) (f) (g) O (h)
O
H
O O
Prop-2-enal But-3-en-2-one Dimethylbut-2-endioic acid Hexa-3E-en-2,5-dione
(Acrolein) (Methyl vinyl ketone) (Dimethyl fumarate)
O
O
(i) (j) (k)

1,5-Diphenylpen-1tene 2-Methylhex-1-en-3-one Penta-1,4-dien-3-one

O
O O
(l) (m) (n) (o) H
OH
OH
O
Cyclopent-3-en-1-one Propenoic acid Cyclohexene-1-carboxylic acid Oct-4Z-enal
(Acrylic acid)

49. Lithium aluminum hydride coordinates to the carbonyl oxygen, and can deliver hydride to the C=C
unit by an intramolecular reaction, but it can also deliver hydride to the carbonyl carbon. The reason
for reactivity at both sites is bond polarization of the C=O, which extends through the C=C unit by
vinylogy, leading to a δ+ on the carbonyl carbon and also the terminal carbon of the C=C unit.
H H
δ–
H Al H
δ+
Li
O
δ+ δ–

50. Briefly explain why an acid catalyst reacts with the carbonyl oxygen of methyl vinyl ketone. Draw
the resultant intermediate. The oxygen atom is more basic than the p-bond of an alkene. This is clear
when examining the oxocarbenium ion via protonation of the carbonyl, which is resonance stabilized as
shown. Protonation of the C=C unit will generate the unstable carbocations shown, which are not
resonance stabilized
H H H O
O O O O
or
51. The allylic cation intermediate is more stable, and the conjugated ketone is more stable than the
nonconjugated ketone.
OMe OMe
Na, NH O
3
H3O+
EtOH
heat

OMe Me Me Me
O O O
H+ 1,2 H-shift H2O H2O
to more stable
allklic cation

Me H Me OH O
O O
-H+ HO H+ HO - MeOH -H+

52.
(a) 345 nm = 82.87 kcal; = 2.9x104 cm-1 (b) 16x102 cm-1 =19.14 kJ; = 6.25x103 nm
-1 3 4
(c) 1765 cm = 5.67x10 nm; = 5.05 kcal; = 5.67x10 Å (d) 325 kJ =5.05 kcal; = 5.67x103 nm
(e) 8000 Å =35.7 kcal; = 1.25x104 cm-1 (f) 185 kcal = 154.5 nm; = 1545 Å; = 6.47x104 cm-1

Io
53. If ε = 38,000 for c = 0.5 g mL-1 (10 dm path length), calculate log I
=εxcxd

= (38,000)(0.5x10) = 190000

Io
54. If I
= c = 1.2 g mL-1 and the path length is 5 dm, calculate ε.
I
ε = log ( Io )/cd = (4.77x10–9)/(1.2)(5) = 7.95x10–10

55.
O O O
(a) (b) (c) (d)
CHO
Methyl vinyl ketone Hexan-3-one Benzaldehyde Cyclohex-2-en-1-one
Weak-nonconjugated Strong-conjugated Strong-conjugated
O O O
(a) (b) (c) (d)
CHO
But-3-en-2-one Hexan-3-one Benzaldehyde Cyclohex-2-en-1-one
(Methyl vinyl ketone)
Strong-conjugated Weak-non-conjugated Strong-conjugated Strong-conjugated
O
(e) (f) (g)
O
Hepta-1,5-diene Cyclohexa-1,3-diene Ethyl hex-2-enoate
Weak-non-conjugated Strong-conjugated Strong-conjugated

OH
(h) (i)

Styrene Hex-3-en-2-ol
Strong-conjugated Weak-non-conjugated

56. The predicted maximum UV absorption peak for each is shown using the data in Section 23.3.D.
O H
O O (c)
(a) (b)
(d)
H

215+12+12 = 239 nm 210 + 30+12 = 252 nm


214+5+5+5 = 229 nm 210+10+12 = 232 nm
H
C
(e) (f) (g)
C

214+30+30+30 = 304 nm
253+5+5 = 263 nm
214+5+30 = 249 nm
(no value is given for C≡C,
so estimate with C=C value).
57. The major product or products are shown. No mechanisms are provided.

HCl , -80 °C
(a)

I Cl
(b) I2 , 60 °C

I Br

HBr , 40 °C
(c)

cat. HCl HO
(d) aqueous acetone

HCl , 50 °C
(e) Cl

OH
HCl , 50 °C
(f)

Cl
HBr , -90 °C
(g)

Br

58.

HBr
Br
-80 °C
59. Reaction with HCl at the acyl carbon is reversible, and formation of the carbocation next to the δ+
carbonyl carbon is energetically unfavorable. The product will be the chloride shown. Reaction with
diatomic bromine give a bromonium ion, which may be difficult to form due to the proximity of the +
and δ+, but if it forms, the bromide ion will attack the bromonium ion to give the dibromide.
δ+ δ+

Cl O O O O
Further from δ+, so Two proximal positive charges
Br this is lower in energy are very destabilizing
Br
δ+
Br O Br–
O

60. The major product or products are shown. No mechanisms are provided.
1. PhMgBr , THF
2. Hydrolysis Ph
(a) CHO

OH
O
Me OH Me O
(b) 1. MeMgBr , ether

2. Hydrolysis
Ph minor
Ph Ph
O 1. Li OH
ether
(c) Bu
2. Hydrolysis

CHO 1. Ph2CuLi , THF , -10 °C


CHO
(d)
2. Hydrolysis
O OH Ph
1. MeMgBr , ether Me
(e) 2. Hydrolysis

O
O
t-BuOOH
(f)
NaOH
O
61.
O
O
O

(a) OH
O

O OH
O

(b)

O
HO
O O HO
(c) O

62. The major product or products for each reaction sequence are shown.
Ph2CuLi
(a) No reaction (N.R.)
Br O
O Br2 , -20 °C
(b)

Br
NaBH4
(c) O
OH
H O O
H2 , Pd
(d)
O O
O LiAlH4 OH OH
(e) +

LiAlH4
(f) No reaction (N.R.)

H2 , Pd
(g)
O NaBH4 OH
(h)

O
1. Ph3P=CH2
(i)
2. H2 , Pd

63.
O
O
HO
O

O
O O HO
O
HO

64. In a sense, this is a trick question. The only UV active compound is the starting material. Neither
the 1,2- nor the 1,4-byproduct have a conjugated chromophore that will yield a significant absorption in
the UV. The 1H NMR is a far better probe to distinguish these products.

Me O O HO Me

1,4- 1,2-

65. In both cases, the major rotamer is the s-trans because it will have the lowest energy when
compared to the s-cis rotamer.
(E)
Ph
Ph

(Z)
Synthesis Problems. Do not attempt these problems until Chapter 25 has been read and
understood.

66. Retrosyntheses are shown. In consultation with your instructor, suggest a synthesis based on the
retrosynthesis.

O
O
(a)
Br OH
NH2 Ph CN Ph CO2H
Ph
(b)

Ph Ph Ph
OH
O
Cl OH

O O

67. Show a synthesis for each of the following molecules from the designated starting material:
1. Mg , THF
2. cyclohexanone
Br 3. PBr3
(a)
4. KOH , EtOH
1. PCC
OH 2. Ph3P=CMe2
(b)

O
1. succinic acid,
H2SO 4
(c)
2. SOCl2
3. AlCl3

1. LDA , THF , -78 °C O


O O
(d) 2. PhCH2Br 3. H3O+

OEt 4. SOCl2 5. benzene , AlCl3 Ph


Ph
1.NaOEt , EtOH , reflux
O Ph
Ph
O
(e) O
2. H3O+
3. H2 , Pd-C

Ph 1. EtOH , H+
2. NaH , THF Ph Ph
(f)
Ph 3. 4-bromo-1-butene HO2C
CO2H 4. aq NaOH; aq H+

Spectroscopic problems. Do not attempt these problems until Chapter 14 has been read and
understood

68.
O O
Conjugated and UV active Non-conjugated, so no
strong UV peaks
Pent-3-en-2-one
Pent-4-en-2-one

2 Alkene H + methyl
on C=C (doublet)
3 Alkene H + no methyl
on C=C
8 6 4 2 0
PPM
6 4 2
PPM

200 100 0
PPM 200 100 0
Two methyl signals PPM
One methyl and one methylene
no methylene

69. Hexa-1,3-diene is conjugated and will be UV active, whereas hexa-1,5-diene is nonconjugated and
will show no significant UV activity. The conjugated diene will react with bromine to give 1,2 and 1,4
products, whereas the nonconjugated diene will show that each alkene reacts independently.

70. Pentan-2-one is not conjugated and will not show bands in the UV, whereas pent-3-en-2-one is
conjugated and will show a strong band in the UV. The carbonyl for the nonconjugated ketone will be
at ~ 1725 cm-1 in the IR, whereas the conjugated carbonyl will be at ~ 1695 cm-1. The 1H and 13C NMR
for the conjugated ketone will show protons and carbon in the C=C region, and the nonconjugated
ketone will not.
71.
Br Br
Both are not conjugated
Br so there is no UV.
IR is the same, with a
C=C peak at about
1650 cm –1
Br

6 4 2 0
PPM
6 4 2 0
PPM
more symmetrical

trans-Penta-1,3-diene
72.

2,5-Dimethylhexa-2,4-diene

73.

O
trans-3-Hepten-2-one
74.

O
Ethyl methacrylate
O
75.

H
trans-Pent-2-enal
76. O
Chapter 24
Solutions Manual
Organic Chemistry. An Acid-Base Approach 2nd Edition
24. A diene must be conjugated and able to achieve a s-cis (cisoid) conformation.

Locked in s-trans so Not conjugated, so


no Diels-Alder no Diels-Alder

25. The circled alkene has two electron withdrawing groups, which lowers the LUMO of the C=C more
than any other choice, so the activation energy for the Diels-Alder reaction will be lower and it will
proceed faster and under milder conditions.

CO2Et
CO2Et OMe
EtO2C

26. The circled alkene has an electron releasing group, which raises the LUMO energy of the C=C more
than any other choice, so the activation energy for the Diels-Alder reaction will be higher and it will be
slower, requiring harsher reaction conditions.

CO2Et
CO2Et OMe
EtO2C

27. This rearrangement occurs via attack of the π-bond of one alkene unit at the terminal carbon of the
other C=C unit. It is clear in B that both the attacking and recipient carbon are much more sterically
hindered than in A. In addition, steric repulsion of the methyl groups inhibits close approach the two
carbons. Both of these phenomena work to make the rearrangement of B much slower than A.

O O

A B
28. When heated, the initial Diels–Alder product, which now has another C=C unit, can form a
substituted furan derivative by loss of ethene, as shown. This reaction is known as a “retro-Diels-Alder”
reaction and occurs at a higher temperature than what is usually required for the initial Diels–Alder
reaction.
CO2Me
CO2Me
CO2Me

CO2Me
CO2Me CO2Me
H2 , Pd

CO2Me CO2Me

CO2Me CO2Me

29. Cyclopentadiene is locked into a s-cis conformation since it is part of a five-membered ring.
Confining the π-bonds to the ring also makes the π-electrons more available for donation, and this
combination makes cyclopentadiene much more reactive.

30. Molecule A reacts faster because the Cope rearrangement relieves the strain of the three-membered
ring, which is lacking in the product. Relief of strain accelerates the rate of the reaction relative to the
reaction of B, where there is no strain.

A B

31. Give the major product for each of the following reactions.
O O

heat O
(a) + O O
O
O O O

CHO heat CHO


(b) +

CO2Et CO2Et CO2Et


heat
(c) +
EtO2C CO2Et
CO2Et
heat CN
(d) CN
+

CO2Et heat CO2Et


(e) +

32.
O O OH
heat keto-enol tautomerism

A [3,3]-sigmatropic
rearrangement, a Claisen Tautomerism favors the phenol,
rearrangement driven by formation of the aromatic ring

33. Addition of BF3 to the Diels–Alder reaction of acrolein and buta-1,3-diene leads to coordination of
the Lewis acid with the aldehyde, which withdraws more electron density from the C=C unit, and lowers
the LUMO energy even more. This lowering of the LUMO energy leads to a smaller energy difference
between the LUMOalkene and HOMOdiene, which is a lower activation energy and leads to a faster
reaction.

34.

(a) (b) (c) (d)

Locked in s-trans Reacts as an alkene, Too sterically hindered, Reacts as an alkene,


so it cannot undergo a but a Diels-Alder reaction DielsAlder is possible, but a Diels-Alder reaction
Diels-Alder reaction is very slow and requires but it will be slow. is very slow and requirse high
high temperatures temperatures

35. There are two electron-releasing alkoxy groups that raise the LUMO of the alkene to a level that the
activation energy is simply too high to occur at low temperatures.
Me OEt

EtO Me
36. The major product for each is shown.

heat O
(a)
O

(b) heat

heat
(c)
O

O O

37. The major product for each reaction is shown.


OMe OMe
ethyl acrylate
(a) CO2Et
heat

Me CN
acrylonitrile
(b)
heat

Ph Ph O

maleic anhydride
(c) O
heat

O
Ph Ph
Me Me
diethyl fumarate CO2Et
(d)
heat
Me Me CO2Et
O

(e) methyl vinyl ketone


heat
Me

CO2Me
methyl acrylate
(f)
heat

Me

38.
disrotatory conrotatory

CO2Et CO2Et
CO2Et
(a)

CO2Et CO2Et
CO2Et
(b)

Ph
CO2Et
CO2Et CO2Et
(c)
CO2Et CO2Et CO2Et

CO2Et CO2Et
CO2Et
(d)
CO2Et EtO2C CO2Et
Ph Ph Ph

(E)
CO2Et CO2Et
CO2Et
(d)

CO2Et (E) EtO2C CO2Et


Ph Ph Ph
39. The Diels–Alder reaction proceeds via an endo transition state, with a disrotatory motion of the
methyl groups, so the circled diastereomer will be the major product. Note that all of the diastereomers
are racemic, which accounts for only eight of the total 24 = 16 stereoisomers. The remainder of the
possible stereoisomers have a trans-relationship for the diester, which is impossible since the cis-
stereochemistry of diethyl maleate is retained.

CO2Et CO2Et

CO2Et CO2Et

CO2Et
All are racemic, so each has an enantiomer

CO2Et CO2Et CO2Et

CO2Et CO2Et

40. There are two carbonyl units, so there is greater secondary orbital interactions for the diester relative
to the mono-ester.

41. The major product is shown for each reaction.


H
O heat
(a) O

heat
(b)

OH OH O
heat
(c)
H

O
heat
(d)
O
42. The diene on the right has a phenyl substituent on each C=C unit. This means that the Cope
rearrangement generates a new diene with more substituted alkene units, which are more stable than the
monosubstituted alkene units of the diene on the left. Since this is an equilibrium process, the
equilibrium will favor formation of the more stable diene, which is on the right.
Ph Ph

Ph Ph

OSiMe3 OSiMe3 OH

43. 1. LDA, THF, -78 °C H3O+


O 2. Me3SiCl O O
3. heat

H H
OH O
heat
44. OH OH

H H

45. Cyclopentadiene reacts with another molecule of cyclopentadiene at low temperature, so the Diels-
Alder product shown is what is in a bottle of commercial cyclopentadiene. If this dimeric compound is
heated, a retro Diels-Alder reaction occurs to regenerate 2 equivalents of cyclopentadiene. If kept cold,
one can use the cyclopentadiene in other reactions.
35 °C

>130 °C
\

Synthesis Problems. Do not attempt these problems until Chapter 25 is read and understood.

46. A synthetic scheme is provided for each problem.


1. methyl vinyl ketone
2. H2 , Pd-C
(a)
3. Ph3P=CMe2

1. diethyl fumerate
(b) 2. O3 , -78 °C CO2Et
HO2C
3. H2O2 HO2C
4. excess SOCl2 CO2Et
5. excess EtOH , heat
1. CH2=CHCN CN
2. O3 , -78 °C OHC
(c)
3. Me2S OHC
1. methyl acrylate OH
2. excess PhMgBr
3. O3 , -78 °C HO Ph
(d)
4. Me2S Ph
5. excess NaBH4
HO
1. heat
2. Ph3P=CHPh
(e) CH=CH2Ph
OH
NMe2
1. LDA , THF , -78 °C
O 2. Me3SiCl O
(f)
O 3. heat 4. H3O+
5. SOCl2 6. Me2NH O
7. MeCO3H

Spectroscopic problems. Do not attempt these problems until Chapter 14 is read and understood.

47.
H
IR: Alkene signal at IR: Alkene signal at
O
1645 cm-1 O 1645 cm-1 and the
aldehyde signals at 1725 cm-1
and 2815 cm-1

6 4 2 0
PPM 10 5 0
PPM

150 100 50 0
PPM 200 100 0
Carbons and protons for two C=C PPM
Aldehyde proton and carbonyl carbon, and
only one C=C unit

48. The diene on the right is more stable, and it will be the major product of this equilibrium reaction.
The main difference is the two methyl
groups on C=C versus two methyl groups
on sp3 carbon. One has six C=C protons
whereas the other has only four.

6 4 2 0 6 4 2 0
PPM PPM

1.88; 2.12

3.58
6.07 6.75
Bicyclo[2.2.1]hepta-2,5-diene
49. (2,5-Norbornadiene)
6.07 3.58 6.75

O
5.78
4.42 1.98;1.73
50. 7-Oxabicyclo[2.2.1]hept-2-ene
5.78 4.42
1.98;1.73

5-Norbornene-2,3-dicarboxylic anhydride
51. O

6.45
4.05; 4.20
O 3-(Vinyloxy)prop-1-ene
52. (Allyl vinyl ether)
5.20; 5.32
4.20
5.90

CN

53.
Cyclohex-3-enecarbonitrile
Chapter 25
Solutions Manual
Organic Chemistry. An Acid-Base Approach 2nd Edition
13.
d d

(a)
O O
O
CO2Et
a O
(b) O
OH

O OH O
(c)
a H

O O O
d
(d) H a

14.
acceptor
O
O
Br (b)
(a) (c)
donor acceptor

Br
acceptor (e)
donor
(f) acceptor
(d)
MgBr O
15. In all cases, the hydrolysis step for hydride reduction, Grignard reactions, or enolate anion reactions
is omitted, but it is part of the synthesis and understood to be there.
H
(a)
O O OH
1. BuMgBr
2. PCC PCC

(b) Br
NH2
1. Mg 2. CH3CHO 1. BH3 , ether 2. NaOH, H2O2
3. PBr3 4. NaN3 5. LiAlH4 3. PBr3

(c) O
OH

1. LDA , THF , -78 °C 1. Hg(OAc)2, H2O 2. NaBH4


2. MeI 3. NaBH4 3. PCC

(d) O
OH

1. LDA , THF , -78 °C 1. O3 , -78 °C 2. H2O2


2. MeI 3. NaBH4 3. EtOH , H+ 4. NaOEt , EtOH , reflux
5. H3O+ 6. 200 °C

(e)
OH
OH aq. OsO4 1. Hg(OAc)2 , H2O
OH
t-BuOOH 2. NaBH4

(f)
Br
H2 , Pd-BaSO4 1. Br2 , CCl4 1. PPh3 2. BuLi
quinoline 2. excess KOH , EtOH , heat 3. CH3CH2CHO

16. In all cases, the hydrolysis step for hydride reduction, Grignard reactions, or enolate anion reactions
is omitted, but it is part of the synthesis and understood to be there.
OH O OH
(a)

Me2CHMgBr 1. PCC 2. CH3CH2CH2CHO


3. PCC
OH Ph O
Ph O
(b)

CH2=CHCH2MgBr 1. PhMgBr 2. PBr3 1. Hg(OAc)2 , H2O


3. KOH , EtOH 2. NaBH4 3. PCC
4. MeCO3H

(c) CO2Et
O
O
1. LiAlH4 2. PCC
1. aq. OsO4 , t-BuOOH 3. Ph3P=CEt2
2. acetone , H+

CN
(d)
N
H
1. LDA , THF , –78°C 2. MeI
Ph 3. LiAlH4 4. PhCH2Br
Ph Ph
(e)
O
Br
1. Ph3P=CHCH3 2. BH3 , ether
3. H2O2 , NaOH 4. PBr3

(f)
CO2H

1. O3 , -78 °C 2. Me2S 3. (CH2CH2CO2Et/LDA) (enolate addition)


4. H3O+ (elimination and hydrolsyis of ester) 5. H2 , Pd

(f)
CO2H

1. O3 , -78 °C 2. Me2S
3. (CH2CH2CO2Et/LDA) (Enolate addition)
4. H3O+ (Elimination and hydrolysis of ester)
5. H2 , Pd
17.
1. PCC
2. Gringard of 2-bromobutane
(a) OH
3. PCC
O

(b) CN 1. Gringard of 2-bromobutane


2. H3O+
1. BH3 , ether 2. NaOH , H2O2 O
3. PCC
(c)
4. Gringard of 2-bromobutane
5. PCC
O
1. Me3CO3H
2. MeMgBr 3. PBr3
(d)
4. KOH , EtOH 5. BH3 , ether
6. H2O2 , NaOH 7. PCC O

1. PCC
2. LDA , THF , -78 °C
(e)
3. EtI
OH 4. separate isomeric products
from alkylation of the other O
side of the carbonyl

1. CH3CH2CHO
(f)
OHC 2. PCC
O

18.
1. LDA , THF , -78 °C O
CHO 2. 1-bromobutane Ph3P=CHCH3 MeCO3H
(a) CHO

1. PBr3
MeMgBr 2. MeC≡C: –Na+
(b) OH
CHO

H2 , Pd-C MeCO3H
O

1. LDA , THF , -78 °C


O 2. CH3CH2Br MeCO3H O
(c) Br (MeCH=CH)2CuLi
3. NaBH4
4. PBr3
19.
1. CrO3 , aq H2SO 4 , heat
2. EtOH , H+
(a) OH
3. LDA, THF , -78 °C CO2H
4. MeI 5. H3O+

1. LDA , THF , -78 °C


2. BuI
CO2Et
(b) 3. H3O+ CO2H

Br 1. NaCN , DMF
(c)
2. H3O+ , heat CO2H
1. O3 , -78 °C
2. H2O2 3. EtOH , H3O+
(d)
4. LDA, THF , -78 °C CO2H
5. MeI 6. H3O+

20. Cyclohexanol is converted to bromocyclohexane with PBr3 and then to cyclohexene with KOH in
ethanol. Ozonolysis of the alkene, with hydrogen peroxide as the second step gives the dicarboxylic
acid. Esterification with ethanol and acid yields the diester, and treatment with sodium ethoxide yields
the ethyl ester of cyclopentanone 2-carboxylic acid via Dieckmann condensation. Hydrolysis to the acid
and heating to 250 °C leads to decarboxylation and formation of cyclopentanone.
OH Br
KOH 1. O3 CO2H EtOH
PBr3 EtOH 2. H2O2 H+

CO2H
CO2Et NaOEt CO2Et CO2H
EtOH H3 O+ 250 °C O
O O
reflux
CO2Et

21.

HgSO4
H2 , Pd aq Hg(OAc)2 NaBH4

O OH
1. BH3 O O
CrO3 , a. H2SO 4
2. H2O2 , NaOH

H OH
22.
Sn , HCl H3O+ , heat
CHO CN CO2H
1. MeMgBr

2, H3O+ , heat
O

23.
PCC

1. MeMgBr OH
2. H3O+
CN 1. O3 , -78 °C
heat
O
2. Me2S

24.
H2 , Pd
1. PBr3
2. KOH , EtOH
OH
Ph3P=CHCH3

CH3CHO

25.
O OH
LiAlH4

NaBH4 LiAlH4
CHO CO2Et
OH
Also reduction 1. BH3 , ether Also reduction of
of ketones 2. H2O2 , NaOH acids or acid chlorides
Chapter 26
Solutions Manual
Organic Chemistry. An Acid-Base Approach 2nd Edition

31.
O

Me Me Cl Cl Cl Me
(a) N (b) HN (c) N (d) N
N N

O
N,3-Dimethylpyrrole 3,4-Diacetylpyrrole 2,4-Dichloropyrazole 1-Methyl-4-chloroimidazole
N
NH2 O2N N Br Br N NH2
N
(e) (f) (g) (h) (i)
N N
N N N
2,4,6-Trimethylpyridine 5-Aminopyrimidine 3-Nitropyrazine 3,5-Dibromopyridazine 2-Amino-5-methyl-
pyrimidine

32.
N
O Cl
S N-Et
(a) (b)Br (c) (d) (e) Me
O H2N N NH2
O Me H2N Me
1-(Furan-2-yl)- 3-Bromo-2-methyl- 5-Chloro-4-isopropyl- Pyridine-2,6-diamine 1-Ethyl-4,5-dimethyl-
propan-1-one thiophene furan-3-amine 1 H-imidazole
NH2
OH O2N Et
N
(f) (g) (h) (i) N
H3CH2CH2C N O2N NH O CHMe2 N
NO2
2-Propylpyrimidin-5-ol 3,4-Dinitro-1H-pyrrole 3,5-Diethyl-2- 6-Nitropyrazin-2-amine
isopropyl-2H-pyran

33. The major product is shown for each reaction.


NO2 H H
CH3 N CH3 Cl N CH3
(a) N (b)
CH3
N
N N
CH3 CH3
(c) O2N (d)
N N O
N N O
CH3 CH3 OH
O2N O2N
(e) SO3H (f) NH2
O O O O
HO
(g) (h) SO2H
O O O O

Ph NO2
HN
(i) N
N
N

Ph H NMe2
N N
34.
(a) (b) (c)
N O N
N

35. Molecule B should react faster. Reaction with A generates an arenium type ion in which the
positive charge will be delocalized on all three nitrogen atoms, which is very destabilizing. Reaction
with B generate one resonance contributor with the positive charge on nitrogen, but two where it is
delocalized on carbon, so the intermediate for B is more stable. While not great, it is better than A.
* H *
N N N N* N N
N N
A * Br B Br
N N * *
H N N

36. In both cases, nucleophilic substitution at C3 will generate a negative charge in the ring that will be
delocalized on either oxygen (bromofuran) or nitrogen (bromopyrrole). The charge density is larger on
oxygen than on nitrogen, so an intermediate with negative charge should lead to greater instability for
the oxygen-containing furan ring when compared to the nitrogen-containing pyrrole ring. Based on this
analysis, 3-bromopyrrole should be more reactive.
37.
C4H9
Me Me
N O N S
Et
(a) (b) (c) (d)
N O Et N NO2 S
Me H
1,3,4-Trimethy- 2,2-Diethyl- 1-Butyl-3- 2,2,6,6-Tetramethyl-
limidazolidine 1,3-dioxolane nitropiperazine 1,4-dithiane
Br Br
O Cl O
O O
(e) N Ac (f) (g) Br (h)
N
Cl O Br O
Ac
1,1'-(3,4-dibromopyra- 2,5-Dichloro-1,4-dioxane 4,5-Dibromo- 2,2-Diethyl-
zolidine-1,2-diyl)- 1,3-dioxolane 1,3-dioxane
bis(ethan-1-one)
H NO2
H2N
N N
(i) S (j) (k) (l)
N N NH
H H N
S H
2-Ethyl-4-isopropyl-5- 2-Isopropylhexahydro- 2-(tert-Butyl)-1- 3,3-Dimethyl-4-nitrohexa-
methyl-1,3-dithiolane pyrimidin-5-amine propylimidazolidine hydropyridazine

38.
O Me Me O
N Br Br
(a) N (b) (c) (d)
N
N O
Me Me O
Me
N-Acetyl-3-ethyl- 1,2,4,5-Tetramethylpiperazine 3,5-Dibromotetra- trans-2,3-Dimethyl-
imidazoline hydrofuran 1,4-dioxane
O2N NO2
O2N O
(e) N (f) (g) N (h)
NH N
H
1-Ethyl-4-nitropyrazolidine cis-3,5-Dinitropiperidine N-Propylaziridine 2,3-Diethyloxirane
Ph
Cl S
N S O
(i) (j) (k) (l)
Ph
Ph
N,2-Diethylazetidine 3-Chlorothiane 2-Phenylthiirane cis-2,3-Diphenyloxetane
39.

NH O NH NH
(a) (b) (c) (d)
N
1,2,3,4-Tetrahydropyridine 2,3-Dihydrofuran 1,2-Dihydropyridine 1,2-Dihydropyrimidine

40.

N
O2N H
N N
(a) (b) (c)
Br
Me
Br
4,6-Dibromoquinoline
6-Methyl-7-nitroisoquinoline 6-eEthylindole

N Cl CN Me
N N
(d) (e) (f)

NO2
3-Butyl-5-nitroisoquinoline 8-Chloroquinoline 1-Methyl-7-cyanoindole

41. The major product or products are shown.

Br

Br2 , AlCl3
(a) +
N
N
Br N
OMe
OMe OMe
Br2 , AlCl3 Br
(b)
+
N
N N
Br
O

acetyl chloride N
(c) +
N AlCl3
N
O
NO2

HNO3 +
(d) N N
H2SO 4 NO2 N

NO2
HNO3
(e)
N H2SO 4
H N
H
Me Me
Br2 , AlCl3
(f)
Br
N N
H H

42. Pyrrole is the more reactive ring in this system, and will react with Br+.
HN O
HN O

Br

43. Electrophilic substitution at C3 generates an intermediate such as the one shown. The charge is
placed on C2, which means that an oxocarbenium ion resonance contributor will be formed. No other
resonance contributors are possible, whereas attack at C2 generates more resonance contributors, has
greater stability, and will be the lower energy intermediate that leads to the major product.
H H
X+ X X

O
O O

S
44.
N
H

45. It is a four-π-electron system, confined to a four-membered ring, which does not follow
Hückel’s rule. It is anti-aromatic, and very unstable, analogous to cyclobutadiene. N
46. A Friedel-Crafts reaction would require reaction at a pyridine ring, which does not undergo
electrophilic aromatic substitution reactions such as this very well. The pyridine ring is deactivated and
simply reacts too slowly with an oxocarbenium ion to give the Friedel-Crafts reaction.
O

HO N

47. 3-Bromopyridine reacts with sodium amide, the products are 3-amino pyridine, along with 2-amino
pyridine and 4-amino pyridine. This reaction proceeds via a benzyne intermediate.

48. The product is 2-hydroxypyridine, which also exists in the lactam form shown, which is known as
2-pyridone. Remember that amides can exist in the imine form (amide I and amide II bands in the IR),
and this equilibrium is a form of tautomerism known as hydroxypyridine–pyridone tautomerism

N OH N O
H

Spectroscopic problems. Do not attempt these problems until Chapter 14 is read and understood.
All of the following problems involve heterocycles.

49. There is a difference in the aromatic proton region. The presence of the nitrogen pushes the ortho
proton further downfield relative to toluene, and there are fewer identical signals.
CH3

N CH3

8 6 4 2 0 8 6 4 2 0
PPM PPM
50. There are few differences in the 1H NMR for 2-methylquinoline from 4-methylquinoline, although
there will be some differences in coupling constants (not observable in the spectra shown). In the 13C
spectra, note that the methyl group of 2-methylquinoline is a bit further downfield due to its proximity to
the nitrogen.
CH3

N CH3
N

8 6 4 2 0
PPM 8 6 4 2 0
PPM

150 100 50 0 150 100 50 0


PPM PPM

51. Oxygen is more effective at deshielding relative to nitrogen, because it is more electronegative, and
more electron withdrawing. Therefore, the aromatic protons in the 1H NMR are further downfield in
furan. Likewise, the aromatic carbon atoms are further downfield in the 13C of furan when compared to
those carbon atoms in pyrrole.

O N
H

8 6 4 2 0 6 4 2 0
PPM PPM

150 100 50 0 100 50 0


PPM PPM
52. Pyrazine is symmetrical, and all protons are identical, and there is only one peak in the 1H NMR.
Pyrimidine has three different protons and they clearly show in the 1H NMR.
N N

N
N
pyrazine pyrimidine

8 6 4 2 0 10 5 0
PPM PPM

N
2,5-Dimethylpyridine
53.

O
2-Ethylfuran
54.

O
N+
-O 3-Nitropyrrole

55. NH

N
4-Pyridinemethanol
OH
56.
Chapter 27
Solutions Manual
Organic Chemistry. An Acid-Base Approach 2nd Edition
40. Triethylamine is a base, and in the presence of a secondary amine and a protic solvent an E2 reaction
can occur to yield the alkene.
Br NEt3

41.
H
H N NEt2
(a) (b) (c) (d)
N N
Ph Ph
Ph
N-Ethylpropan-1-amine 3-Phenylazetidine N, N-Diethyl-1,3-diphenyl- 3-Methyl-1-propyl-
pentan-1-amine pyrrolidine
H
Me NH2 N
Me H Me
(e) N (f) (g) (h) N

(2R,3 R)-2,3-Dimethyl- 2-Methylhexan-3-amine N,3,6-Trimethyl- N, N-Diethylhexan-3-amine


piperidine decan-4-amine
Ph
Et N
(i) N Et (j) N (k)

1,2-Diethylaziridine 1-Benzyl-1,2,3,4,7,8- Triethylamine


hexahydroazocine

42. The major product or products are shown. No mechanisms are provided.
1. PBr3
2. KCN , THF
(a) OH 3. LiAlH4 , THF CH2NH2
4. hydrolysis
1. PCC , CH2Cl2 NHCH2Ph
OH 2. PhCH2NH2 , cat H+
(b) 3. H2 , Pd-C
1. NaN3 , THF NH2
Br 2. LiAlH4 , THF
(c)
3. hydrolysis (R)
(S)
1. HNO3 , H2SO 4 NH2
2. H2 , Pd-C
(d)

EtNH2 , Zn° , HCl


(e)

O NHEt
NHMe MeI
NMe2

(f)

1. NH3 ; 200 °C
2. LiAlH4 , THF
CO2H CH2NH2
(g) 3. hydrolysis

43. Phenyl groups are electron withdrawing relative to the propyl groups, which are electron releasing.
If the phenyl groups withdraw electron density from the N—H bond, diphenylamine will have a more
polarized N—H unit and will be more acidic.

44. The conjugate acid of triethylamine is Et3NH+ and the conjugate acid of ethylamine is EtNH3+. In
general, a primary amine is a stronger base in solution relative to a tertiary amine. If ethylamine is the
stronger base, then the conjugate acid will be a weaker acid when compared to the conjugate acid of the
weaker base, triethylamine.

45. Give the major product for each of the following reactions:

Pentan-2-one, cat H+ N
(a) N H

Br 1. Me3N
2. Ag2O , H2O
(b) 3. 200 °C

EtNH2 , cat H+
CHO CH=NEt
(c)
1. BuLi , THF , -78 °C O OH
2. Butan-2-one
3. Benzaldehyde
(d) Ph
N 4. Hydrolysis
H
1. NaBH4 , EtOH
O 2. aq NH4Cl
(e) 3. PCl3 , Pyridine

O Ph
3-Phenylbutanoic acid
N
(f)
N 25 °C H O
H
H

Et3NH+ Br-
(g) O- Na+ OH + Et3N

46. A triethylammonium salt has β-hydrogen atoms on the ethyl groups, and the least substituted alkene
possible from such a salt is ethylene.
-OH
H
N
47.
O O O
NH2 Ph O
(a) (b) OH (c) OH (d) OH
OH N N
NH2 H
Ph Me
3,4-Diphenyl-5-amino- 4-Aminohex-5-enoic acid Aziridine-2-carboxylic acid N-Methylpiperidine-
hexanoic acid 4-carboxylic acid
O OH O Ph O
H2N O NH O
(e) OH (f) OH (g) (h) OH
OH NH2 HO OH NH2
3-Aminobenzoic acid (2R)-Amino-(3R,4S)- N-Ethyl-3-amino-1,5- (2S)-Amino-3-phenyl-
dihydroxyhexanoic acid pentanedioic acid propanoic acid

O O

(i) (j) OH
OH
NH HN
Me
Pyrrolidine-(2S)- N,3-Dimethyl-(2S)-
carboxylic acid aminobutanoic acid

48. (a) Compound A should have the lowest pKa due to the electron-withdrawing effects of the OMe
group.
NH2 NH2

MeOCO2H CO2H
A B
(b) The second pK2 value is for loss of a proton from the ammonium salt of the amino acid. The
N-phenyl group is electron withdrawing, which will make the N-phenylammonium salt more acidic.
The more acidic compounds will have the larger pK2, so the answer is C.
Ph Me
N N
H H

C CO2H D CO2H
49.
NH3 NH3 NH3
NH3 NH3 NH3
(R) (R) (R)
(R) (R) Ph
CO2 CO2 CO2
CO2 CO2 CO2
H3C
NH3 NH3 HO NH3
NH3 SMe NH3
(R) (R) (R)
Me (R) (R)
CO2 CO2 CO2
CO2 CO2
OH OH SH

NH2 N NH
H2N NH2
NH3 NH3 NH3
NH3 (R)
(R) (R) (R)
(R) HN HN CO2
CO2 HN CO2 CO2
CO2

50. The phenyl group is electron withdrawing whereas the alkyl group in isoleucine is electron
releasing. The electron withdrawing inductive effects of the phenyl group lead to a more acidic
compound, so both pK values are smaller.

51. Acidification in an aqueous medium to pH 4 leads to hydrolysis of the ester to the acid, but it also
converts the alkaline group to an ammoniums salt, which is water-soluble. Therefore, the ammonium
salt will not likely be soluble in hexane.
Ph Br NH2 NH3+
PBr3 , Br2 NH3 1. aq NaOH
O O O OH
Ph Ph 2. pH 4 Ph
O O O O

52.
1. NaN3 2. H2 , Pd
3. H3O+
H2NCH2CO2H
1. Na phthalimide
2. hydrazine
H2NCH2CO2H BrCHCO2Et
NH3 , heat H2NCH2CO2H

53.
O O
(S) excess BuOH (S)
(a) OH OBu
H+
NH2 NH2
O
O
1. neutral pH (S)
(S) OH
(b) OH
2. propanoyl chloride HN
NH2
O O O
(S) 1. EtOH , H+
(c) (S)
(S) OH (S) OEt
2. Ac2O
NH2 NHAc
O 1. H3O+ OEt O
2. EtOH , cat H+
O
(S)
(d) OEt 3. succinic anhydride (S) N
NaOAc
NHAc
1. Br2 , PBr3
O O
O 2. H2O
3. EtOH , cat H+ Ph OEt
(e) Ph OH
O Br
(S)
1. LiAlH4 (S)
OH OH
(f) 2. aq H+; neutralize
NH2 NH2
O O
(S) 1. PhCOCl
(S)
(g) OH OH
2. dilute H2O/H+
NH2 NHCOPh
54.
1. PBr3 2. NaCN O
3. LDA , THF
Ph H2N CH C OH
(a)
OH 4. Br2 5. NaN3 Ph CH2
6. H2 , Pd
O
1. PBr3 , Br3 H2N CH C OH
(b) CO2H
2. NH3 H3C CH CH3

1. LiAlH4 2. PBr2 O
3. NaCN 4. H3O+
(c) CH3CO2H H2N CH C OH
5. PBr3 , Br2 6. NH3
CH3
O
1. EtOH , H+
2. NaH H2N CH C OH
3. 2-bromobutane
(d) HO2C CO2H CH CH3
4. H3O+ 5. 200 °C
6. PBr3 , Br2 7. NH3 H3C CH2

55.

CO2H O
O Ac2O
CO2 1. O
Ph Cl NaOAc
NH
+
NH3 2. aq H Ph N Ph
O O O
O
O pyridine H3 O+
(a) O
Ph O
Me2CHO
N N NH3
Ph
O O
O
O pyridine H3O+
(b) O O
Ph O
N N NH3
Ph
O
O O
O pyridine H3O+
(c) O
Ph PhCH2CHO Ph O
N Ph N NH3
Ph
56.
CO2
1. NaCN , DMF , heat (CH2)12
(a) HOCH2(CH2)10 CH2Br
2. CrO3 , aq. H2SO 4
4. H3O+ NH3
1. EtOH , H+ 2. LDA , THF , -78 °C Ph
C O2H CO2
3. 1-bromo-2-phenylbutane
(b)
4. H2 , Pd 5. H3O+ NH3
CN
1. O3 , -78 °C 2. H2O2
3. ethylene glycol , H+ H3N
4. SOCl2 5. EtOH CO2
(c)
6. LDA , THF , -78 °C
7. PhCH2Br 8. H3O+
9. NH3 , H2 , Pd-C 10. H3O+ Ph
NH3
1. O3 , -78 °C 2. H2O2
3. NaBH4 4. PBr3
(d) CO2
5. NaCN 6. H2 , Pd
7. H3O+

57.
O

O Ph H2N CH C OEt O O
O O
Cl O CH3 H
H2N CH C OH CbzHN CH C OH CbzHN CH C N CH C OEt
NEt3 DCC
H H H CH3

1. H3O+ O O O O O O
CbzHN H H 1. H3O+ H H
CH C N CH C N CH C OEt H2N CH C N CH C N CH C OH
O 2. H2 , Pd-C
2. CH CH3
H CH3 CH H CH3
H2N CH C OEt CH3
H3C
CH CH3
H3C CH3
, DCC

58. The electron pair on nitrogen is partly delocalized onto the adjacent carbonyl, which makes the
electrons less available for donation. In other words, it is less basic.

59. At neutral pH, an amino acid exists as a zwitterion, so the amine unit is not NH2, but rather NH3+.
60. In phe-ala, the PhCH2 group and the methyl group are probably anti in the best conformation.

61.
CO2H
ninhydrin
R RCHO + CO2 + 4 H2O
O NH2
O HO
O
+ glycine
(a) O N
R=H H H
O O
O
O
SH O HO
+ methionine O
(b) O N
R = CH2CH2SH
H O O
O
O
O HO
+ serine OH O
(c) O N
R CH2OH H
O O
O
O
O O HO
+ valine
(d) O N
R = Me2CH H
O O O
O
O O HO
N
(e) + histidine
O N
R = CH2-imidazole N H
H O O
O
O
H2N NH O HO
(f) + arginine O
O N
R= CH2C(=NH)NH2
H O O
O

62.
NO2
O O O O O
H H H H
H2N CH C N CH C N CH C N CH C N CH C OH 1. O2N F

(a) H CH3 CH2 CH2 CH CH3 2. 6N HCl


CH2 C O CH2
S NH2 CH3
CH3 NO2
O
O O O O
H H H H H2N CH C OH
O2N N CH C N CH C N CH C N CH C OH
H
CH3 CH2 CH2 CH CH3
CH2 C O CH2
S NH2 CH3
CH3
NO2
O O O O
H H H 1. O2N F
H2N CH C N CH C N CH C N CH C OH
CH2 CH2 CH CH3 CH2 2. 6N HCl
(b)
CH2 CH2 CH3 OH
C O C O NO2
O O O O
NH2 NH2 H H H
O2N N CH C N CH C N CH C OH H2N CH C OH

H2C CH CH3 H2C CH2

H2C CH3 OH CH2

C O C O
O NH2
OH NH2
C NO2
O CH
HN 1. O2N F
O O C CH2
H CH
H2N CH C N CH C N 2. 6N HCl
(c) H CH2
CH2 CH2 O O
N OH
H C H2N CH C OH
N NO2 O CH
HN CH2
NH HN O C CH2
CH
O2N HN CH C N N
H CH2
CH2 NH
N
H

HN

63.
NMe2
NMe2
O O O O
H H H
H2N CH C N CH C N CH C N CH C OH 1. O
CH2 CH2 CH OH H SO2Cl
HO
(a) SO2
CH2 CH3 2. 6 N HCl H N
HN N H
CH2
NH NH2
+ tyr + thr + gln
C NH OH
NH2
NMe2
NMe2
O O O O
H H H
H2N CH C N CH C N CH C N CH C OH 1.
CH2 CH2 CH2 HC OH SO2Cl
O
(b) HO
CH2 CH2 OH CH3 2. 6 N HCl SO2
HO2C N
C O O H + glu + ser + thr
OH OH
NMe2
NMe2

O O O
H H 1.
H2N CH C N CH C N CH C OH O
SO2Cl
CH2 CH CH3 CH2 HO SO2
2. 6 N HCl
(c) N + ile + lys
CH2 CH2 H
Ph
CH3 CH2
CH2
NH2

64.
O O O O
H H H O O O
H2N CH C N CH C N CH C N CH C OH H H
H2N CH C N CH C N CH C OH
CH2 CH2 CH OH H
1. N C S CH2 CH OH H
(a) CH2 CH3
CH3 S
CH2 2. CF3CO2H O
NH NH NH
C NH OH
OH H2N
NH2
O O O O O O O
H H H H H
H2N CH C N CH C N CH C N CH C OH H2N CH C N CH C N CH C OH
1. N C S
CH2 CH2 CH2 CH OH CH2 CH2 CH OH S
(b) CH2 CH2 OH CH3 CH2 OH CH3 O
2. CF3CO2H
NH
C O C O C O
OH OH OH
HO2C

O O O O O
H H H
H2N CH C N CH C N CH C OH HN CH C N CH C OH
CH2 CH CH3 CH2 1. N C S CH CH3 CH2
(c)
CH2 CH2 CH2 CH2 S
2. CF3CO2H
O
CH3 CH2 CH3 CH2
NH
H2C H2C
NH2 NH2 Ph
65.
O
O O O O O
H H H H2N CH C NHNH2 O
H2N CH C N CH C N CH C N CH C OH H2N CH C OH
CH2 H2N CH C NHNH2
CH2 CH2 CH OH H H
(a) CH2 CH2 O
CH2 CH3 NH2NH2
CH2
CH2 H2N CH C NHNH2
100°C NH
NH CH OH
C NH
C NH OH CH3
NH2 OH
NH2

O O O O O O
H H H
H2N CH C N CH C N CH C N CH C OH H2N CH C NHNH2 H2N CH C OH
CH2 CH2 CH2 CH OH CH2 O CH OH
(b) CH2 CH2 OH CH3 NH2NH2 CH2 H2N CH C NHNH2 CH3
O
C O C O 100°C C O CH2
H2N CH C NHNH2
OH OH OH CH2
CH2
C O
OH
OH
O O O
H H O O
H2N CH C N CH C N CH C OH
H2N CH C NHNH2 H2N CH C OH
CH2 CH CH3 CH2 O
(c) NH2NH2 CH2 CH2
CH2 CH2 H2N CH C NHNH2
100°C CH2
CH3 CH2 CH CH3
CH2
CH2 CH2
CH2
NH2 CH3
NH2
66.
O O
H O H2N CH CH2OH
H2N CH C N CH C OH
1. LiAlH4 H2N CH C NH2 CH CH3
(a) CH2 CH CH3
2. H3O+ CH2 CH2
OH CH2
OH CH3
CH3
O O
H O H2N CH CH2OH
H2N CH C N CH C OH 1. LiAlH4
(b) H2N CH C OH CH CH3
CH3 CH CH3 2. H3O+
CH3 CH3
CH3
O O
H O H2N CH CH2OH
H2N CH C N CH C OH
1. LiAlH4 H2N CH C OH CH2
(c) CH2 CH2
2. H3O+ CH2 CH CH3
SH CH CH3
SH CH3
CH3

67.
trypsin
(a) ala-ala-thr-cys-asn-val-phe-leu-thr-his-arg-pro-phe

ala-ala-thr-cys-asn-val-leu-leu-thr-his-arg pro-phe
chymotrypsin
ala-ala-thr-cys-asn-val-phe-leu-thr-his-arg-pro-phe

ala-ala-thr-cys-asn-val-phe leu-thr-his-arg-pro-phe

trypsin
(b) tyr-ile-ile-ile-arg-gln-asp-val-his-his-phe-ile-tyr
tyr-ile-ile-ile-arg gln-asp-val-his-his-phe-ile-tyr

chymotrypsin
tyr-ile-ile-ile-gly-gln-asp-val-his-his-phe-ile-tyr

tyr tyr-ile-ile-ile-gly-gln-asp-val-his-his-phe ile-tyr


68.
O
O
O OH
H2N (S) OH (S)
N (S) H2N (S)
N
H O
H
Ph
Ph
Anti Syn

69.
O O O O
H H H
H2N CH C N CH C N CH C N CH C OH
CH3 CH2 CH2 CH2
CH2 OH
C O
NH2
OH OH
O O O O
H H
H2N (S) N (S) H2N (S) N (R)
N (S) N (S) CO2H N (S) N (S) CO2H
H H H H
CH3 O CH3 O
70.
(4R)-Amino-(3S)-hydroxy-(5S)-methylheptanoic acid

OH O
Aplidine (S) (S)
(S) O (S) L-Alanine
(R)

O NH O O
HN
Me O
(S) (S)
L-Proline N (R) (S) O O
N (S) N O
L-Leucine
O H (S)
O N
O N (S)
D-Leucine
Me
S-proline
4-Amino-(3S)-methylbutanoic acid
L-4-Methoxy tyrosine
OMe

71. For gly-lys-ser-phe-phe-ala-ile-ile-trp-leu-asp-met-pro-arg-glu-tyr-ile-lys-arg, assume the N-


terminus is on the left (gly) and C-terminus is on the right (arg).
trypsin
(a) gly-lys-ser-phe-phe-ala-ile-ile-trp-leu-asp-met-pro-arg-glu-tyr-ile-lys-arg
gly-lys ser-phe-phe-ala-ile-ile-trp-leu-asp-met-pro-arg glu-tyr-ile-lys arg
chymotrypsin
(b) gly-lys-ser-phe-phe-ala-ile-ile-trp-leu-asp-met-pro-arg-glu-tyr-ile-lys-arg

gly-lys-ser-phe phe ala-ile-ile-trp leu-asp-met-pro-arg-glu-tyr ile-lys-arg


carboxypeptidase B
(c) gly-lys-ser-phe-phe-ala-ile-ile-trp-leu-asp-met-pro-arg-glu-tyr-ile-lys-arg
gly-lys ser-phe-phe-ala-ile-ile-trp-leu-asp-met-pro-arg glu-tyr-ile-lys arg

Staphylococcal
protease
(d) gly-lys-ser-phe-phe-ala-ile-ile-trp-leu-asp-met-pro-arg-glu-tyr-ile-lys-arg
gly-lys-ser-phe-phe-ala-ile-ile-trp-leu-asp met-pro-arg-glu tyr-ile-lys-arg
O
O O O
H H
MeO N CH C N CH C N CH C OEt
H
72. CH2 CH2 CH CH3
OH CH2
CH3

73. The ethyl ester of 2-bromo-2-methylbutanoic acid is a tertiary halide, and the reaction with
phthalimide anion is a SN2 reaction. The activation energy to attain the pentacoordinate SN2 transition
state is too high for this reaction to occur.
O EtO O S N2 is not
possible
N:–
Br

74.
Ph O NH2 Ph N CO2Et 1. H2 , Pd-C Ph HN CO2H

CO2Et 2. H3 O+
H H

75.
O
O O H2N
H2N H2N NH2
NH2 NH2 S
S S O
O O
N NH
Ph NH OH NH N NH
Ph HO HN
76.
CO2H O
O
Ac2O
CO2 1. Ph Cl NaOAc
NH O
2. aq H+ Ph
NH3
N Ph
O
O O O
pyridine O
O O H3O+
H
Ph NH3
N Ph O N

O O O
pyridine
O H3 O+ O
O
H
Ph NH3
N Ph
Ph O N
O Ph O
O
pyridine H3 O+
O O
H O
N Ph N
Ph O Ph NH3
Ph
Ph Ph
Ph Ph

Spectroscopic problems. Do not attempt these problems until Chapter 14 is read and understood.
These problems involve amines, amino acids, and derivatives of amino acids.

77.
NH2
H
N IR: The secondary amine will have
Me one peak at about 3300 cm –1,
whereas the primary amine will have two. 2-Aminopentane
N-Methyl-1-aminobutane

triplet methyl + No N-methyl, but a doublet


N-methyl at 3.3 ppm methyl + a triplet methyl

4 2 0 6 4 2 0
Note: C—N downfield PPM PPM
carbon missing
in the other

40 20 0 40 20 0
PPM PPM
78. In the IR, 4-aminobutanoic acid will have the broad peak associated with the COOH unit at 2500-
3000 cm-1, and the amino protons will be buried in this signal. In addition, there will be the carbonyl at
1725 cm-1. This sharply contrasts with 4-cyanobutan-1-ol, with a broad signal at 3300 cm-1 for the OH
plus a sharp signal at ~ 2240 cm-1 for the nitrile. The 1H NMR data for the two compounds are very
different. The amino acid has the acid peak at ~ 12 ppm in the 1H NMR, and a carbonyl carbon at ~180
ppm in the 13C NMR (assuming the amino acid and not the zwitterion). The cyano–alcohol has the
nitrile carbon at ~ 130 ppm, a doublet methyl in the 1H NMR and a triplet methylene downfield at ~3.8
ppm, due to the CH2—O unit.

CO2H OH
CO2H

NH2 CN CN

10 5 0 4 2 0
PPM
PPM

150 100 50 0 100 50 0


PPM PPM
79. Decarboxylation must occur at the malonic acid stage, before the reaction with phthalimide.
Bromination of the malonate will give a tertiary bromide, which cannot react with phthalimide via
substitution. Therefore, the question asks for differences between A and B. The most obvious
difference is the number of carbon atoms in A, one more than B. There are also differences in the sp3
carbon atoms in A and B, where A has the methine carbon further downfield. Likewise, there are
differences in the methine-methylene pattern in A and the methylene–methine pattern in B.
O O O O

O O O O EtO OEt EtO OEt


Br
HO OH EtO OEt
Cannot do a
SN2 because it is
O O O O quaternary. Must
decarboxylate first
Br Br
HO OH OH OEt
Br

A B
O O
O O O
Br N:– H2N
OEt N OEt OH
O

A B

10 5 0 10 5 0
PPM PPM

150 100 50 0 150 100 50 0


PPM PPM
80.
1. HBr
2. NaN3 CO2H
CO2H
3. H2 , Pd/C
NH2 NH2

CO2H MINOR

Doublet methyl
No methyl groups

10 5 0
PPM
10 5 0
PPM

methyl + 3
methylene C

all methylene C
150 100 50 0
PPM

150 100 50 0
PPM
81. It is a bit difficult to see in the 13C NMR spectrum on the bottom left, but there are three carbonyl
peaks, and only two in the spectrum on the bottom right. This clear difference would allow one to
distinguish these two compounds. There are other subtle differences as well.
O O IR is not much O O
H help as these EtHN H
AcHN CH C N CH C OEt CH C N CH C NMe2
spectra are
CH3 H similar CH3 H
N-Acetyl ethyl ester of ala-gly N-Ethyl-dimethyl amide of ala-gly

8 6 4 2 0 8 6 4 2 0
PPM PPM

150 100 50 0 150 100 50 0


PPM PPM

O O

H
O N

H
82. N-Formyl-2-phenylalanine methyl ester.

OH
O
OH
N
H
83. DL-Alanyl-DL-serine NH2 O
O

84. Ethyl L-leucinate NH2

O O

O O

85. Dimethyl 2-bromomalonate Br

O
O
NH O
N
H
86. Ethyl gly-val (N-acetyl) OEt
Chapter 28
Solutions Manual
Organic Chemistry. An Acid-Base Approach 2nd Edition
30.
O H (R) CH2OH CO2H CO2H CHO
(CHOH) n (CHOH) n (CHOH) n (CHOH) n
(CHOH) n
CH2OH CH2OH CO2H CO2H
CH2OH
Glycose Glycitol Glyconic acid Glycaric acid Uronic acid
OH OH OH
OH
(a) (b) (c)
HO CHO HO OH HO CO2H
OH OH OH
OH OH OH

(d) CO2H (e) HO2C CHO


HO2C
OH OH OH OH

31.
OH OH O OH

(a) (b) HO OH (c) CHO


HO CHO HO
OH OH OH OH
OH OH OH OH OH
CHO
(d) CHO (e) HO (f) HO
HO OH
OH
OH OH OH OH OH O

32.
OH OH
H OH
H OH H OH (d) O
HO
(a) OH (b) (c) O OH
O O OH OH OH H
OH OH
OH OH OH
L-Threose L-Ribose D-Xylose L-Allose
OH OH OH OH OH OH

(e) O (f) O (g) O


HO HO HO
OH OH H OH OH H OH OH H
L-Mannose D-Idose L-Talose
33.

OH OH
H + H+
H -H+
O + H+ O O HOEt
Bu H Bu H -H+
O +H+ O
Bu H Bu H Bu H Et H Et
Hemi-acetal
H H Et H Et
O - H2O Bu H O O
+ EtOH - H+
Bu H
Bu H O
Et O - EtOH Bu H + H+ Bu H
O Et O O
Et Et 50 Et
Acetal

34.
O OH OH OH
O HO O
(a) (b) (c)
HO OH HO OH HO OH
OH HO HO
α- D-Arabinopyranose α-L-Xylofuranose β- D-Allofuranose
O OH O OH OH
HO HO HO O
(d) (e) (f)
HO OH HO OH HO OH
OH OH HO
β-L-Mannopyranose α-D-Idopyranose β-L-Talofuranose

35. %α (+90) + %β (-38) = +18, and since %α + %β = 1, %α = 1-%β.


Therefore, (1-%β)(+90) + %β (-38) = +18 = 90 - 90%β - 38%β = +18, and %β (-128) = 18 -90,
%β (-128) = -72, so %β = -72 / -128 = 0.56. Therefore,
%β = 56% and %α = 44%.

36. The anomeric is not effective on C3, only on C2, so the equatorial conformation on the right should
be favored.
O

O
OMe OMe
37.
O HO OH HO HO OH
OH
OH OH
(a) (b) (c) (d)
HO HO OH HO HO OH
HO HO OH HO
OH O O O
α-L-Ribulofuranose α- D-Psicopyranose β-L-Fructofuranose β- D-Tagatopyranose

38.
HO OH OH
HO
α- D-Glucopyranose β- D-Allopyranose
O β- D-Galactopyranose
O OH O O
(a) HO (b) HO O
O
OH OH
HO OH HO OHHO
OH β- D-Altropyranose
OH
O-α- D-Glucopyranosyl-(1→1)-β- D-allopyranose O-β- D-Altropyranosyl-(1→4)-β- D-galactopyranose
HO
HO OH
α- D-Glucopyranose HO
OH
O O α- D-Mannopyranose
(c) (d) O O O
HO O HO
OH
HO OH HO OH
HO OH
α- D-Talopyranose OH β- D-Idopyranose
OH
O-α- D-Talopyranosyl-(1→1)-α- D-glucopyranose O-β- D-Idopyranosyl-(1→4)-α- D-mannopyranose

39.
HO

HO α- D-Lyxopyranose O α- D-Xylopyranose
OH
OH O HO
O HO
O O O
(a) O (b) HO
OH
O OH
O α- D-Altropyranose
HO HO O
HO OH α- D-Fructopyranose
OH OH OH
α- D-Fucofuranose α- D-Ribulofuranose OH
HO
O-α- D-Fuc f-(1→4)-α- D-Lyx p-(1→1)-α- D-Alt p HO O-α- D-Rib f-(1→4)-α- D-Xyl p-(1→1)-α- D-Fru p

HO α- D-Allopyranose

(c)
O HO
OH
O
HO O
O α- D-Idopyranose
α- D-Altrofuranose HO OH
HO O
OH
HO
OH
O-α- D-Alt f-(1→4)-α- D-All p-(1→1)-α- D-Ido p
40.
HO HO
HO HO
HO O HO O
O AgNO3 O
(a)
HO O NH3 HO OH
OH HO OH HO CO2–Ag+
α- D-Gulopyranose
α- D-Arabinofuranose OH
OH OH HO
OH
HO
HO
O aq CuSO4
(b) O O
OH
HO O Na tartrate
HO O CO2–Na+
α- D-Fucofuranose OH
OH HO
α- D-Mannopyranose OH HO
OH
OH
HO HO OH AgNO3
O No reaction
(c) OH
NH3
O O
HO
OH α- D-Galactopyranose
α- D-Galactofuranose
OH OH
O CO2–Na+
aq CuSO4
(d)
Na tartrate HO OH
HO OH HO
HO α- D-Arabinofuranose
O OH OH
HO AgNO3 HO CO2–Na+
(e) NH3
HO OH HO OH
OH α-D-galactopyranose OH
O OH OH
aq KMnO4 CO2H
(f)
HO OH HO OH
OH α- D-Lyxopyranose OH
O OH OH OH
1. NaBH4
(g)
2. aq NH4Cl
HO OH HO OH
α- D-Rhamnopyranose OH
OH

OH HO OH OH
HO O Na/Hg
(h)
HO OH
OH
OH HO α−D-Talofuranose HO

O OH OH
HO Br2 , pH 5 HO CO2H
(i)
HO OH HO OH
OH α- D-Allopyranose OH
OH OH
O OH H2 , Ni(R) HO
HO
(j)
HO OH
HO OH
OH
OH α-D-Glucopyranose

O OH OH
HO HNO3 HO CO2H
(k)
HO OH HO OH
OH α−D-Idopyranose
OH

41. Compounds (a) - (f) are all reducing sugars.


42. Draw the major product for each of the following reactions.
O OH O OAc
HO Ac2O , NaOAc AcO
(a)
HO OH AcO OAc
α- D-Glucopyranose
OH OAc

O OH O OEt
HO Et2SO 4 EtO
(b)
HO OH EtO OEt
OH α- D-Altropyranose
OEt
O OH O OEt
HO EtOH , H+ HO
(c)
HO OH HO OH
α−D-Talopyranose
OH OH

HO OH O OH
O O
acetone , H+
(d)
O
HO OH OH
HO α−D-Idofuranose HO

O OH
O OH
PhCHO , cat H+
(e)
O OH
HO OH
O
OH α−D-Arabinopyranose
Ph
43.
OH OH 1. NH2OH OH OH
2. Ac2O , ZnCl2
(a) CHO
3. NH3 4. aq H+ CHO
OH OH OH
OH OH OH 1. NH2OH OH OH
2. Ac2O , ZnCl2
(b) CHO
CHO 3. NH3 4. aq H+
OH OH OH OH

OH OH 1. NH2OH OH OH
2. Ac2O , ZnCl2
(c) CHO
3. NH3 4. aq H+ CHO
OH OH OH
OH OH OH 1. NH2OH OH OH
2. Ac2O , ZnCl2
(d) CHO
CHO 3. NH3 4. aq H+
OH OH OH OH

OH OH OH 1. NH2OH OH OH
2. Ac2O , ZnCl2
(e) CHO
CHO 3. NH3 4. aq H+
OH OH OH OH
OH OH OH 1. NH2OH OH OH
2. Ac2O , ZnCl2
(f) CHO
CHO 3. NH3 4. aq H+
OH OH OH OH

44.
AcO HO

OH O
Br AcO O
HO
H+
1. Ac2O , OAc
O O H3O+ OH
2. HBr , AcOH 1. Cl3CN
OH OAc 2. AcO O
OAc OAc O
O OH
AcO HO O
OH OH OAc O

AcO
HO
OAc OAc
AcO HO OH
AcO O-α- D-Ara f-(1→4)-α- D-Ido p
45.
O O NH2 O
N Me H N
NH N N NH
(a) OH (b) OH (c) OH (d) OH
N N NH2 N O N N N O
O O O O
H H H H H H H H
H H H H H H H H
OH OH OH OH OH OH OH OH

9-(β- D-Ribofuranoyl)guanine 1-(β- D-Ribofuranoyl)thymine 9-(β- D-Ribofuranoyl)adenine 1-(β- D-Ribofuranoyl)uracil

46.
A A G T G U C T T A
(a) CH2 CH2 CH2 CH2 (b) OH OH OH OH OH OH
3' 3' 3' 3' 3' 3' 3' 3' 3' 3'
P P P P OH P P P P P P OH
5' 5' 5' 5' 5' 5' 5' 5' 5' 5'
G C A C C
(c) OH OH OH OH OH
3' 3' 3' 3' 3'
P P P P P OH
5' 5' 5' 5' 5'

47.
NH2

N
N

N NH2
O N
(a) -O P O N
O N
O- H H
N NH2
H HO N
OH
O P O N
O
O- H H
H HO N O
OH
O P O
O
O- H H
H H
OH OH
O
(b)
NH
O
-O N O O
P O
O
O- H H N
NH
H HO
OH
N N NH2 O
O P O
O
H H Me
O- NH
H H
OH O
O
O P O N O
Me O
NH O- H H
(c)
O H H
O OH OH
O P O N O
O Me
O- H H NH
H HO
OH O
O P O N O
O Me
O- H H NH
H HO
OH
O P O N O
O
O- H H
H H
OH OH

48. It depends on whether it is the α or the β sugar precursor. At best the α OH will be converted to the
α-methoxy compound and the β OH will be converted to the β-methoxy compound. At worst,
mutarotation of the sugar will lead to a mixture of α and β methoxy compounds.

49. Presumably, there will an anomeric effect with the nitrogen, probably not as prominent as with the
oxygen substituent, but it is anticipated that the amine will prefer the axial position.
O OH O NH2
HO HO O
HO OH
HO
HO OH HO OH
HO NH2
OH OH
50.
O OH O OAc O Br
Ac2O , NaOAc HBr

HO OH AcO OAc AcO OAc


OH OAc OAc
O N3 O NH2 O NH2
NaN3 H2 , Pd H3O+
THF AcO OAc AcO OAc HO OH
OAc OAc OH

51.
OH OH O OH OH OH
Na(Hg) NaBH4

HO OH HO OH HO OH
OH OH OH

OH OH OH O OH OH
52. AgNO3 , NH3
O
HO
KMnO4
OH OH OH OH OH OH

53.
O NH2
NH2
F H OH
NH N N
N
N
N O N O O N
(a) (b) (c) N (d) –O PO
Cl N 3 O
HO
–O
3PO
–O
3PO
O H H
O O
H H H H
H H H H
H H OH OH
H H H H OH OH
OH OH OH OH
Ph N Ph Cl Ph
O
N N O N
O N O N (g) –O3PO N
(e) –O3PO (f) –O (h) –O3PO N (i) –O3PO
O 3PO O O O O
H H H H H H H H H H
H H H H H H H H H H
OH OH OH OH OH OH OH OH OH OH
54. %α (+112) + %β (+19) = 0.8(+112) + 0.2(+19) = 89.6 + 3.8 = 93.4°
%α (+112) + %β (+19) = +53, and since %α + %β = 1, %α = 1-%β.
Therefore, (1-%β)(+112) + %β (+19) = +53 = 112 - 112%β +19%β = +53, and %β (-93) = 53 - 112,
%β (-93) = -59, so %β = -59 / -93 = 0.63. Therefore, %β = 63% and %α = 37%.

55. %α (+90) + %β (-38) = +18, and since %α + %β = 1, %α = 1-%β.


Therefore, (1-%β)(+90) + %β (-38) = +18 = 90 - 90%β - 38%β = +18, and %β (-128) = 18 -90,
%β (-128) = -72, so %β = -72 / -128 = 0.56. Therefore, %β = 56% and %α = 44%.
OH O OH
(S) H3C
O (R) (R)
HO (S) (S)
β- D-Rhamnose (R) (S)
OH (S)
HO HO OH
(S) (R)
OH

56.
OH OH O
OH O
HO HO
HO
α-Xylopyranose α-Arabinopyranose
HO HO
OH HO OH O
OH HO
HO OH O OH
Major at equilbrium Major at equilbrium
57. The base-paired anti-parallel strands for A-G-G-T-A are:
OH OH
H H
H H O-
O O P O
N N
O
N OH
N H H
H H O-
NH2 O
O N O P O
NH2 O OH
HN H H
N H H O-
N
O H2N N O
N O P O
–O
3PO
N N O
O O OH
H H HN H H
N N
H H NH H H O-
OH O O O
H2N N N O P O
O P O N N NH2
O O O OH
O- H H HN H H
N N
H H NH H H
OH O O
O N N OPO3 –
O P O N N NH2 O
O
H H N
O- N
H H NH
OH O NH2
O P O N O
O NH2
O- H H
H H N
OH N
O
O P O N N
O
O- H H
H H
OH OH

58.
N O O N O
O O O
H2N N P P H2N N P
O O P O O
HO O O HO
HO HO
N N OH N N OH
HO HO
Adenosine triphosphate Adenosine monophosphate
O H O H
N N
O O
O O O
N O N O
P P P
O O O O
HO O HO
HO
HO OH HO OH
uracil diphosphate
uracil monophosphate
O
H
N
N
O
N N
59. -O P O
O
O- H H
H H
OH OH

O Me
O Me Me N
Me N
N
N
HO N
O N
60. N O
O N
H H
Me
H H
OH OH
Caffeine
1,3,7-Trimethyl-1 H-purine-2,6(3H,7H)-dione

Spectroscopy Problems. Do not attempt these problems until the concepts in Chapter 14 have
been mastered.
These problems involve carbohydrates or nucleotide derivatives.

61.
OH OH O OH
HO
glucose O
HO glucopyranose
HO OH
IR: Aldehyde H at 2815 cm–1 OH OH H
OH

aldehyde H

10 5 6 4 2 0
PPM PPM

aldehyde C

200 100 100 50


PPM PPM
62.
NH2 NH2

N N
N N

HO N HO N
N N
O O
H H H This CH2 will
H H H H show as a unique
OH OH OH
signal

8 6 4 2 0 8 6 4 2 0
PPM PPM

150 100 50 0 150 100 50 0


PPM PPM
The signals at about 2.3 in the deoxy compound constitute what is known as a doublet of
doublets, but this special case is known as an AB quartet. The adjacent carbon atoms have stereogenic
centers, which makes the two hydrogen atoms of the methylene unit diastereotopic (they are different,
and give different signals). This type of signal was not discussed n Chapter 14.

OH OH
OH
HO

63. D-Glucitol OH OH

O OH

HO OH

64. β-D-Xylose OH
OH

OH

N O

O N O
65. Thymidine H

O O
HO

HO OH

66. Methyl α-D-glucopyranoside OH

You might also like